Document PDF
Document PDF
Document PDF
PATRICE LASSÈRE
UNIVERSITÉ PAUL SABATIER
Laboratoire de Mathématiques Émile Picard, UMR CNRS 5580,
31 062, TOULOUSE Cedex 4, FRANCE. [email protected]
j MODE D’EMPLOI j
Table des matières
partie 2. POLYNÔMES 49
Sur les polynômes de la forme P = QP 00 avec deg(Q) = 2 . . . . . . . . . . . . . . . . . . . . . . . . . . . 50
Trois exercices sur les polynômes . . . . . . . . . . . . . . . . . . . . . . . . . . . . . . . . . . . . . . . . . . . . . . . . . . . 50
Polynômes harmoniques et homogènes en deux variables . . . . . . . . . . . . . . . . . . . . . . . . . . . . 52
Polynomes et fractions rationelles, approximation . . . . . . . . . . . . . . . . . . . . . . . . . . . . . . . . . . 53
Polynômes, nombres premiers . . . . . . . . . . . . . . . . . . . . . . . . . . . . . . . . . . . . . . . . . . . . . . . . . . . . . . 54
Polynômes dans Z[X] . . . . . . . . . . . . . . . . . . . . . . . . . . . . . . . . . . . . . . . . . . . . . . . . . . . . . . . . . . . . . . 55
Polynômes trigonométriques : un théorème de Fejèr-Riesz . . . . . . . . . . . . . . . . . . . . . . . . . . . 55
Autour du résultant de deux polynômes . . . . . . . . . . . . . . . . . . . . . . . . . . . . . . . . . . . . . . . . . . . . . 57
Le théorème de Gauss-Lucas . . . . . . . . . . . . . . . . . . . . . . . . . . . . . . . . . . . . . . . . . . . . . . . . . . . . . . . . 58
Le théorème de Gauss-Lucas : nouvelle approche . . . . . . . . . . . . . . . . . . . . . . . . . . . . . . . . . . . . 59
Une inégalité autour des polynômes . . . . . . . . . . . . . . . . . . . . . . . . . . . . . . . . . . . . . . . . . . . . . . . . 60
Encore un calcul de ζ(2) . . . . . . . . . . . . . . . . . . . . . . . . . . . . . . . . . . . . . . . . . . . . . . . . . . . . . . . . . . . 60
Nombre de racines réelles du 2005-ième itéré de P (x) = x2 − 1 . . . . . . . . . . . . . . . . . . . . . 61
Racines de P (z) et de 2zP 0 (z) − dP (z) . . . . . . . . . . . . . . . . . . . . . . . . . . . . . . . . . . . . . . . . . . . . . 62
Un polynôme de degré 6 et un peu de géométrie . . . . . . . . . . . . . . . . . . . . . . . . . . . . . . . . . . . . 63
Sur les racines multiples du polynôme dérivé . . . . . . . . . . . . . . . . . . . . . . . . . . . . . . . . . . . . . . . 64
partie 3. GÉOMÉTRIE 65
Optimisation dans un triangle . . . . . . . . . . . . . . . . . . . . . . . . . . . . . . . . . . . . . . . . . . . . . . . . . . . . . . 66
Sur la longueur de l’intersection entre une parabole et un disque . . . . . . . . . . . . . . . . . . . . 66
Inégalités dans un triangle (1) . . . . . . . . . . . . . . . . . . . . . . . . . . . . . . . . . . . . . . . . . . . . . . . . . . . . . . 68
Inégalités dans un triangle (2) . . . . . . . . . . . . . . . . . . . . . . . . . . . . . . . . . . . . . . . . . . . . . . . . . . . . . . 69
Coniques : le théorème de Joachimsthal . . . . . . . . . . . . . . . . . . . . . . . . . . . . . . . . . . . . . . . . . . . . . 69
Heptadivision d’un triangle . . . . . . . . . . . . . . . . . . . . . . . . . . . . . . . . . . . . . . . . . . . . . . . . . . . . . . . . . 70
Disposition de n points sur une sphère . . . . . . . . . . . . . . . . . . . . . . . . . . . . . . . . . . . . . . . . . . . . . 73
Une suite associée à un polygône . . . . . . . . . . . . . . . . . . . . . . . . . . . . . . . . . . . . . . . . . . . . . . . . . . . 73
Sur la longueur de l’ellipse . . . . . . . . . . . . . . . . . . . . . . . . . . . . . . . . . . . . . . . . . . . . . . . . . . . . . . . . . 74
Même périmètre et même aire . . . . . . . . . . . . . . . . . . . . . . . . . . . . . . . . . . . . . . . . . . . . . . . . . . . . . . 75
Deux inégalités . . . . . . . . . . . . . . . . . . . . . . . . . . . . . . . . . . . . . . . . . . . . . . . . . . . . . . . . . . . . . . . . . . . . 76
ii
partie 4. COMBINATOIRE ET PROBABILITÉS 79
Combinatoire : les nombres de Bell . . . . . . . . . . . . . . . . . . . . . . . . . . . . . . . . . . . . . . . . . . . . . . . . . 80
Un peu de dénombrement autour d’une série entière . . . . . . . . . . . . . . . . . . . . . . . . . . . . . . . . 81
Autour du « nombre de dérangements » . . . . . . . . . . . . . . . . . . . . . . . . . . . . . . . . . . . . . . . . . . . . 82
Distance entre deux racines d’un polynôme . . . . . . . . . . . . . . . . . . . . . . . . . . . . . . . . . . . . . . . . . 85
Distribution de deux points sur un segment (1) . . . . . . . . . . . . . . . . . . . . . . . . . . . . . . . . . . . . . 85
« Probabilité » que deux entiers soient premiers entre-eux . . . . . . . . . . . . . . . . . . . . . . . . . . 86
Nombre de matrices symétriques à coefficients dans {0, 1} et série entières . . . . . . . . . . 89
Distribution aléatoire de deux points sur un segment . . . . . . . . . . . . . . . . . . . . . . . . . . . . . . . 89
Dénombrement et séries entières/génératrices . . . . . . . . . . . . . . . . . . . . . . . . . . . . . . . . . . . . . . . 90
Groupes et probabilités . . . . . . . . . . . . . . . . . . . . . . . . . . . . . . . . . . . . . . . . . . . . . . . . . . . . . . . . . . . . 91
Dénombrement et algèbre linéaire . . . . . . . . . . . . . . . . . . . . . . . . . . . . . . . . . . . . . . . . . . . . . . . . . . . 91
Les dés sont pipés . . . . . . . . . . . . . . . . . . . . . . . . . . . . . . . . . . . . . . . . . . . . . . . . . . . . . . . . . . . . . . . . . . 92
Avec un peu d’algèbre linéaire . . . . . . . . . . . . . . . . . . . . . . . . . . . . . . . . . . . . . . . . . . . . . . . . . . . . . 94
Probabilité d’obtenir un multiple de cinq en jetant n dés . . . . . . . . . . . . . . . . . . . . . . . . . . . 95
Combinatoire et matrices . . . . . . . . . . . . . . . . . . . . . . . . . . . . . . . . . . . . . . . . . . . . . . . . . . . . . . . . . . 96
102006 divise n!. . . . . . . . . . . . . . . . . . . . . . . . . . . . . . . . . . . . . . . . . . . . . . . . . . . . . . . . . . . . . . . . . . . . . . 97
Dénombrement dans les groupes . . . . . . . . . . . . . . . . . . . . . . . . . . . . . . . . . . . . . . . . . . . . . . . . . . . . 97
partie 5. ANALYSE 1 99
Deux fonctions f, g dérivables telles que f 0 g 0 ne soit pas une dérivéee . . . . . . . . . . . . . . . 161
Dérivation . . . . . . . . . . . . . . . . . . . . . . . . . . . . . . . . . . . . . . . . . . . . . . . . . . . . . . . . . . . . . . . . . . . . . . . . . 162
Approche matricielle du théorème des accroissements finis. . . . . . . . . . . . . . . . . . . . . . . . . . . 163
Comportement asymptotique du « point intermédiaire » dans la formule de
Taylor-Lagrange . . . . . . . . . . . . . . . . . . . . . . . . . . . . . . . . . . . . . . . . . . . . . . . . . . . . . . . . . . . . . . . . . . . . 164
Trois preuves du théorème de Darboux. . . . . . . . . . . . . . . . . . . . . . . . . . . . . . . . . . . . . . . . . . . . . . 164
Sur le point d’inflexion. . . . . . . . . . . . . . . . . . . . . . . . . . . . . . . . . . . . . . . . . . . . . . . . . . . . . . . . . . . . . . 166
iv
Rolle sur R . . . . . . . . . . . . . . . . . . . . . . . . . . . . . . . . . . . . . . . . . . . . . . . . . . . . . . . . . . . . . . . . . . . . . . . . . 166
Dérivabilité et accroissements finis . . . . . . . . . . . . . . . . . . . . . . . . . . . . . . . . . . . . . . . . . . . . . . . . . . 166
Dérivabilité et accroissements finis . . . . . . . . . . . . . . . . . . . . . . . . . . . . . . . . . . . . . . . . . . . . . . . . . . 167
Toute application convexe et majorée sur R est constante . . . . . . . . . . . . . . . . . . . . . . . . . . . 167
Régularité et existence de développement limité en un point. . . . . . . . . . . . . . . . . . . . . . . . . 168
Parité, dérivabilité et développement limité . . . . . . . . . . . . . . . . . . . . . . . . . . . . . . . . . . . . . . . . . 169
Convexité et Accroisements Finis . . . . . . . . . . . . . . . . . . . . . . . . . . . . . . . . . . . . . . . . . . . . . . . . . . 169
Zéros des dérivées d’une fonctions C ∞√à support compact . . . . . . . . . . . . . . . . . . . . . . . . . . 170
Sur l’inégalité de Kolmogorov M1 ≤ 2 M0 M1 . . . . . . . . . . . . . . . . . . . . . . . . . . . . . . . . . . . . . . 172
Une série et une fonction . . . . . . . . . . . . . . . . . . . . . . . . . . . . . . . . . . . . . . . . . . . . . . . . . . . . . . . . . . . 172
Un
Pn fameuxk+1 théorème d’Émile Borel . . . . . . . . . . . . . . . . . . . . . . . . . . . . . . . . . . . . . . . . . . . . . . . . 173
k=1 (−1) Cnk k n = (−1)n+1 n! . . . . . . . . . . . . . . . . . . . . . . . . . . . . . . . . . . . . . . . . . . . . . . . . . . . . 175
Rπ 276
Calcul de 0 cos(cos(x))ch(sin(x)) cos(nx)dx, n ∈ N, via Fourier . . . . . . . . . . . . . . . . . . . . . 277
Preuve
Pn du théorème des moments de Hausdorff par les séries de Fourier . . . . . . . . . . . . 278
2k+1 2n+1
k=0 C2n+2 = 2 (2n + 1)(2n + 2) via les séries entières . . . . . . . . . . . . . . . . . . . . . . . . . . 278
ix
.................................................................................... 400
.................................................................................... 400
.................................................................................... 400
.................................................................................... 400
.................................................................................... 400
.................................................................................... 400
.................................................................................... 400
.................................................................................... 400
.................................................................................... 401
Bibliographie 403
Index 405
x
Première partie
ALGÈBRE LINÉAIRE ET
MULTININÉAIRE
CHAPITRE 1
ALGÈBRE LINÉAIRE
Ê H est un hyperplan de Mn (C) : il existe donc une forme linéaire non nulle ϕ sur Mn (C)
telle que H = ker(ϕ). Tous les éléments Ei,j = ((δi,j k,l
))k,l de la base canonique de Mn (C)
sont, si i 6= j des matrices nilpotentes donc, vu les hypothèses, dans H . Dans ce cas, la
matrice
0 1 0 ... 0
.. . . . . . . ..
n−1 . . . . .
X . .. ..
A = En,1 + Ek,k+1 =
.
. . . 0
k=1
0 . .
. 1
1 0 ... ... 0
appartient à H comme combinaison linéaire d’éléments de H mais visiblement A ∈ GLn (C)
d’où le résultat.
ϕ(In )
t := ∈ C.
ϕ(N )
Si ϕ(In ) = 0 alors In ∈ H ∩ GLn (C) et le tour est joué. Sinon, puisque ϕ(In − tN ) = 0 i.e.
In − tN ∈ H , et comme N est nilpotente (N n = 0) on peut écrire
n−1
! n−1
!
X X
In = In − tn N n = (In − tN ) tk N k = tk N k (In − tN ).
k=0 k=0
Le résultat suivant est essentiel : soit M ∈ Mn (K) une matrice donc le polynôme caractéris-
tique est scindé et s’écrit χM = (X − λ1 )α1 . . . (X − λd )αd où les λi sont les valeurs propres
deux à deux distinctes de M de multiplicités αi ∈ N? . Alors M est diagonalisable si, et
seulement si, M annule (X − λ1 ) . . . (X − λd ).
Ici χM = (X − 1)2 (X − b)2 . Distinguons deux cas :
ê Si b = 1. M est diagonalisable si, et seulement si, M = I4 soit a = c = 0 et C = 0.
ê Si b 6= 1. Dans ce cas, M est diagonalisable si, et seulement si, (M −I4 )(M −bI4 ) = 0.
Un calcul direct montre que cette dernière condition équivaut à a = c = 0. En résumé, M
est diagonalisable si, et seulement si
(a = c = 0) et ( b 6= 1 ou C = 0) .
0 1 0 0
Supposons qu’il existe r ≥ 2 et A ∈ M2 (C) tels que Ar = 0 0 . Alors A2r = 0 0 , et le
2 2r
polynôme caractéristique χA (x) = ax + bx +c de A divise donc x ; ceci implique c = b = 0,
soit χA (x) = x2 et (Cayley-Hamilton) A2 = 00 00 . Ainsi (comme r ≥ 2)
0 1 r 2 r−2
0 0 =A =A A = 00 00
Comme Ker(A) doit être une droite (A n’est pas inversible et de rang supérieur à celui de
N , donc de rang 2) incluse dans le noyau de N = BA, Ker(A) est engendré par (1, 0, 0).
De même Im(B) doit être un plan qui doit contenir l’image de N i.e. le plan engendré par
(1, 0, 0) et (0, 1, 0).
0 a a0 d e f
On est donc amené à chercher sous les formes A = 0 b b00 et B = d0 e0 f0 . Un peu
0 0 0
0 c c
0 1 0 1 0 0
de calcul montre que A = 0 0 1 et B = 0 1 0 conviennent.
0 0 0 0 0 0
ê Pour n > 3 il suffit de border les matrices précédentes par des zéros. q
ce qui montre que ϕ ∈ E . L’algèbre E est donc constituée des endomorphisme de Mn (R) qui
laissent stable les sous-espaces Sn et An . Il existe donc un isomorphisme évident
E −→ L (Sn ) × L (An )
qui à ϕ associe le couple ϕ/Sn , ϕ/An . Il en résulte que
2 2
n2 (n2 + 1)
n(n + 1) n(n − 1)
dim(E ) = + = .
2 2 2
q
i Remarque : on retrouve dans la dernière formule le résultat connu mais plus délicate
à établir, à savoir que la dimension du commutant est toujours une somme de carrés p21 +
· · · + p2k ∈ {d, d + 1, . . . , d2 ) (ici d = n2 ) telle que p1 + · · · + pk = d.
Nous allons vérifier que cette image est constituée de toutes les matrices A ∈ M3 (R) sauf
celles admettant 0 comme valeur propre multiple et qui ne sont pas diagonalisables.
Soit B ∈ M3 (R) et cherchons A ∈ M3 (R) vérifiant ϕ(A) = A3 . On distingue plusieurs cas
pour B
ê B est diagonalisable sur R.
Il existe alors une base {u, v, w} de R3 , des réels α, β, γ tels que
Bu = αu, Bv = βv, Bw = γw.
λ, µ, ν ∈ R désignant les racines cubiques des valeurs propres α, β, γ, définissons la matrice
A ∈ M3 (R) par
Au = λu, Av = µv, Aw = νw.
A est bien réelle et vérifie A3 = B et B admet bien un antécédent par ϕ.
ê B possède une valeur propre non réelle.
Les valeurs propres de B sont donc un réel α et deux complexes non réels conjugués ω et ω.
Il existe un vecteur réel non nul u et un vecteur complexe z non nul tels que
Bu = αu, Bz = ωz, et par suite, Bz = ωz.
Notons λ le réel racine cubique de α et soit θ une racine cubique de ω. La matrice A définie
dans la base (u, z, z) de C3 par
Au = λu, Az = θz, Az = θz
vérifie A3 = B. En outre A envoie u sur λu, z sur θz et z sur θz. A est donc réelle et et B
admet bien un antécédent par ϕ.
ê B possède une valeur propre réelle non nulle λ d’ordre 2 et n’est pas diagonalisable.
Les valeurs propres de B sont λ, λ, µ où µ est un autre réel. Posons α = λ1/3 , β = µ1/3 , on
a donc
R3 = ker(B − λI3 )2 ⊕ ker(B − µI3 )
La dimension de ker(B − λI3 ) n’est pas deux sinon B serait diagonalisable, elle vaut donc
1. Puisque dim ker(B − λI3 )2 = 2, dim ker(B − λI3 ) = 1, considérons u ∈ ker(B − λI3 )2 \
ker(B − λI3 ), v = (B − λI3 )u et w un vecteur non nul de ker(B − µI3 ). on a ainsi construit
une base (u, v, w) de R3 qui vérifie
Bu = λu + v, Bv = λv, Bw = µw.
Une matrice A ∈ M3 (R3 ) vérifiant pour un certain réel c
Au = αu + cv, Av = αv, Aw = βw
vérifiera
A3 u = λu + 3α2 cv, A3 v = λv, A3 w = µw
1
de sorte que si c = : A3 = B.
3α2
ê B possède une valeur propre non nulle λ d’ordre 3 et n’est pas diagonalisable
Dans ce cas B = λ(I3 + N ) où N est une matrice nilpotente non nulle. Posons α = λ1/3
et cherchons A sous la forme A = α(I3 + M ) avec M nilpotente. On a M 3 = 0 donc
A3 = λ(I3 + 3M + 3M 2 ) et tout se ramène à l’équation 3M + 3M 2 = N qui est vérifiée par
1
M = (3N − N 2 ). Une fois de plus B admet un antécédent.
9
ê B admet 0 comme valeur propre d’ordre 2 ou 3 et n’est pas diagonalisable.
Si l’équation A3 = B admet une solution, A admet aussi comme vaelur propre d’ordre 2
ou 3. Si 0 est valeur propre d’ordre 3 alors A est nilpotente, A3 = 0, ce qui est absurde
puisque B n’est pas diagonalisable. Supposons donc que O soit valeur propre d’ordre 2 de A
et notons α l’autre valeur propre (réelle). R3 = ker(A2 ) ⊕ ker(A − αI3 ) mais A3 = B implique
Bx = 0, ∀ x ∈ ker(A2 ) soit R3 = ker(B) ⊕ ker(B − α3 I3 ) : B est alors diagonalisable ce qui
est exclut : tous les cas sont épuisés et la conclusion annoncée s’impose. q
P (A) AP 0 (A)
(♦) P (B) = , ∀ P ∈ C[X].
0 P (A)
En particulier, le polynôme minimal de B vérifie
µB (A) = 0
et
(8) µA divise µB .
Supposons B diagonalisable, µB ne possède que des racines simples et comme A et B ont
même ensemble valeurs propres avec (8) nous avons µA = µB qui avec (♦) implique
Aµ0A (A) = 0.
µA divise donc Xµ0A , les deux polynômes µA et Xµ0A étant de même degré d
dµA = Xµ0A
soit µA = X d . Mais µA = µB n’a que des racines simples, donc d = 1, µA (X) = X et
finalement A = 0. q
λ1 0 ... 0
0 λ2 ... 0 −1
A = G
... .. .. .. G
. . .
0 ... 0 λn
si bien que
PA (λ1 ) 0 ... 0
0 PA (λ2 ) . . . 0 −1
PA (A) = G
... .. .. .. G = 0.
. . .
0 ... 0 PA (λn )
L’application continue
M ∈ Mn (C) 7→ PM (M ) ∈ Mn (C)
est donc nulle sur Dn (C), partie dense de Mn (C) (c.f. exercice 69) : elle est donc identiquement verifier
nulle. q
Ë Preuve 2 : On cherche à montrer que PA (A)x = 0 pour tout vecteur x. Soit donc x un
vecteur non nul de Cn (si x = 0 il n’y a rien à démontrer), notons Ex le plus petit (au sens
de l’inclusion) sous-espace stable par A de Cn contenant x. Ex est de dimension 1 ≤ d ≤ n,
et admet pour base {x, Ax, . . . , Ad−1 x}.
Il existe donc an−1 , . . . , a1 , a0 dans C tels que
0 0 ... 0 a0
1 0 . . . 0 a1
0 1 ... 0 a2 .
CP = . .
.. . . . . . .. ..
. .
0 ... 0 1 an−1
on a donc PA (X) = PCP (X)PB (X), mais alors
ê Pour une autre démonstration moins classique voir l’exercice 94 et pour un joli bestiaire
de preuves de Cayley-Hamilton consulter /httpblabla?????????
πϕ (t) = t2 , 0 est donc valeur propre de ϕ : dim (ker(ϕ)) ≥ 1, de l’autre côté puisque
πϕ (t) 6= t, ϕ n’est pas l’endomorphisme nul i.e. dim (ker(ϕ)) ≤ 6 et enfin dim (ker(ϕ2 )) ≤ 7.
Soit ψ, la restriction de ϕ à ker(ϕ2 ), par le théorème du rang :
soit
dim (ker(ϕ)) ∈ {4, 5, 6}
et les trois exemples ci-dessous montrent que les valeurs 4, 5, 6 sont toujours possibles ({e1 , . . . , e7 }
désigne une quelconque base de notre espace) :
ϕ est bien entendu élément de L (Mn (R)). Soit λ une valeur propre de ϕ : il existe une
matrice A non nulle telle que
ϕ(A) = −A + tr(A)In = λA
si bien que
ntr(A)
A = (1 + λ)−1 tr(A)In =⇒ tr(A) = =⇒ λ = n − 1 & A ∈ vect(In )
(1 + λ)
i.e. λ = n − 1 est aussi valeur propre, et le sous-espace propre associé est la droite vectorielle
engendrée par In donc de dimension 1.
ê En conclusion, ϕ admet deux valeurs propres −1 et n − 1, les sous-espaces propres
associés sont de dimensions respectives n2 − 1 et 1 donc de somme n2 = dimMn (R) et ϕ est
2 2
diagonalisable (et son polynôme caractéristique Pϕ (x) = (−1)n (x + 1)n −1 (x + 1 − n)...).q
Dans l’espace vectoriel M2 (R) de dimension 4 il n’y a qu’une alternative : ou bien la famille
{A, B, C} est liée et il n’y a rien à démontrer, ou bien elle est libre et dans ce cas l’ensemble
vect{A, B, C} = {aA + bB + cC, (a, b, c) ∈ R3 }
sous-espace vectoriel de dimension 3 est alors tenu, pour des raisons évidentes de dimension,
de rencontrer le sous-espace
a b 2
, (a, b) ∈ R
0 a
de dimension 2 constitué de matrices à valeurs propres doubles.Le résultat suit. q
Par Dunford A = D+N avec D diagonalisable de mêmes valeurs propres que A, N nilpotente
et N D = DN . donc pour k ≥ n on aura N k = 0 et
k
X n
X n
X
k k
A = (D + N ) = Cnl Dk−l N l = Cnl Dk−l N l =D k−n
Cnl Dn−l N l .
l=0 l=0 l=0
n−l l
Soit, en posant α = sup0≤l≤n {kDk kN k }
n n
!
X X
kAk k ≤ kDkk−n Ckl kDkn−l kN kl ≤ αkDkk−n Ckl
l=0 l=0
or, pour 0 ≤ l ≤ n et k ≥ n
Ckl ≤ k(k − 1) . . . (k − l + 1) ≤ k l ≤ k n
si bien que
1 1 n 1
(F) ρ(A) ≤ kAk k k ≤ (α(n + 1)) k k k kDk−n k k ,
1 n
en remarquant enfin que lim (α(n + 1)) k k k = 1 et
k→∞
1
1
k−n
k 1
k−n k−n
lim kD k = lim kD
k k k−n = lim kDk−n k k−n = ρ(D) = ρ(A)
k→∞ k→∞ k→∞
1
k
avec (F), il vient finalement ρ(A) = lim kA k . k q
k→∞
Si une telle matrice existe la matrice Ak est encore à coefficients > 0 pour tout entier k ∈ N ;
mais A nilpotente implique (Cayley-Hamilton par exemple) An = 0, contradiction. q
Notons λ1 , . . . , λd les valeurs propres de A comptées avec leur multiplicités. A est diagona-
lisable dans C et pour tout entier k
d
X
k
tr(A ) = λkj
j=0
par conséquent, si les λj ne sont pas tous non nuls (i.e. A n’est pas nilpotente) le rayon de
convergence de notre série entière est supérieur ou égal (et en fait égal) à
A est annulée par P (X) = X N − 1 scindé à racines simples, elle est donc diagonalisable sur
C. Ses deux valeurs propres λ1 , λ2 sont racines de P donc des racines N -ièmes de l’unité.
Enfin, A étant à coefficients réels, si elle admet une valeur propre non réelle : les deux seront
conjuguées (λ1 = λ2 )
Si les valeurs propres sont réelles λ1 , λ2 ∈ {−1, +1} et A étant diagonalisable A2 = I2 .
Sinon, elles sont conjuguées, mais A étant à coefficients dans Z implique
i Remarque : En résumé, l’ordre d’une matrice A ∈ GLn (Z) appartient à {1, 2, 3, 4, 6, +∞}
et ces valeurs sont atteintes. Par exemple l’ordre des matrices ci-dessous
1 0 0 1 0 −1 0 −1 0 −1 1 1
, , , , ,
0 1 1 0 1 −1 1 0 1 1 0 1
est respectivement 1, 2, 3, 4, 6 et +∞.
sp(A) = {−1, 0, 1} et PA (X) = −X(X + 1)(X − 1) est scindé à racines simples : A est
diagonalisable.
0 n’est pas valeur propre de A : dans ce cas A ∈ GL3 (R) et alors
A4 = A2 =⇒ A2 = I2 ,
A annulée par un polynôme scindé à racines simples est encore diagonalisable (ce dernier cas
correspond au cas sp(A) = {−1, 1} et donc PA (X) = (X − 1)2 (X + 1) ou (X − 1)(X + 1)2 .
q
Seul le cas où A ∈ GLn (C) mérite explication. Pour une telle matrice désignons par C1 , . . . , Cn
ses colonnes.
ê Supposons les colonnes (Ci )n1 deux à deux orthogonales. Si
Di = kCi k−1 Ci , et B = [D1 , . . . , Dn ]
la matrice B est orthogonale donc |det(B)| = 1 ; vu que
q √
∀ 1 ≤ j ≤ n : kCj k = a21,j + · · · + a2n,j ≤ c n
on a
(4) |det(A)| = |det(B)| kC1 k . . . kCn k ≤ cn nn/2 .
soit
(8) kDk k ≤ kCk k, ∀1 ≤ k ≤ n
puisque bien entendu det[C1 , . . . , Cn ] = det[D1 , . . . , Dn ] ; l’inégalité est démontrée vu (8) et
(4). q
Soit n ∈ N? . Notons Zn la collection des zéros (comptés avec leurs multiplicités) de l’ensemble
Sn des polynômes de Sylvester de degré inférieur où égal à n.
ê Étape 1 : Pour n ∈ N? : sn := card(Sn ) est fini.
Soit p(z) = z n + an−1 z n−1 + · · · + a0 = ni=1 (z − ζi ) ∈ Sn , avec les formules de Newton, si
Q
0 ≤ k ≤ n − 1 on a
X X
|ak | = (−1)k ζi1 . . . ζik ≤ |ζi1 . . . ζik | ≤ Ckn ≤ n!,
1≤i1 <···<ik ≤n 1≤i1 <···<ik ≤n
les coefficients ak étant entiers, il n’y a qu’un choix fini de ak et sn est fini.
ê Étape 2 : ( ζ ∈ Zn ) =⇒ ζ k ∈ Zn , ∀ k ∈ N .
i Remarques : ê On peut tout aussi bien montrer qu’un polynôme de Sylvester est
sans
k
Qnzéros de module strictement compris entre 0 et 1 de la manière suivante : soit p(z) =
z i=k+1 (z − ζi ) ∈ Kn , toujours avec Newton : 1 ≤ |ak | = |ζk+1 . . . ζn | ≤ 1, soit |ζk+1 | =
· · · = |ζn | = 1.
ê On pourra aussi consulter les ouvrages de J.M.Arnaudies & J.Bertin "Groupes, Algèbre
et Géométrie" tome 1, pages 127-128, Ellipse, (1993) ou E.Leichnam "Exercices corrigés de
Mathématiques, Polytechnique, ENS" (Algèbre et Géométrie), exercice 1-30, Ellipse, (1999)
pour d’autres approches.
ê Un entier algébrique est une racine d’un polynôme unitaire P (X) = X d + ad−1 X d−1 +
· · · + a1 X + a0 ∈ Z[X]. On a
« Un entier algébrique est soit entier, soit irrationnel. »
Cette assertion repose essentiellement sur les idées précédentes, en voici donc les étapes
principales :
α ∈ C est un entier algébrique si, et seulement si, il est valeur propre d’une matrice à
coefficients entiers.
Si A ∈ Mn (Z) admet une valeur propre λ rationnelle, alors A admet un vecteur propre
à coordonnées entières associé à la valeur propre λ.
Soit α un entier algébrique racine d’un polynôme P ∈ Z[X] de degré n ≥ 2. Si α ∈ Q\Z
considérons k ∈ Z tel que k < α < k + 1 et A := CP − kIn ∈ Mn (Z). On vérifie facilement
que λ := α − k est valeur propre de A, puis en considérant la suite (Am X)m où X ∈ Zn
vérifie AX = λX, conclure.
Soit
(F) A = aE11 + bE12 + cE21 + dE22
la décomposition de A dans la base canonique de M2 (K).
Car pour n = 2, l’entier 3 est le seul élément de {2, 3, 4} qui ne vérifie pas ces deux propriétés.
Ce dernier résultat est délicat à établir 1 ; pour notre exercice, il est d’ailleurs plus rapide
(([10], 2000/01, ex. 15)) d’établir (c’est d’ailleurs un corollaire immédiat du précédent)
que la codimension du commutant est toujours paire ce qui n’est pas le cas de l’entier 3
(4 − 3 = 1...).
Posons A = {M ∈ Mn (R) | M 2 = In }. Nous allons montrer que les points isolés de A sont
In et −In . Le cas n = 1 étant trivial, on suppose dans la suite n > 2.
Soit M ∈ A. Le polynôme X2 − 1 = (X − 1)(X + 1) étant annulateur de M , la matrice
Ip 0
M est semblable à une matrice avec 0 6 p 6 n. On a
0 −In−p
tr M = 2p − n, tr M = n ⇔ M = In et tr M = −n ⇔ M = −In .
ê Solution 2 : Avec
∞
X (−1)n
cos(A) = A2n
n=0
(2n)!
on vérifie que
sin2 (A) + cos2 (A) = I2
si bien que
1 2005
sin(A) =
0 1
implique
2 0 −2.2005
cos (A) = .
0 0
Cette dernière équation implique que la matrice cos2 (A) est nilpotente, étant de taille 2 × 2
elle ne peut être que nulle ce qui est absurde. o
Í Montrer que On (R) (resp. Un (C)) est un sous-groupe compact maximal dans
GLn (R) (resp. GLn (C)) (commencer par montrer que les valeurs propres de tout
élément d’un sous-groupe compact de GLn (K) sont de module 1).
Ê On (R) est fermé dans Mn (R) car On (R) = ϕ−1 ({In }) où ϕ est l’application continue
sur Mn (R) définie par A 7→ t AA. |||.||| étant la norme sur Mn (R) subordonnée à la norme
euclidienne k.k de Rn , A ∈ On (R) implique kAXk = 1 et par suite |||A||| = 1. On (R) fermé
bornée dans Mn (R) est bien compact. La procédure est analogue pour Un (C).
Í À suivre...... o
Ë On vérifie facilement que fA est linéaire ; pour des raisons de dimension, il est donc
suffisant de montrer que l’application A 7→ fA est injective :
Soit A = ((ai,j )) telle que fA = 0. (Ei,j ) désignant la base canonique de Mn (K) on a pour
1 ≤ k, l ≤ n
!
X
0 = fA (Ek,l ) = Tr(AEk,l ) = Tr ai,j Ei,j Ek,l
1≤i,j≤n
n
!
X
= Tr ai,k Ei,l car Ei,j Ek,l = δj,k Ei,l
i=1
n
X
= ai,k Tr(Ei,l ) = al,k .
i=1
Et A est bien nulle.
Ì Soit f une telle forme et 1 ≤ i, j ≤ n, si i 6= j
f (Ei,j ) = f (Ei,i Ei,j ) = f (Ei,j Ei,i ) = 0,
et
f (Ei,i ) = f (Ei,j Ej,i ) = f (Ej,i Ei,j ) = f (Ej,j ) := λ.
Ainsi, f et λTr coïncident sur la base canonique de Mn (K) : elles sont égales.
i Remarque : On trouvera aussi dans [15], exercice 7.7 une autre démonstration s’ap-
puyant sur la première question.
Í Un hyperplan H de Mn (K) est le noyau d’une forme linéaire f non nulle ; il existe donc
A ∈ Mn (K) non nulle, telle que f = fA et H = {X ∈ Mn (K) : Tr(AX) = 0}. Il s’agit donc
de montrer qu’il existe X ∈ GLn (K) telle que Tr(AX) = 0. Pour cela, soit r ≥ 1 le rang de
A, il existe2 P et Q dans GLn (R) telle que
Ir 0
P AQ = Jr = .
0 0
Alors, pour X ∈ Mn (K), Tr(AX) = Tr(P −1 Jr Q−1 X) = Tr(Jr Q−1 XP −1 ) et il suffit donc
de trouver une matrice inversible Y telle que Tr(Jr Y ) = 0 (X = QY P répondra alors à la
question). Par exemple, la matrice de permutation
0 0 ... ... 0 1
...
1 0 0
0 1 0 0
Y = ..
... ... ..
. .
. . . . . ..
.. . . .
0 ... ... ... 1 0
convient puisque Jr Y est de diagonale nulle. o
i Remarque :
2donner une référence
i Nous restons ici fidèlement sur la trace de Francinou, Gianella et Nicolas [18] qui pro-
posent plusieurs solutions de ce classique problème.
Ê ê Sur C, le polynôme caractérique de A est scindé. Raisonnons par l’absurde en sup-
posant A non nilpotente. Alors A possède au moins des valeurs propres non nulles que l’on
note λ1 , . . . , λr , 1 ≤ r ≤ n de multiplicités respectives n1 , . . . , nr . Par hypothèse nous avons
pour tout k ∈ N? :
tr(Ak ) = n1 λk1 + · · · + nr λkr .
Écrire ces relations pour k variant de 1 à r équivaut à dire que le vecteur (n1 , . . . , nr ) est
solution du système linéaire
λ 1 λ 2 . . . λr
x1
λ21 λ22 . . . λ2r x2
. = 0.
.. . . . . . .
λr1 λr2 . . . λrr xr
Et ce système est de Cramer puisque le déterminant de la matrice du système vaut3
Y
λ1 . . . λr (λi − λj )
1≤i<j≤r
Ë Soient λ1 , . . . , λn les valeurs propres de A, comme rang(Ak ) = λk1 + · · · + λkn pour tout
entier k ∈ {1, . . . , n}, l’implication (1) ⇒ (2) est immédiate.
Pour (2) ⇒ (1), remarquons que spec(A − In ) = { λ − 1, λ ∈ spec(A)}, il est donc suffisant
de montrer que la seule valeur propre de A − In est 0 ou encore que A − In est nilpotente.
D’aprés la première question, A − In est nilpotente, si et seulement si, tr(A − In )k = 0 pour
tout entier k ∈ {1, . . . , n}, et cette dernière vérification est élémentaire car :
k k
k
X k l k−l
X k
tr(A − In ) = (−1) tr(A ) = n (−1)l = (1 − 1)k = 0.
l=0
l l=0
l
o
Mais les matrices B k sont à coefficients entiers : la seule alternative est donc qu’il existe
un entier k0 tel que B k0 = 0. La matrice B est donc diagonalisable et nilpotente : c’est la
matrice nulle et finalement A = In o
Ê Nous avons
1 ω . . . ω4
ω ω2 . . . ω5
2
ω ω3 . . . ω6 ,
A=
.
.. .. ..
. .
ω4 ω5 . . . ω8
les colonnes de A sont proportionnelles : A est donc de rang 1 et donc semblable à un matrice
de la forme
0 ... 0 ?
.. .. ..
. . .
B = . .
.. ... ?
0 . . . 0 tr(A)
Mais tr(A) = 1 + ω 2 + ω 4 + ω 6 + ω 8 = 0 implique que B 2 = 0 puis A2 = 0 : la matrice A
n’est donc pas diagonalisable.
Ë A2 = 0 implique An = 0 pour tout entier n ≥ 2, soit exp(A) = I5 + A. o
Il faut se souvenir qu’une matrice M à coefficients entiers est inversible avec un inverse à
coefficients entiers si et seulement si det(M ) = ±1 (si M admet un tel inverse N alors,
det(M ) det(N ) = det(M ) det(N ) = 1 soit det(M ) = ±1 ; réciproquement si det(M ) = ±1
alors ±M f (où Mf est la transposée des cofacteurs de M ) est l’inverse de M ).
Considérons alors le polynôme de degré au plus 2, f (x) = det(A + xB). Vu les hypothèses
et la remarque préliminaire f (x) = ±1 pour x = 0, 1, 2, 3 et 4 ; par le principe des tiroirs f
prends une des valeurs ±1 au moins trois fois ce qui le force à être constant, en particulier
det(A + 5B) = ±1, CQFD o
On va montrer que S est le carré d’une matrice antisymétrique réelle si et seulement si,
toutes ses valeurs propres sont négatives et ses valeurs propres strictement négatives sont de
multiplicité paire.
ê (condition suffisante). Si S est la matrice nulle, alors S = 02 et on peut supposer dé-
sormais S 6= 0. S est symétrique réelle, ses valeurs propres sont réelles et peuvent, vu les
hypothèses, s’écrire sous la forme
−a21 , −a21 , −a22 , −a22 , . . . , −a2l , −a2l , 0, . . . , 0
où a1 , a2 , . . . al ∈ R?+ (et ne sont pas forcément distincts). Toujours parce que S est symétrique
réelle, elle est diagonalisable dans une base orthonormée :
∃ O ∈ On (R) : O−1 SO = diag(−a21 , −a21 , −a22 , −a22 , . . . , −a2l , −a2l , 0, . . . , 0).
Notons pour k ∈ {1, . . . , l}
0 −ak
Ak =
ak 0
et considérons la matrice diagonale par blocs
R = diag(A1 , A2 , . . . , Al , 0, . . . , 0) ∈ Mn (R).
R est bien antisymétrique réelle et un calcul par blocs montre que
R2 = diag(−a21 , −a21 , −a22 , −a22 , . . . , −a2l , −a2l , 0, . . . , 0) = O−1 SO = tOSO,
OR tO est une matrice antisymétrique réelle vérifiant (OR tO)2 = S, CQFD.
ê Pour la condition nécessaire, si S = R2 avec R antisymétrique, comme R = − tR on a
S = R2 = − tRR qui implique
∀ X ∈ Rn , t
XSX = − t X tRRX = −kRXk2
qui prouve que S est symétrique réelle négative : son spectre est donc inclu dans R− .
39
40/408 Petit Bestiaire d’Exercices pour l’Oral de l’Agrégation Interne Patrice Lassère
Les valeurs propres de S sont les carrés des valeurs propres de R qui sont donc imaginaire
√
pures et stables par conjugaison puisque R est à coefficients réels. (λ ∈ spec(S) =⇒ ±i λ ∈
spec(R)) : elles sont donc de multiplicité paire. CQFD. o
hA0 , U i2 ≤ kA0 k2 kU k2
soit
!2 ! !
X X X
(F) |ai,j | ≤ a2i,j 12
1≤i,j≤n 1≤i,j≤n 1≤i,j≤n
mais
X
a2i,j = trace( tAA) = trace(A)
1≤i,j≤n
et puisque A est une matrice de projection trace(A) = rang(A) ≤ n soit avec (F)
!2
X
|ai,j | ≤ n2 rang(A) ≤ n3 .
1≤i,j≤n
d’où le résultat. q
t 1
(A − B) = B − A ⇐⇒ B =t B = (A +t A)
2
si bien que
X 2
X (aij − aji )2
inf (aij − mij ) = .
M =((mij ))∈Sn
1≤i,j≤n 1≤i,j≤n
2
Non bien sûr ! λ = 1 est son unique valeur propre : si elle était diagonalisable elle serait
semblable et donc égale à I2 . q
L’application
p
ϕ : (x, y) ∈ E × E 7→ ϕ(x, y) = hx, yi − hx, xihy, yi ∈ R.
est continue sur E × E muni de sa topologie naturelle d’espace produit (la continuité du
produit scalaire résulte de l’inégalité de Cauchy-Schwarz) et, vu le cas d’égalité dans l’inéga-
lité de Cauchy-Schwarz O = ϕ−1 (R? ). C’est donc un ouvert comme image réciproque d’un
ouvert par une application continue. q
Muni du produit scalaire hA, Bi = tr(A tB), Mn (R) est un espace euclidien et
E := { A ∈ Mn (R) : tr(A) = 0}
est un sous-espace vectoriel de Mn (R) de dimension n2 − 1 : c’est en effet le noyau de la
forme linéaire
X d X
X d d
X
2 2
hu(ei ), fj i = hu(ei ), fj i = ku(ei )k2 ,
1≤i,j≤d i=1 j=1 i=1
nous sommes donc déja assurés que (4) ne dépend pas (fj )d1 . Il reste à prouver que
d
X d
X
2
ku(ei )k = ku(fi )k2
i=1 i=1
X d
X
trace(At A) = a2i,j = ku(ei )k2
1≤i,j≤d i=1
de même
d
X
ku(fi )k2 = trace(Bt B)
i=1
où B = mat(u, (fj )d1 ) = P−1 AP et P ∈ GLd (R) est la matrice de passage entre les deux bases.
Ces deux bases étant orthonormées, P est orthogonale i.e. P −1 = t P et par suite
trace(Bt B) = trace(P−1 APt (P−1 AP) = trace(P−1 APt Pt At P) = trace(P−1 At AP) = trace(At A).
Exercice 44 (Toute matrice carrée réelle est produit de deux matrices symé-
triques réelles ) [10]
Ê Montrer que pour toute matrice carrée réelle, il existe une matrice de passage à
sa transposée qui soit symétrique.
Ë En déduire que toute matrice carrée réelle est le produit de deux matrices sy-
métriques réelles.
Ê La solution repose sur le fait suivant : « toute matrice A ∈ Mn (R) est semblable à
sa transposée mais si de plus A est une matrice cyclique (i.e. semblable a une matrice
compagnon), on peut imposer à la matrice de passage d’être symétrique réelle. » que l’on va
pouvoir étendre à tout Mn (R).
En effet, soient
0 0 ... 0 −a0
a1 a2 ... an−1 1
1 0 ... 0 −a1 a2 a3 ... 1 0
0 1 ... 0 −a2 . . . .
A = , S = ∈ GLn (C),
. . . .
. .. .. . . . . ... . .
. . . . .
. . . an−1 1 ... 0 0
0 ... 0 1 −an−1 1 0 ... 0 0
On procède par récurrence sur la taille n de la matrice, la propriété est clairement vraie
pour n = 1.
(F) B (ou A, c’est pareil par symétrie) admet une valeur propre λ non nulle telle que
ker(A) ∩ ker(In − λB) 6= {0}.
Admettons pour le moment (F) et prouvons l’assertion au rang n. Soit λ une valeur propre
non nulle de B telle que ker(A) ∩ ker(I − λB) 6= {0} et soit eλ ∈ ker(A) ∩ ker(I − λB) \ {0}.
On considère alors une base orthogonale B de Rn de premier eλ
terme . Les matrices
de
0 0 λ 0
A et B dans cette base sont respectivement de la forme et où les
0 A0 0 B0
2 Il existe O ∈ O (R) telle que S = O−1 DO = tODO avec D = diag(λ , . . . , λ ), (λ > 0, ∀ i) ; si on pose √D =
n √ 1 n i
√ √
diag( λ1 , . . . , λn ) alors P = DO convient.
Il reste donc à établir la propriété (F), c’est la partie délicate qui se déduit des deux lemmes
suivants :
Preuve du lemme 1 : Considérons une telle matrice et supposons par l’absurde qu’il
existe un coefficient aji 6= 0. Soit Xt = (xk )n1 le vecteur colonne où xj = 1, xi = taji , t ∈ R
et où les autres composantes sont nulles, alors tXt U Xt = ajj + ta2ji change de signe lorsque t
décrit R ce qui est absurde.
Quitte à modifier les n − r vecteurs de base er+1 , . . . , en on peut supposer U3 diagonale, plus
précisément soit Q ∈ On−r telle que
t
QU3 Q = diag(ur+1,r+1 , . . . , un,n )
Ir 0
Dans la nouvelle base correspondant à la matrice orthogonale notre matrice
0 Q
U20
t t U1 − tDr
P U P − tP V P s’écrit t 0
U2 diag(ur+1,r+1 , . . . , un,n )
Il en résute que det(U3 ) = (−1)r ur+1,r+1 , . . . , un,n = 0 qui assure que la matrice U
symétrique positive admet un élément diagonal disons uii , (i ∈ {r + 1, . . . , n}) nul ; on peut
donc appliquer le lemme 1 et affirmer que la ième colonne correspondante dans U est nulle.
Ainsi P U tP ei = 0, mais on a aussi
t Dr 0
P V P ei = Dei = ei = 0
0 0
car i ∈ {r + 1, . . . , n} ou encore U tP ei = V tP ei = 0, soit finalement
t
P ei ∈ ker(U ) ∩ ker(V ).
CQFD
Preuve de (F) : Soit λ une valeur propre non nulle de B, l’hypothèse (Hn ) implique que
det(In − λ−1 B − xA) = 0, ∀x ∈ R
on est donc dans le cadre du lemme 2 avec V = A et U = In − λ−1 B qui sera positive si on
choisit pour λ la plus grande des valeurs propres non nulle. o
i C’était un exercice « estival » aimablement proposé par notre collégue J.B.Hiriart-
Urruty.
POLYNÔMES
Exercice 46 (Sur les polynômes de la forme P = QP 00 avec deg(Q) = 2 )
Putnam 1999
Soit P ∈ R[X] un polynôme de degré d. On suppose que P = QP 00 où Q est de
degré 2. Montrer que si P admet au moins deux racines distinctes alors, il doit
avoir d racines distinctes.
ê Solution 1 : Supposons que P n’admet pas d racines distinctes, il admet donc une
racine au moins double que l’on peut supposer, sans perdre de généralité être égale à 0 (quitte
à remplacer x par x−a). Désignons par n la multiplicité de ce zéro, on a donc P (x) = xn R(x)
avec R(0) 6= 0. Soit
P 00 (x) = (n2 − n)xn−2 R(x) + 2nxn−1 R0 (x) + xn R00 (x).
Comme R(0) 6= 0 et n ≥ 2, x = 0 est une racine d ’ordre au plus n − 2 de P 00 et par
conséquent (car c’est aussi une racine d’ordre n de P ) x2 divise Q. Mais Q est de degré 2
et donc de la forme Q(x) = Cx2 où C = d−1 (d − 1)−1 (en comparant les termes de degré d
dans P = Cx2 P 00 ...).
Pour conclure, l’égalité P (x) = ad xd + · · · + a1 x + a0 = d−1 (d − 1)−1 x2 P 00 (x) implique
aj = d−1 (d − 1)−1 j(j − 1)aj , 0 ≤ j ≤ d, soit aj = 0 pour 0 ≤ j ≤ d − 1 et finalement
P (x) = ad xd .
o
p(j)
X
d − n(z) = αi,j .
i=1
Or les ensembles {x1,j , . . . , xp(j),j } constituent, quand j parcourt {1, . . . , r}, une partition de
C. La somme des d − n(z), z ∈ V , est donc égale à la somme des racines de P 0 comptées
avec leurs multiplicités, c’est-à-dire au degré de P 0 : la somme cherchée est d − 1.
Ì On suppose bien évidemment P non nul et non constant. On raisonne par récurrence
sur le degré m de P en X, le cas de m = 0 étant trivial. Supposons donc m > 1, et
R, ai,j (λn −λi+j ) = 0 donc, comme R est infini, ai,j = 0 dès que i+j 6= n.
∀λ ∈ R, λn (X 2 + Y 2 )Q(λX, λY ) = λ2 (X 2 + Y 2 )Q(λX, λY )
puis, du fait que R[X, Y ] est intègre, ∀λ ∈ R, λn−2 Q(X, Y ) = Q(λX, λY ) i.e. Q ∈ En−2 .
k (−1)k 2k+1
pour 0 6 2k 6 n, a2k = (−1) Cn2k a......slant2k
+ 1 6 n, a2k+1 = Cn a1 .
n
Pn
Ainsi l’application qui à P = i=0 ai X i Y n−i ∈ Ker ∆ associe (a0 , a1 ) ∈ R2 est un isomor-
phisme donc Ker ∆ est de dimension 2. De plus tout P ∈ Ker ∆ s’écrit P = a0 R + a1 I
avec
X
R= (−1)k Cn2k X 2k Y n−2......leqslant2k+16n (−1)k Cn2k+1 X 2k+1 Y n−2k−1 .
062k6n
Ê Pour m = 0, Rm,n est un espace vectoriel (pour les opérations usuelles sur les fonc-
xn xn
tions) mais pour m > 1, ce n’est pas le cas : par exemple (x+1) / Rm,n .
m + (x+2)m ∈
Notons
A = a0 + a1 X + · · · + an X n , B = b0 + b1 X + · · · + bm X m .
Puisque AB 6∈ 4Z[X] nécessairement l’un des deux polynômes, disons B, possède au moins
un coefficient impair.
Supposons alors que tous les coefficients de A ne soient pas pairs, soit as (resp. bk ) le premier
coefficient impair de A (resp. B) alors le coefficient de X s+k dans AB est
ab + a1 bk+s−1 + · · · + as−1 bk+1 + as b k + a b + · · · + ak+s b0
| 0 k+s {z } |{z} | s+1 k−1 {z }
pair · (?) + pair · (?) + · · · + pair · (?) + impair · impair + (?) · pair + (?) · pair + (?) · pair
| {z } | {z } | {z }
pair impair pair
2n1
(F) Qε (z) = z Qε .
z
La démonstration s’enchaîne alors de la manière suivante :
ê Étape 1 : Pour tout ε > 0, le polynôme Qε est sans racines sur le cercle unité.
C’est une conséquence immédiate de la formule (8) qui implique |Qε (eix )| ≥ ε.
−1
ê Étape 2 : Soit ε > 0. Si ζ ∈ C est une racine de Qε , alors ζ est aussi racine de Qε
avec la même multiplicité.
Remarquons déja que, Qε (0) ≥ ε > 0, donc ζ 6= 0. On a alors Qε (z) = (z − ζ)d R(z) où
R ∈ C2n−d [X] vérifie R(ζ) 6= 0. En exploitant (F), on peut aussi écrire Qε (z) = (1−ζz)d S(z)
−1
où le polynôme S(z) = z 2n−d R(z −1 ) vérifie S(ζ ) 6= 0. C.Q.F.D.
ê Étape 3 : Les éventuelles racines de Q0 sur le cercle unité sont de multiplicités paires.
Supposons au contraire qu’une telle racine eiθ soit de multiplicité 2m − 1, m ∈ N? . La
suite de polynômes (Qε )ε>0 étant simplement convergente sur R vers Q0 , les racines d’un
polynôme dépendant continuement de ses coefficients et les polynômes Qε étant sans racines
sur le cercle unité, il existe r > 0, εr > 0 tels que pour tout 0 < ε < εr le polynôme
Qε possède exactement 2m − 1 racines (comptées avec leurs multiplicités) dans le disque
D(eiθ , r). Vu l’étape précédente, et quitte à réduire εr , nous sommes alors assurés que pour
toute racine ζ de Qε
−1
ζ ∈ D(eiθ , r) ∩ D(0, 1) ⇐⇒ ζ ∈ D(eiθ , r) ∩ C \ D(0, 1)
ces zéros ont donc même multiplicité et le nombre de racine de Qε dans D(eiθ , r) (0 < ε < εr )
est forcément pair, d’où la contradiction et les racines éventuelles de Q0 sur le cercle unité
sont de multiplicité paire.
ê Étape 4 : la conclusion ζ1ε , . . . , ζnε désignant les racines de Qε dans le disque unité, nous
avons donc
n
(−1)n cn Y
Qε (z) = ε (z − ζjε )(1 − ζjε z)
ζ1 . . . ζnε j=1
où la somme ne porte que sur les racines ζ1 , . . . , ζn de module strictement plus petit que 1
(conséquence des deux premières étapes). Il ne reste plus qu’à faire tendre ε vers vers zéro,
et, toujours en invoquant la continuité « coefficients-racines » d’un polynôme :
n
(−1)n cn Y
Q0 (z) = (z − ζj )(1 − ζj z)
ζ1 . . . ζn j=1
( si par exemple |ζ| < 1 est racine de Q0 , il existera une suite (ζε )ε de racines de Qε de
−1
module < 1 et de limite ζ ; vu l’étape 2, la suite de racines (ζε )ε se doit de converger vers
g(x) = P (eix )2
et le résultat est démontré. q
i Remarque : Ce résultat est dû à L. Fejèr et F. Riesz (voir F. Riesz & B.S. Nagy
« Functional Analysis », Dover, pages 117-118 ou bien Q.I. Rahman & G. Schmeisser
« Analytic Theory of Polynomials », Oxford Publications (2002) page 410). On peut aussi
remarquer que pour notre choix de P , toutes ses racines sont dans le disque unité fermé.
ê Soit α une racine commune aux deux polynômes P, Q : il existe alors deux polynômes
non nuls P0 , Q0 tels que
ê Réciproquement, supposons qu’il existe deux polynômes non nul U, V tels que
soit (8).
Ì Pour démontrer le théorème de Gauss-Lucas, on raisonne par l’absurde : soit z un racine
a n
de P 0 n’appartenant pas à H, on a alors P 0 (z) = 0 et > 0 ce qui est absurde vu (8)
P (z)
donc z ∈ H. o
Considérons l’application
|P (1) − P 0 (1) + P (−1) + P 0 (−1)|
f : (a0 , a1 , . . . , a2004 ) ∈ C2005 \{0C2005 } 7→ f (a0 , a1 , . . . , a2004 ) = R1 ,
−1
|P (t)|dt
où P (x) = a0 + a1 x + · · · + a2004 x2004 ∈ C2004 [X].
ê Il n’est pas difficile de vérifier que f est homogène de degré 0, i.e.
f (ta0 , ta1 , . . . , ta2004 ) = f (a0 , a1 , . . . , a2004 ), ∀ t ∈ R? .
ê Le second point est que f est continue sur la sphère unité de CR2004 (c’est une frac-
1
tion rationnelle en les variables a0 , a1 , . . . , a2004 ) dont le dénominateur −1 |P (t)|dt s’annule
seulement si P = 0 i.e. a0 = a1 = · · · = a2004 = 0).
Par compacité de la sphère unité, l’application continue f y est bornée (disons par C > 0)
et par homogénéité f ≤ C sur C2004 \ {02004 }. L’inégalité pour P = 0 étant une égalité, la
démontration est terminée.
Avec P (x) = x2004 on vérifie sans peine que C ≥ 4.106 . q
Ë La double inégalité résulte de l’inégalité classique sin(t) ≤ t ≤ tan(t) valable sur ]0, π/2[
(faire une étude de fonction ou invoquer un argument de convexité). Si on écrit cette inégalité
pour t = kπ/2n + 1, en les sommant pour 1 ≤ k ≤ n il vient avec (4)
n
n(2n − 1) X (2n + 1)2 n(2n − 1)
≤ 2 2
≤n+ .
3 k=1
π k 3
Il ne reste plus qu’à diviser par (2n + 1)2 /π 2 et faire tendre n vers +∞ pour retrouver la
valeur bien connue
X 1 π2
ζ(2) = = .
n≥1
k2 6
o
ê Comme P1 (x) = x2 −1 ≥ −1 sur R, nous avons pour tout x ∈ R : Pn+1 (x) = P1 (Pn (x)) ≥
−1 et l’équation Pn (x) = a n’admet pas de solution pour tout a < −1.
ê Montrons que l’équation Pn (x) = a admet exactement deux racines réelles distinctes
pour tout a > 0. Pour cela, procédons par récurrence sur n ≥ 1 : pour n = 1 c’est clair.
Supposons √ √ n, Pn+1 (x) = a équivaut√à P1 (Pn (x)) = a qui implique
l’assertion vraie au rang
Pn (x) = − a + 1 ou Pn (x) = a + 1 ; l’équation Pn (x) = a + 1 > 1 admet √ exactement
√ hypothèse de récurrence ; l’équation Pn (x) = − a + 1 est sans
deux solutions distinctes par
solutions réelles puisque − a + 1 < −1. Ainsi l’équation Pn (x) = a admet exactement deux
racines réelles distinctes pour tout a > 0.
ê Nous allons maintenant montrer que l’équation Pn (x) = 0 admet exactement n + 1
√ procède à nouveau par récurrence sur n. Si n = 1 les solutions
solutions réelles distinctes. On
sont ±1 et si n = 2 : 0 et ± 2. Supposons l’assertion vraie au rang n ≥ 3. Comme on peut
écrire Pn+2 (x) = P2 (Pn (x)) = Pn2 (x)(Pn2 (x)−2), l’ensemble des solutions réelles de l’équation
P
√n+2 (x) = 0 est exactement
√ la réunion des racines réelles de équations Pn (x) = 0, Pn (x) =
2 et Pn (x) = − 2. Vu l’hypothèse de récurrence l’équation Pn (x) = 0 admet n + 1 √ racines
réelles distinctes
√ et les deux premières étapes nous assurent que les équations P n (x) = 2>0
et Pn (x) = − 2 < −1 admettent respectivement 2 et 0 racines réelles ; ces trois ensembles
étant deux à deux disjoints, l’équation Pn+2 (x) = 0 admet n + 1 + 2 = n + 3 solutions et
l’assertion est bien établie.
En particulier l’équation P2005 (x) = 0 admet 2006 racines réelles distinctes. o
Ê Supposons, sans perdre de généralité, que le point A2 est l’origine. Le polynôme peut
s’écrire alors sous la forme
P (x) = (a0 x4 + a1 x3 + a2 x2 + a3 x + a4 )x2 + a5 x.
L’équation y = a5 x est donc celle de la tangente à la courbe à l’origine et détermine le
segment A1 A3 . Vu l’hypothèse, les abcisses des points A1 et A3 sont respectivement a et −a
avec a > 0 : les réels a et −a sont racines doubles du polynôme a0 x4 + a1 x3 + a2 x2 + a3 x + a4
soit a0 x4 + a1 x3 + a2 x2 + a3 x + a4 = a0 (x2 − a2 )2 et
P (x) = a0 (x2 − a2 )2 x2 + a5 x.
L’égalité des aires est alors conséquence de l’égalité (par parité de x 7→ a0 (x2 − a2 )2 x2 ) des
intégrales
Z 0 Z a
2 2 2 2
a0 (x − a ) x dx = a0 (x2 − a2 )2 x2 dx.
−a 0
Soit P un tel polynôme de degré d ≥ 1, soient a1 < · · · < ar les racines distinctes de P de
multiplicité respective α1 , . . . , αr , ai est donc racine d’ordre αi − 1 de P 0 ; par Rolle P 0 admet
une nouvelle racine bi sur chaque intervalle ]ai , ai+1 [ (1 ≤ i ≤ r − 1) si bien que nous avons
déja ri=1 (αi − 1) + r − 1 racines pour P 0 qui est de degré d − 1. Soit
P
X r X r
d−1≥ (αi − 1) + r − 1 = αi − r + r − 1 = d − 1
i=1 i=1
0
d’où l’égalité et P ne peut donc avoir d’autres racines ; les bi sont donc racines simples et
les éventuelles racines multiples de P 0 sont parmi celles de P . o
GÉOMÉTRIE
Exercice 62 (Optimisation dans un triangle )
On désigne par h1 , h2 , h3 les hauteurs d’un triangle et par ρ le rayon de son cercle
inscrit. Déterminer le minimum de
h1 + h2 + h3
ρ
lorsque l’on décrit tous les triangles non dégénérés du plan.
Sans perdre de généralité (quitte à faire une translation), on peut prendre comme cercle celui
d’équation (C ) : x2 + (y − 1)2 = 1 et comme parabole, celle d’équation (Pk ) : y = kx2 (si
la parabole n’est pas tangente au cercle, on imagine bien qu’en « l’enfoncant » un peu plus,
la longueur de l’arc inscrit ne peut qu’augmenter). (Pk ) est alors tangente à (C ) en (0, 0)
et pour k > 12 , l’intersecte en les deux points
√
2k − 1 2k − 1
± , .
k k
La longueur d’arc inscrite dans le disque est donc
Z √2k−1 √ Z 2√2k−1 √
k 1
L(k) = 2 1 + 4k 2 t2 dt = 1 + u2 du,
0 k 0
(après le changement de variable u = 2kt). Il s’agit donc d’étudier le maximum de
Z √
1 1 2 2k−1 √
(8) L : k ∈ [ , +∞[ 7→ L(k) = 1 + u2 du.
2 k 0
4 trouver une ref.
Patrice Lassère Petit Bestiaire d’Exercices pour l’Oral de l’Agrégation Interne 67/408
i Remarques : ê La fonction continue L nulle en 1/2 tend vers tout de même vers 4
en +∞ car
1 2
L(k) = I(k) + 4 −
k k
et √
Z √
1 1 2 2k−1 du 2 2k − 1
0 ≤ I(k) = √ ≤ −→ 0.
k k 0 1 + u2 + u k k→∞
ê Vu les variations de L, notre fonction est bornée sur [1/2, +∞[ et atteint son maximum
pour une valeur 1/2 < m < +∞. En utilisant un logiciel de calcul, on peut donner une
valeur approchée de m.........autour de 4, 001...semble-t-il.
On peut s’étonner que pour résoudre cet exercice on n’étudie pas la fonction L. En effet
il n’est pas trés difficile de trouver une primitive :
Z √ √
1 1
1 + u2 du = √ + log t + 1 + t2
2t 2t − 1 2
mais son apparence peu sympathique nous enlève les dernières envies de calculer la dérivée
de L pour étudier ses variations....
AP BP CP
Ê Posons a = PD
,b = PE
,c = PF
P est
, alors le barycentre de
(A, 1)
et (D, a)
(B, 1) et (E, b)
(C, 1) et (F, c)
ou bien, x, y, z désignant les coordonnées barycentriques de P dans le repère (A, B, C) :
P
est le barycentre de (A, x) et (D, y + z)
P est le barycentre de (B, y) et (E, z + x)
P est le barycentre de (C, z) et (F, x + y)
avec
y+z x+z x+y
a= ,b = ,c =
x y z
si bien que
x y x y z z
a+b+c= + + + + +
y x z z x y
et de l’inégalité classique u+ u1 ≥ 2, (∀ u > 0), résulte la première inégalité à établir à savoir :
a + b + c ≥ 6.
√
Ë Pour la seconde, on utilise l’inégalité x + y ≥ 2 xy, (∀ x, y ≥ 0) :
y+zx+zx+y
abc =
x y z
√
yz × xz × xy
≥8 = 8.
xyz
√
(F) xy + xz + zy ≥ S 3
en élevant au carré et en utilisant la formule de Héron
S 2 = (x + y + z)xyz
on est amené à vérifier
(F) ⇐⇒ x2 y 2 + x2 z 2 + y 2 z 2 ≥ x2 yz + xy 2 z + xyz 2
pour vérifier cette dernière inégalité il suffit d’appliquer celle de Cauchy-Schwarz aux vecteurs
u = (xy, yz, zx) et v = (yz, zx, xy), en effet
a cos(θ)
Mθ = ∈ E ∩ C ⇐⇒ a2 cos2 (θ) + b2 sin2 (θ) − 2αa cos(θ) − 2βb sin(θ) + γ = 0
b sin(θ)
2
a − b2 a2 b 2
iθ 4iθ 3iθ 2iθ
⇐⇒ Q(e ) := e + e (ibβ − aα) + e γ+ +
4 2 2
a2 − b 2
− eiθ (aα + bβ) +
4 = 0,
B0
C0
C
B A0
B0
I
C0
J
K
C
B A0
Je dis que :
A
(F) AACA0 =
3
En effet (cf. figure ci-dessus) ABC et ACA0 ont la même hauteur h = AH et la base de
ACA0 vaut 1/3 de la base de ABC. Or on sait que l’aire d’un triangle est donnée par :
base × hauteur
aire = d’où (F).
2
A A
De même, on a : ABAB 0 = ACBC 0 = .
3 3
On essaie maintenant d’écrire l’aire de ABC comme la somme des aires de sous-triangles, le
petit triangle IJK, les 3 triangles qu’on vient d’examiner, puis on corrige en soustrayant ce
qui a été compté 2 fois :
Ceci s’écrit :
A A A
A =x+ + + − (AAB 0 I + ACA0 K + ABC 0 J )
3 3 3
B0
E0
I
T
C0 C 00
J
h0 K
C
B E A0
On trace la parallèle à (BC) passant par B 0 : (E 0 B 0 ) k (BC) avec E 0 ∈ (AB).
On trace aussi la parallèle à (AB) passant par B 0 : (B 0 E) k (AB) et E ∈ (BC).
On définit le point C 00 comme l’intersection de (B 0 E) avec la parallèle à (BC) passant par
C 0 . Alors :
T est le milieu de [C 0 C 00 ]
En effet :
1◦ ) dans E 0 BB 0 , C 0 milieu de [E 0 B] (c’est Thalès) et (E 0 B 0 ) k (C 0 T ) entraîne par la
réciproque du théorème des milieux que T milieu de [BB 0 ].
2◦ ) dans BB 0 E, T milieu de [BB 0 ] et (T C 00 ) k (BE) entraîne par la réciproque du
théorème des milieux que C 00 milieu de [B 0 E].
3◦ ) E 0 B = B 0 E car E 0 BEB 0 parallélogramme, donc C 0 B = B 0 C 00
4◦ ) pour [BB 0 ] et [C 0 C 00 ] qui s’intersectent en T , Thalès entraîne T B/T B 0 = 1 =
T C 0 /T C 00
.
Donc C 0 T = 21 BE. Or BE = 31 BC et finalement
1
C 0 T = BC
6
On a (C T ) k (BC), on peut donc appliquer Thalès à [C 0 C] et [BT ] qui s’intersectent en J :
0
C 0T C 0J 1 C 0J 1 1
= ⇐⇒ = =⇒ C 0 J = JC =⇒ C 0 J = CC 0
BC JC 6 JC 6 7
0 0 0
Les triangles BJC et BCC ont même hauteur (h sur la figure) et leurs bases dans un
rapport de 7 :
1 1
ABJC 0 = ABCC 0 =⇒ ABJC 0 = AABC
7 21
Le même raisonnement appliqué à chacun des deux autres triangles donne le même résultat.
q
Maintenant puisque
n
X X
2
(x1 + x2 + · · · + xn ) = x2i + 2 xi xj
i=1 1≤i<j≤n
on peut écrire
X n
X
2 2 2
2 (xi xj + yi yj + zi zj ) = (x1 + · · · + xn ) + (y1 + · · · + yn ) + (z1 + · · · + zn ) − (x2i + yi2 + zi2 )
1≤i<j≤n i=1
2 2 2
= (x1 + · · · + xn ) + (y1 + · · · + yn ) + (z1 + · · · + zn ) − n
:= X 2 + Y 2 + Z 2 − n
soit, en reportant dans (8)
X
Pi Pj2 = n(n − 1) − (X 2 + Y 2 + Z 2 − n) = n2 − X 2 − Y 2 − Z 2 ≤ n2
1≤i<j≤n
CQFD q
où l’on a effectué le changement t = π/2−s dans la seconde intégrale de la seconde ligne. Pour
conclure, il suffit de remarquer que l’intégrande dans la dernière intégrale est une fonction
croissante sur [0, π/4].
Ë Nous avons
Z 2π p Z π/2 q
L= x0 (t)2 + y 0 (t)2 dt =4 a2 sin2 (t) + b2 cos2 (t)dt
0 0
Z π/2 p
=4 a2 + (a2 − b2 ) cos2 (t)dt
0
Z π/2 p
= 4a 1 − e2 cos2 (t)dt
0
Nous avons pour |u| < 1
∞
√ X (2n − 3)!!
1−u=1− un .
n=1
(2n)!!
En posant u = e cos(t) ∈] − 1, 1[, on a par convergence normale de la série entière sur [0, π/2]
∞
!
π X (2n − 3)!! 2n π/2
Z
L = 4a − e cos2n (t)dt ,
2 n=1 (2n)!! 0
f continue sur le compact [0, a] atteint son maximum en un point c ∈ [0, a] et il faut
remarquer que f (c) > 0 (sinon D aurait une aire nulle mais un perimètre a > 0...). Notons
k l’aire (ou le périmètre) de D. D est visiblement inclu dans le rectangle [0, a] × [0, f (c)] :
son aire est donc inférieure ou égale à celle du rectangle af (c). D’un autre coté, k > 2f (c)
car 2f (c) est strictement plus petit que la distance de (0, 0) à (c, f (c)) plus la distance de
(c, f (c)) à (a, 0) distance strictement plus petite que le périmètre k de D. Nous avons donc
2f (c) < k ≤ af (c) ; en particulier, ceci impose a > 2.
Réciproquement, pour a > 2 le domaine D associé à l’application constante f (x) = 2a/(a−2)
est un rectangle d’aire 2a2 /(a − 2) et de périmètre
2a 2a2
2a + 2 = .
a−2 a−2
Donc, tout nombre réel a > 2 convient. o
par conséquent dans le terme de gauche, l’un des deux facteurs se doit d’être supérieur ou
égal à n.
ê Troisième solution : Par convexité sur R?+ de l’application f : x 7→ f (x) = x−1 nous
avons !
n n
1 X ai 1X ai
f ≤ f
n i=1 bi n i=1 bi
soit ! !
n n
X ai X bi
n2 ≤ ·
i=1
bi a
i=1 i
et on retrouve l’inégalité de la seconde solution.
COMBINATOIRE ET PROBABILITÉS
Exercice 73 (Combinatoire : les nombres de Bell ) [15]
Pour tout n ∈ N? , on désigne par Bn le nombre de partitions de l’ensemble [1, . . . , n]
avec par convention B0 = 1.
Xn
Ê Montrer que pour tout n ∈ N : Bn+1 = Cnk Bk .
k=0
Ë Montrer que le rayon de convergence R de la série génératrice exponentielle
f (z) = ∞ Bn n ∞
P
k=0 n! z de la suite (Bn )0 est strictement positif et calculer f (z) pour
|z| < R.
∞
1 X kn
Ì Montrer que Bn = .
e k=0 k!
Bn
≤ 1 et le rayon de convergence R de la série entière f (z) = ∞ Bn n
P
On a donc k=0 n! z est
n!
supérieur ou égal à 1.
On va utiliser la formule démontrée dans la première question pour calculer f (z). Pour
z ∈] − R, R[
∞ ∞
X Bn n X Bn+1 n+1
f (z) = z =1+ z ,
k=0
n! k=0
(n + 1)!
donc
∞
0
X Bn+1
f (z) = zn
k=0
n!
∞ n
!
X 1 X
= Cnk zn
k=0
n! k=0
∞ n
!
X X Bk 1
= zn
k=0 k=0
k! (n − k)!
Patrice Lassère Petit Bestiaire d’Exercices pour l’Oral de l’Agrégation Interne 81/408
P∞ Bn n
On reconnait alors dans le dernier terme le produit de Cauchy des séries entières k=0 n! z =
f (z) et ∞ zn z
P
k=0 n! = e de rayon de convergence strictement positif : on a donc
Les pôles de cette fraction rationnelle sont des racines de l’unité : elle est donc dévelop-
pable en série entière à l’origine (et le rayon de convergence vaut 1). On a donc
+∞
! +∞ ! +∞ ! +∞ !
X X X X
(1 − x)−1 (1 − x3 )−1 (1 − x9 )−1 (1 − x15 )−1 = xa x3b x9c x15d
a=0 b=0 c=0 d=0
∞
X
= uk x k ,
k=0
α + b + 3c + 5d = 15.
ê d = 3 implique α = b = c = 0 : un cas.
ê d = 2 implique c = 1 (alors trois possibilités : (α, b) ∈ {(0, 2), (2, 0), (1, 1)}) ou c = 0
(alors α + b = 5 soit 6 possibilités) : neuf cas.
ê d = 1 implique c = 3, 2, 1 ou 0 soit 2, 5, 8 et 11 possibilités : 26 cas.
ê d = 0 implique c = 5, 4, 3, 2, 1 ou 0, soit 1, 4, 7, 10, 13 et 16 possibilités : 51 cas.
L’équation (8) a donc 51 + 26 + 9 + 1 = 87 solutions : la réponse à la question est donc non.
o
induite sur les n − k éléments restants (soit Dn−k possibilités)7 soit card(Pk ) = Ckn Dn−k , puis
n
X n
X
n! = Cnk Dk = k
Cn−k Dk .
k=0 k=0
ê On a
p p
X
k p−k
X n!(n − k)!
(−1) Cnk Cn−k = (−1)k
k=0 k=0
k!(n − k)!(p − k)!(n − p)!
p
(
X
k p k p p 0 si p ≥ 1,
= (−1) Cn Cp = Cn (1 − 1) =
k=0
1 si p = 0.
ê Des deux formules précédentes
n
! n
X (−1)k X
n! = (−1)k Cnk (n − k)!
k=0
k! k=0
n n−k
!
X X
= (−1)k Cnk l
Cn−k Dl vu (8)
k=0 l=0
n n−l
!
X X
= (−1)k Cnk Cn−k
l
Dl = Dn vu (4)
l=0 k=0
soit
X Dk e−z
D(z) = zk = , ∀ z ∈ D(0, 1).
k≥0
k! 1−z
Il ne reste plus qu’à déterminer les coefficients du développement en série entière à l’origine
k
e−z
pour obtenir Dk . Après un produit de Cauchy des deux séries k (−1) z k et
P
de z 7→ 1−z k!
P k
k z on trouve bien
n
! n n
Dn X X (−1)k X (−1)k
= ak bn−k = ×1= .
n! k=0 k=0
k! k=0
k!
ê Nous avons
∞
1 X (−1)l Dk X (−1)k Dk
= = + := + Rk ,
e p=0
l! k! l≥k+1
k! k!
soit
k! 1 1
+ = Dk + + k!Rk .
e 2 2
P (−1)l
La série l l!
vérifiant le critère des séries alternées, la majoration de son reste
1
|Rk | ≤
(k + 1)!
assure pour pour k ≥ 1 :
1 1
|k!Rk | < ≤
k+1 2
qui nous donne finalement
k! 1
Dk = E + .
e 2
p p
La distance entre les deux racines de p est égale à |∆| = |b2 − 4c| ; elle est donc inférieure
à 1 si et seulement si −1 ≤ b2 −4c ≤ 1. Il faut donc que le point (b, c) se trouve dans la région
2 2
délimitée par le carré [0, 1] × [0, 1] et les deux paraboles y = x 4−1 y = x 4+1 . La probabilité
R1 2
cherchée est donc l’aire de ce domaine, soit 0 x 4+1 dx = 13 divisée par l’aire du carré (soit
1) : elle vaut donc 1/3. q
x
x 2x 2x
L’entier le plus proche de est zéro si 2x < y et (pour n ≥ 1) vaut 2n si 4n+1
y
< y < 4n−1
(on ignore bien entendu les éventuelles extrémités de ces intervalles qui sont de probabilités
nulles). La probabilité cherchée est donc l’aire grisée de la figure ci-contre, soit :
[ 2x 2x
2
(x, y) ∈ [0, 1] : <y<
n≥1
4n + 1 4n − 1
soit
1 1 1 1 1
p= + − + − + ...
4 3 5 7 9
Mais comme
π 1 1 1
= 1 − + − + ...
4 3 5 7
5 π
on trouve p = − q
4 4
À Montrer que ! !
k
[ X \
card Vi = (−1)1+card(I) card Vi
i=1 ∅6=I⊂{1,...,n} i∈I
2
X
card(I) n
=− (−1) E Q
i∈I pi
∅6=I⊂{1,...,n}
n
X n 2
=− µ(d)E
d=2
d
Et en déduire rn .
n
X µ(d) log(n)
Á Montrer que rn − =0 .
d=1
d2 n
X 1 X µ(d) X X µ(l)
 Montrer que 2 2
= 2
= 1.
n≥1
n d≥1
d i≥1 l divise i
i
à Conclure.
Q
soit I ⊂ {1, .
. . , n} une partie non vide, le nombre de multiples de i∈I pi dans {1, . . . , n}
est E Q n pi soit
i∈I
! 2
\ n
card Vi = E Q
i∈I i∈I pi
Q card(I)
il faut maintenant remarquer que puisque µ i∈I pi = (−1) et µ(l) = 0 pour tout
autre entier l ∈ {2, . . . , n} (ces derniers possèdent un facteur carré) :
2 n
X
1+card(I) n X n 2
(−1) E Q =− µ(d)E
i∈I pi d
∅6=I⊂{1,...,n} d=2
soit finalement
k
! n
[ X n 2
card Vi =− µ(d)E .
i=1 d=2
d
k
! n n
[ X n 2 X n 2
n2 − card Vi = n2 + µ(d)E = µ(d)E
i=1 d=2
d d=1
d
n
cas favorables 1 X n 2
rn = = 2 µ(d)E .
cas possibles n d=1 d
n 2 n 2 n 2
≥E > −1
d d d
qui implique
1 n 2 1 1 2
0≥ 2
E − 2 ≥ 2−
n d d n nd
de sorte que
n n n
X µ(d) X µ(d) n 2 X µ(d)
rn − = E −
d2 d=1 n2 d d2
d=1 d=1
n
X 1 n 2 1
= µ(d) 2 E − 2
d=1
n d d
n
X 2 1
≤ |µ(d)| −
d=1
nd n2
n
X 2 1
≤ −
d=1
nd n2
n
1 2X1 1 log n log n
≤ + = +O =O .
n n d=1 d n n n
Ì & Í La série d µ(d)/d2 étant convergente
P
∞
X µ(d)
(1) lim rn = ,
n→∞
d=1
d2
d’un autre coté, nous pouvons écrire
∞ ∞
! ∞ !
π 2 X µ(d) X 1 X µ(d)
=
6 d=1 d2 i=1
i2 d=1
d2
X µ(d) X 1 X X1
= = µ(l) := S(i)
2
i2 d2 i≥1
i2 i≥1
i2
(i,d)∈N l/i
où les deux avant dernières égalités sont justifiées par l’absolue convergence des deux pre-
mieres séries (on peut alors « sommer par paquets »). il reste donc à estimer S(i) pour i ≥ 1.
αN
S(1) = 1 et pour i ≥ 2 soit i = q1α1 . . . qN la décomposition de i en facteurs premiers ; un
diviseur l de i s’écrit donc sous la forme l = q1β1 . . . qN βN
avec 0 ≤ βj ≤ αj . Mais µ(l) 6= 0
s
signifie que βj = 0 ou 1 et dans ce cas µ(l) = (−1) où s = card{1 ≤ j ≤ N : βj = 1}.
Il existe donc une bijection entre l’ensemble des diviseurs l de i tels que µ(l) = (−1)s et
l’ensemble des N -uplets (β1 , . . . , βN ) ∈ {0, 1}N avec exactement s composantes égales à 1 et
ce dernier est bien entendu de cardinal CNs si bien que
X N
X X N
X
S(i) = µ(l) = µ(l) = CNs (−1)s = (1 − 1)N = 0.
l/i s=0 l/i&µ(l)=(−1)s s=0
en résumé (
1 si i = 1
S(i) =
0 si i ≥ 2.
soit ∞
π 2 X µ(d)
=1
6 d=1 d2
et enfin
∞
X µ(d) 6
lim rn = = .
n→∞
d=1
d2 π2
un+1 = un + nun−1 , n ≥ 1.
Pour la seconde partie, il suffit de remarquer que f est développable en série entière n vn xn
P
sur R, puis que les coefficients vn satisfont à la même relation de récurrence que les un pour
conclure facilement. q
Parmi tous les cas possibles soit [0, d] × [0, d], les couples (x, y) qui nous intéressent sont ceux
qui se trouvent dans les deux triangles rectangles de sommets respectifs (0, l), (0, d), (d − l, d)
et (l, 0), (d, 0), (d, d − l) qui se réunissent pour former un carré de coté d − l.
(l − d, d)
(d, l − d)
(0, d − l)
(l, 0) (d, 0) x
(d − l)2
La probabilité cherchée est donc p = . q
d2
| |888|8| |88
correspond au partage 0, 3, 1, 0, 2
n
Cette procédure résout visiblement notre problème et il y a clairement Cn+p−1 choix possibles
n
i.e. dn,p = Cn+p−1 .
Ë 0ù avec les séries entières : Vu le cours sur les séries entières, par convergence absolue
on a le produit de Cauchy
X X X X X
an x n bn x n . . . cn xn = dn xn avec dn = ak bk0 . . . ck00 xk
n n n n k+k0 +···+k00 =n
X
avec cn,p = ak1 ak2 . . . akp où les aki = 1, donc vu la formule ci-dessus :
k1 +k2 +···+kp =n
1 X
n n
X X
n n
X
n
X
p
= Cn+p−1 x = x x . . . x = cn,p xn
(1 − x) n≥0 n n n n
et il n’y a plus qu’à identifier les coefficients. q
i Remarque : le candidat à l’agrégation externe peut (et doit) se placer dans l’unique
cadre des séries formelles.
Soit G un groupe fini non commutatif, notons Z son centre et pour x ∈ G désignons par Gx
l’ensemble des éléments de G commutant avec x. Z et Gx sont deux sous-groupes de G ; Z
est lui même un sous groupe de Cx . Le théorème de Lagrange, nous assure de l’existence de
trois entiers m, kx , lx ∈ N vérifant
|G| = m|Z|, |G| = kx |Gx |, et |Gx | = lx |Z|.
soit
kx lx = m
.
ê Si x ∈ Z, alors Gx = G, kx = 1 et lx = m.
ê Sinon, Gx 6= G (car x 6∈ Z) donc kx > 1 et lx > 1.
Soit A = ((aij )) ∈ On (R) ∩ Mn (Z), A étant orthogonale, ses colonnes sont de norme 1
n
X
∀1≤j≤n : a2ij = 1
i=1
mais les coefficients aij ∈ Z, donc
i.e.
1
1 + t + t2 + · · · + t10 = (a1 + a2 t + · · · + a6 t5 )(b1 + b2 t + · · · + b6 t5 )
∀t ∈ R :
11
10 1
le coefficient de t étant 11 , nécessairement a6 > 0 et b6 > 0 et par suite a1 + a2 t + · · · + a6 t5
et b1 + b2 t + · · · + b6 t5 possèdent chacun une racine réelle comme polynômes de degré impair
mais ceci est absurde car 1 + t + t2 + · · · + t10 est sans racines réelles, contradiction.
i Remarque : voici une autre manière pour résoudre la première partie de ce problème, elle
est un peu plus simple mais (à mon goût), moins élégante. On conserve les mêmes notations
qu’au dessus.
On a
1
P (X1 + X2 = 2) = a1 b 1 =
11
1
P (X1 + X2 = 12) = a6 b6 =
11
1
P (X1 + X2 = 7) = a1 b 6 + a2 b 5 + a3 b 4 + a4 b 3 + a5 b 2 + a6 b 1 =
11
des deux premières égalités on tire
1 1
b1 = b6 =
11a1 11a6
x2 + y 2
l’inégalité classique ≥ 2 nous donne
xy
1 a1 a6 2 1
+ ≥ >
11 a6 a1 11 11
soit, sur le troisième terme
P (X1 + X2 = 7) = a1 b6 + a2 b5 + a3 b4 + a4 b3 + a5 b2 + a6 b1
1 a1 a6
= + + a2 b 5 + a3 b 4 + a4 b 3 + a5 b 2
11 a6 a1
1 1
> + a2 b 5 + a3 b 4 + a4 b 3 + a5 b 2 >
11 11
i.e.
1
P (X1 + X2 = 7) >
11
contradiction.
Soit l’identification naturelle P({1, 2, . . . , n}) ' {0, 1}n donnée par
(
0 si i 6∈ A,
A ∈ P({1, 2, . . . , n}) ! eA = (eA (i))ni=1 où eA (i) =
1 si i ∈ A.
Dans le Q-espace vectoriel Qn de dimension n, les n + 1 vecteurs eA1 , . . . , eAn+1 sont Q-
linéairement dépendants ce qui peut se traduire facilement par
n+1
X
∃ λ1 , . . . , λn ∈ Z non tous nuls, tels que λi eAi = 0.
k=1
et qui équivaut à
X X
λi eAi = λj eAj
i∈I j∈J
si désirée. o
(r)
Ê première solution : Désignons pour n = r(5) ∈ N par pn la probabilité qu’aprés n
tirages la somme des faces soit congrue à r modulo 5. Nous avons bien entendu
(0) (1) (2) (3) (4)
p0 = 1 et p0 = p0 = p0 = p0 = 0
et il est facile de vérifier que pour n ∈ N?
6
X pr−j
n−1
(8) p(r)
n = .
j=1
6
(r)
Ces formules nous permettent de calculer pn pour quelques petites valeurs de n pour conjec-
turer que
1 4 1 4
p(r)
n = + n
si n ≡ 0(5) et p(r) n = − sinon.
5 5.6 5 5.6n
Ces conjectures se démontrent alors facilement par récurrence avec (8).
|66 {z
. . . 6} XY1 . . . Yn−k−1
k fois
où X ∈ {1, 2, 3, 4, 5} et Y1 , . . . , Yn−k−1 sont fixés. Chaque élément de la partition est donc
constituée de 5 suites dont, modulo 5 la somme des chiffres est exactement 0, 1, 2, 3, 4 soit
exactement une dont la somme de chiffres est 0(5). Ainsi le nombre de suites dont la somme
des chiffres est divisible par 5 est card(S )/5 = (6n − 1)/5 si n est pas un multiple de 5 et
1 + card(S )/5 = 1 + (6n − 1)/5 sinon (dans ce cas (6, 6, . . . , 6) est aussi solution).
La probabilité cherchée est donc (cas favorables sur cas possibles)
1 4 1 4
+ si n ≡ 0(5) et − sinon.
5 5.6n 5 5.6n
Ì troisième solution : Désignons par pk la probabilité que la somme des faces soit égale à
k et considérons la série génératrice associée
n
z + z2 + z3 + z4 + z5 + z6
X
k
f (z) = pk x =
k≥1
6
où la seconde égalité peut être vérifiée facilement par récurrence sur n. Il s’agit donc de
calculer k≥1 p5k ; pour cela soit ε = e2iπ/5 la première racine cinquième de l’unité, nous
P
avons
X f (ε) + f (ε2 ) + f (ε3 ) + f (ε4 )
p5k = .
k≥1
6
jn
Il est clair que f (1) = 1 et pour j = 1, 2, 3, 4 f (εj ) = ε6n si bien que
4 si n ≡ 0(5),
n
f (ε) + f (ε2 ) + f (ε3 ) + f (ε4 ) = 6−1
sinon.
6n
et finalement
1 4
+
5 5.6n
si n ≡ 0(5),
X
p5k =
1 − −1
k≥1
sinon.
5 5.6n
o
où ∞ h i ∞ h i
X n X n
α2 = , α5 = .
i=1
2i i=1
5i
Mais pour n ≥ 2
∞ h i ∞ h i
X n X n
α5 = < α2 = .
i=1
5i i=1
2i
Par conséquent, pour que l’écriture décimale de n! se termine par exactement 2006 zéros il
est essentiel que α5 = 2006. Il faut donc déterminer les entiers n tels que n! = 2α2 3α3 52006 . . . .
Comme ∞ h i ∞ ∞
X n X n n X −i n
α5 = 2006 = < = 5 = ,
i=1
5i i=1
5i 5 i=0 4
nous avons n > 4 × 2006 = 8024. Avec un calcul facile l’exposant de 5 dans la décomposition
de 8025! vaut ∞
X 8025
α5 = i
= 1605 + 321 + 64 + 2 = 2004.
i=1
5
Il faut nous faut donc deux zéros supplémentaires, par conséquent, le plus petit entier n tel
que n! se termine par 2006 zéros est n = 8035 et l’ensemble cherché est 8035, 8036, 8037, 8038, 8039.
Exercice 89 (Dénombrement dans les groupes) [10]
Soient G un groupe fini, A, B deux parties de G telles que
o
card(A) + card(B) > card(G).
Montrer que AB = G (où AB = { ab, a ∈ A, b ∈ B}).
ANALYSE 1
CHAPITRE 3
TOPOLOGIE
Ê Dire que la famille (Ui )i∈N? n’est pas libre dans l2 (N) c’est dire qu’il existe N ∈
N? , λ1 , . . . , λN ∈ R non tous nuls, tels que
λ1 U1 + · · · + λN UN = 0l2 (N)
soit
λ1 (αk )k + λ2 (α2k )k + · · · + λN (αN k )k = (λ1 αk + λ2 α2k + · · · + λN αN k )k = 0l2 (N) ,
ou encore, si P (X) = λ1 X + λ2 X 2 + · · · + λN X N ∈ R[X]
P (αk ) = 0, ∀ k ∈ N? .
Comme α ∈] − 1, 1[ le polynôme P a trop de zéros pour ne pas être le polynôme nul donc
λ1 = λ2 = · · · = λN = 0
et la famille (Ui )i∈N? est bien libre dans l2 (N).
∞
!
X
⇐⇒ xk αkj = 0, ∀ i ∈ N? .
k=0
Ì F ⊥ = {0l2 (N) } implique que F = l2 (N) (c’est une conséquence immédiate du théorème
de projection orthogonale sur un convexe fermé dans un espace de Hilbert car F ⊥ = (F )⊥ )
autrement dit, la famille {Ui } est totale dans l2 (N). o
Exercice 91 (La somme de deux sous espaces fermés est-elle fermée ? ) [10]
Soient E un espace vectoriel normé (sur K = R ou C), F, G deux sous espaces de
E. On suppose F fermé et G de dimension finie, montrer que F +G := { x+y, x ∈
F, G ∈ G} est fermé.
Aprés une récurrence élémentaire, on peut se ramener au cas où G est une droite vectorielle
Kg de E. Si g ∈ F alors F + G = F qui est fermé et il n’y a rien à démontrer. Supposons
donc que g 6∈ F , soit x ∈ F + G, et montrons que x ∈ F + G.
Il existe une suite (xn )n dans F + G qui converge vers x dans E, les vecteurs xn sont donc
de la forme xn = fn + λn g où fn ∈ F et λn ∈ K. Remarquons que si la suite (|λn |)n ne tends
pas vers +∞ alors l’exercice est résolu : en effet on peut alors on peut alors extraire de la
suite (λn )n une suite bornée, puis, via Bolzano-Weierstrass dans K, une sous-suite (λϕ(n) )n
convergente vers λ ∈ K. Dans ce cas, la relation fϕ(n) = xϕ(n) − λϕ(n) g montre que (fϕ(n) )n
converge vers f = x−λg qui appartient à F puisque F est fermé par hypothèse et finalement
x = f + λg ∈ F + G.
Il reste à vérifier que (|λn |)n ne tends pas vers +∞. Si tel était le cas, les relations (valables
pour n assez grand)
xn fn
g= −
λn λn
impliqueraient que F 3 fn /λn → −g soit, g ∈ F = F ce qui est exclu puisque g 6∈ F . o
Ê ò Extraite de la base orthonormée (en )n≥0 la suite (xn )n≥0 est orthonormée. Pour la
suite (yn )n≥0 on a : kyn k2 = (1 − 4−n ) + (2−n )2 = 1 ; et pour n 6= m les vecteurs e2n , e2n+1
sont orthogonaux aux vecteurs e2m , e2m+1 car les ensembles {2n, 2n + 1} et {2m, 2m + 1}
sont disjoints : la suite (yn )n≥0 est aussi orthonormée. Ce sont des bases hilbertiennes des
espaces X et Y respectivement, car dans un esapce de Hilbert, toute suite orthonormée est
une base hilbertienne de l’espace vectoriel fermé qu’elle engendre.
P
ò Soit x = n≥0 ckP ek ∈ H. Si x P∈ X ∩ Y , de la question précédente nous avons aussi
les developpements x = n≥0 an xn = n≥0 bn yn . Calculons c2k+1 :
X X
c2k+1 = hx, e2k+1 i = h an xn , e2k+1 i = an hxn , e2k+1 i = 0,
n≥0 n≥0
i Il est utile de se souvenir qu’un hyperplan est le noyau d’une forme linéaire non identique-
ment nulle sur E ; il faut aussi ne pas oublier (voir exercice ? ? ?) qu’un hyperplan H = ker(ϕ)
est fermé si, et seulement si, la forme linéaire ϕ est continue. Rappelons aussi (voir l’exer-
cice ? ?) que dans un espace vectoriel normé de dimension infinie, il existe toujours des
formes linéaires discontinues.
ò Supposons H fermé et montrons que E\H n’est pas connexe. Soit ϕ une forme linéaire sur
E de noyau H, posons H + = ϕ−1 (R?+ ), H − = ϕ−1 (R?− ). Comme ϕ n’est pas identiquement
nulle H + et H − sont non vides, montrons que H + est ouvert1. Soit x ∈ H + et r > 0 tel que
B(x, r) ⊂ E \ H (une telle boule existe car H est fermé donc E \ H est ouvert) ; la boule
B(x, r) est convexe et ϕ est linéaire donc ϕ(B(x, r)) est une partie convexe de R ne contenant
pas l’origine mais rencontrant R?+ c’est donc un intervalle de R?+ . Ainsi ϕ(B(x, r)) ⊂ R?+ et
donc B(x, r) ⊂ H + qui est bien ouvert. De la même manière H − est fermé. On dispose ainsi
d’une partition de E \ H en deux ouverts non vides H + et H − : E \ H est pas connexe et à
fortiori n’est pas connexe par arcs.
Preuve du lemme : Il s’agit de montrer que pour tous c, d ∈ D il existe un chemin continu
γ : [0, 1] → D tel que γ(0) = c et γ(1) = d.
Supposons pour commencer que c ∈ C et d ∈ D ⊂ C, alors il existe une suite (xn )n
dans C de limite d et de premier terme x1 = c. Considérons alors le chemin continu γ :
[0, 1] → D défini comme suit : pour tout n ∈ N? : γ(1 − 1/n) = xn , la restriction de γ à
[1 − 1/n, 1 − 1/(n + 1)] est affine, et enfin γ(1) = d. Comme C est convexe et inclu dans D
il est clair que γ([0, 1[) ⊂ C ⊂ D et γ(1) = d implique γ([0, 1]) ⊂ D. La restriction de γ à
chaque segment de [0, 1[ est affine par morceaux : γ est donc continue sur [0, 1[. Il reste donc
à vérifier que γ est bien continue au point 1. Pour cela, soit ε > 0 et N ≥ 1 tel que n ≥ N
implique xn ∈ B(d, ε). Pour t ∈]1 − 1/N, 1[ il existe n ≥ N tel que t ∈ [1 − 1/n, 1 − 1/(n + 1)],
soit γ(t) ∈ [xn , xn+1 ], comme xn et xn+1 sont dans la boule convexe B(d, ε) il en est de même
de γ(t) i.e. kγ(t) − dk = kγ(t) − γ(1)k < ε. γ est bien continue au point 1.
Pour le cas général c, d ∈ D, et on choisit un point e dans C, vu ce qui précède il existe
dans D deux chemins continus sur [0, 1], l’un reliant e à c et l’autre e à d et c’est alors une
procédure classique pour en construire un reliant c à d. C.Q.F.D.
Montrons maintenant que si H n’est pas fermé, alors E \ H est connexe par arcs. H est
un sous-espace vectoriel de E contenant strictement H, donc H = E. Soit a ∈ E \ H,
l’application x 7→ x + a étant un homéomorphisme a + H = a + H = E. Ainsi a + H ⊂
E \ H ⊂ a + H. a + H étant convexe, le lemme précédent assure que E \ H est connexe par
arcs. o
1on peut bien sûr invoquer la remarque préléminaire pour conclure immédiatement, toutefois la preuve qui suit est suffi-
sament intéressante pour la présenter.
Ë Les deux normes ne sont toutefois pas équivalentes puisque (par exemple) la suite (fn )n≥2
dans E définie par fn (t) = tn /n tends vers zéro pour la norme k · k∞ mais pas pour la norme
k · k car kfn k∞ = 1/n alors que kfn k = n−1 + 1 + n. o
Ê N1 et N2 sont deux normes sur Rn [X] espace vectoriel de dimension finie n + 1 : elles
sont donc équivalentes.
Ë Avec Taylor-Lagrange nous avons pour tout x ≥ a, δ ≥ u > 0
un un+1 (n+1)
f (x + u) = f (x) + f 0 (x)u + · · · + f (n) (x) + f (ζ) où ζ ∈]x, x + u[,
n! (n + 1)!
soit
n n+1
δ n+1
f (x) + f 0 (x)u + · · · + f (n) (x) ≤ |f (x+u)|+ δ
u
(n+1)
kf k ≤ kf k++ kf (n+1) k
n! (n + 1)! (n + 1)!
nous avons donc pour tout x ≥ a
n
0 (n) u
sup f (x) + f (x)u + · · · + f (x) = sup |Px (u)| = N2 (Px ) ≤ M
0≤u≤δ n! 0≤u≤δ
0, 1, 2)
p
M1 ≤ 2M0 M2 .
La preuve repose sur l’inégalité de Taylor-Lagrange qui assure que pour tout réel x et tout
h>0:
h2 M2
|f (x + h) − f (x) − hf 0 (x)| ≤
2
En appliquant maintenant l’inégalité de Taylor-Lagrange entre cette fois-ci x et x − h on a
h2 M2
|f (x − h) − f (x) + hf 0 (x)| ≤ .
2
Ces deux inégalités et l’inégalité triangulaire donnent pour tout x ∈ R, h > 0
|f (x + h) − f (x − h) + 2hf 0 (x)| ≤ h2 M2
soit
hM2 M0
|f 0 (x)| ≤ + := g(h), ∀ h ∈ R?+ .
2 h
√
r
M2
Un calcul rapide montre que l’inf de g sur R?+ vaut 2M0 M2 pour h = soit finalement
2M0
p
|f 0 (x)| ≤ 2M0 M2 .
√
Ceci étant vrai pour tout x ∈ R, f 0 est bornée et M1 ≤ 2M0 M2 .
ê Avec plus de persévérence on démontre (voir par exemple ....) les inégalités de Kolmo-
gorov : soit f ∈ C n (R). Si f et f (n) sont bornées sur R, il en est de même des dérivées
intermédiaires et on a (avec les mêmes notations que dans la remarque précédente)
k(k−n) k k
1− n
Mk ≤ 2 2 M0 Mnn , k ∈ {0, 1, . . . , n}.
Ê Une réunion quelconque d’ouverts sera visiblement ouverte. Pour deux ouverts O1 , O2
et a ∈ O1 ∩ O2 , il existe b1 , b2 ∈ N? tels que Na,b1 ⊂ O1 , Na,b2 ⊂ O2 et par conséquent
Na,b1 b2 ⊂ O1 ∩ O2 : T est stable par intersection finie et définit bien une topologie sur Z.
Par construction, tout ouvert non vide est clairement de cardinal infini et la seconde assertion
résulte de l’égalité élémentaire
b−1
!
[
Na,b = Z \ Na+k,b
k=1
qui assure que Na,b est fermé comme complémentaire d’un ouvert.
Ë Comme tout entier n 6= ±1 admet un diviseur premier p et appartient donc à N0,p , nous
avons
[
Z \ {+1, −1} = N0,p .
p∈P
Maintenant, si P est fini alors ∪p∈P N0,p est fermé comme réunion finie de fermés et, comme
complémentaire d’un fermé, {−1, 1} est ouvert ce qui est absurde puisque tout ouvert non
vide est de cardinal fini : l’ensemble des nombres premiers est bien infini. o
Ê Montrer que l’on définit ainsi une norme sur C 0 ([0, 1]).
Ë Cette norme est-elle équivalente à la norme « sup » ?
n−1 1
P n ≥ 1 et (Ik )0 la subdivision de pas constant
Ë Soit n
de l’intervalle [0, 1]. Puisque la
série k ak converge, il existe 0 ≤ k0 ≤ n − 1 tel que
X 1X
ak ≤ ak .
k∈Ik0
n k≥0
N (fn ) = n1 et kfn k∞ = s−1 pour tout n ≥ 1 assure que les deux normes ne peuvent être
équivalentes sur C 0 ([0, 1]) (parce que cela implique qu’il ne peut exister de constante C > 0
telle que kf k∞ ≤ C.N (f ) ou bien parce que N (fn ) = n1 implique que la suite (fn )n converge
vers zéro dans (C 0 ([0, 1], N ) ce qui ne peut être le cas dans (C 0 ([0, 1], k.k∞ ) puisque les
applications fn y sont de normes s−1 ). q
Ê Nous donnons un exemple où l’intersection peut être vide : pour cela on munit l’ensemble
N des entiers naturels de la métrique
0 si m = n,
d(m, n) = 1
1 + sinon
m+n
Il est classique de vérifier que (N, d) est un espace métrique complet (l’espace est complet
car toute boule de centre n et de rayon r ∈]0, 1[ est réduite au singleton {n} avec pour consé-
quence que les seules suites de Cauchy sont les suites constantes clairement convergentes).
On considère alors les ensembles
In = {n, n + 1, n + 2, . . . } , n ∈ N?
n
avec le choix de notre métrique In n’est d’autre que la boule fermée Bf (n, 1 + ) ; En effet
2
n 1 1
x ∈ Bf (n, 1 + ) ⇐⇒ 1 + ≤1+ ⇐⇒ (x ≥ n)
2 n+x 2n
et bien entendu
\
In = ∅.
n≥1
Ë Ici encore la réponse est non, pour un exemple considèrons dans l’espace de Banach
(C 0 ([0, 1]) , k.k∞ ) la suite décroissante définie par
1
En = f ∈ C ([0, 1]) : f ([0, 1]) ⊂ [0, 1], f (0) = 0 et f (x) = 1 ∀ x ∈ [ , 1]
0
n
Ces ensembles sont clairement (pour la norme « sup ») fermés, bornés, convexes mais c’est
un exercice élémentaire de vérifier qu’aucune fonction continue ne peut appartenir à l’inter-
section des En . q
Ê Supposons qu’une telle norme N existe, il existe une constante C > 0 vérifiant
N (f 0 ) ≤ CN (f ), ∀ f ∈ C 0 (R)
inégalité absurde (N est une norme donc, N (fn ) 6= 0) : une telle norme ne peut exister.
Ë Sur R[X] la situation est différente, en effet il existe des normes pour lesquelles la déri-
vation n’est pas continue on peut par exemple considèrer
X
N (P = ak X k ) := sup |ak |
k
k
N (Pn ) = 1, N (Pn0 ) = n,
égalités qui rendent impossible la continuité de D dans (R[X], N ). Toutefois, à partir d’une
norme quelconque N sur R[X], il est facile d’en construire une nouvelle N1 rendant continue
la dérivation : il suffit pour cela de considérer
X
N1 (P ) = N (P (k) ), P ∈ R[X]
k≥0
P étant un polynôme, la somme est toujours finie, et les autres axiomes de la norme sont
trivialement réalisés, en outre
X X
∀ P ∈ R[X] : N1 (D(P )) = N1 (P 0 ) = N (P (k) ) ≤ N (P (k) ) = N1 (P )
k≥1 k≥0
√
Que N1 et N2 soient deux normes sur Q[ 2] est immédiat. Considérons alors pour n ∈ N :
√ √
un = (1 − 2)n , vn = (1 + 2)n ,
avec la formule du binôme on a
√ √
un = (1 − 2)n = an − bn 2, an , bn ∈ N,
√ √
vn = (1 + 2)n = an + bn 2,
un + vn
an =
2
si bien que limn un = 0, limn vn = +∞ et limn an = +∞, par conséquent
√
N1 (un ) = |1 − 2|n =⇒ lim N1 (un ) = 0,
n
N2 (un ) = |an | + |bn | =⇒ lim N2 (un ) = +∞
n
ces deux limites assurent la non équivalence des deux normes. q
i Remarque : « ...en dimension finie toutes les normes sont équivalentes... » oui mais
pour des espaces vectoriels sur le corps K = R ou C, ou plus généralement sur un corps à
boule unité compacte. C’est en effet la compacité de la boule unité dans Rd (ou Cd ) (pour
la norme « sup » par exemple) qui est l’ingrédient essentiel de la démonstration, ici le corps
est Q où la boule unité associée n’est plus compacte et plus rien ne marche.
ê Nous avons déja vu dans l’exercice précédent que Dn0 (C) est ouvert. Bien entendu
Dn0 (C) ⊂ Dn (C) donc
Pour l’inclusion inverse, supposons qu’il existe A ∈ Dno (C) admettant une valeur propre λ
d’ordre supérieur ou égal à deux, il existe alors P ∈ GLn (C) telle que A = P −1 DP avec
λ 0 0 ... 0
0 λ 0 . . . 0
0 0 λ3 . . . 0
D= . .
. . ... . . . ...
..
0 . . . 0 . . . λn
?
soit alors, pour k ∈ N
λ k −1 0 ... 0
Mλ 0 . . . 0
0 λ 0 ... 0
0 λ3 . . . 0 λ k −1
0 0 λ3 ... 0= .
Dk = .. .. . . .. avec Mk = .
. .
.. . . ... .. .. . . . 0 λ
. .
0 0 . . . λn
0 ... 0 ... λn
Le polynôme minimal de Dk est un multiple du polynôme minimal de Mk qui vaut (x − λ)2 ,
il n’est donc pas scindé à racines simple : la matrice Dk et par suite Ak = P −1 Dk P n’est pas
diagonalisable. Mais par construction, limk Ak = A, et la matrice A ne peut être intérieure
à Dn (C). Les matrices intérieures à Dn (C) ne peuvent avoir de valeurs propres multiples :
Dno (C) ⊂ Dn0 (C) d’où l’égalité avec (8).
ê Il s’agit maintenant de prouver que Dn (C) et Dn0 (C) sont denses dans Mn (C), on va
montrer que Dn0 (C) = Mn (C). Soit A ∈ Mn (C), il existe P ∈ GLn (C), une matrice T =
((tij )) ∈ Mn (C) triangulaire supérieure telles que
A = P T P −1 .
On pose alors :
(
1 si tii = tjj pour tout 1 ≤ i 6= j ≤ n,
α=
inf{ |t11 − tjj | : 1 ≤ i, j ≤ n} sinon
et on considère la suite de matrices (Tk := T + ∆k )k≥1 où
α
0 0 ... 0
k
α
0 2k 0 ... 0
∆k = .. ..
. . ... ... 0
α
0 0 . . . 0 nk
par le choix de α, les valeurs propres des matrices Tk , (k ≥ 1) sont deux à deux distinctes
car si
α α
tii + = tjj + avec tii 6= tjj
ik jk
on aurait :
α 1 1 α
|tii − tjj | = − < ≤ α
k i j k
i Remarques : ê Ce résultat est faux dans Mn (R) ceci fait l’objet de l’exercice suivant.
Remarquons tout de même le cas cas n = 2 qui est lumineux : l’application « discriminant »
a b
ϕ : A= ∈ M2 (R) 7→ ϕ(A) = (a − d)2 + 4bc ∈ R
c d
est continue, mais Ak ∈ D2 (R) =⇒ ϕ(Ak ) ≥ 0, si bien que
(F) (Ak )k ⊂ D2 (R) & lim Ak = A =⇒ ϕ(A) = lim ϕ(ak ) ≥ 0
k k
il suffit alors de choisir A ∈ M2 (R) à valeurs propres complexes non réelles de sorte que
ϕ(A) < 0 qui, avec (F) assure que A 6∈ D2 (R).
ê En fait l’adhérence dans Mn (K), (K = R ou C) de Dn (K) est l’ensemble des matrices
à polynôme caractéristique scindé sur K, fait qui explique la densité si K = C (dans C tout
polynôme est scindé) et la non densité si K = R, c’est l’objet de l’exercice suivant.
(la dernière inégalité n’est rien d’autre que |z| ≥ |Im(z)|.....) La réciproque est évidente
puisque (6) implique que les racines de P sont réelles.
Ì Le polynôme caractéristique d’une matrice diagonale dans Mn (R) étant scindé sur R,
les questions précédentes assurent que l’adhérence dans Mn (R) de l’ensemble des matrices
diagonalisables est inclue dans l’ensemble des matrices triangularisables. L’inclusion inverse
est facile à vérifier (raisonner comme dans la seconde partie de ’exercice précédent...). o
G = aZ + bZ
= {na + mb, n, m ∈ Z}
qam
= {na + , n, m ∈ Z}
p
a
= {(np + qm) , n, m ∈ Z}
p
a
= Z par Bezout.
p
ê Réciproquement s’il existe c > 0 tel que aZ + bZ = cZ il existe n, m ∈ Z? tels que a = nc
et b = mc soit a/b = n/m. CQFD
√
Ì Vu la question
√ précédente, G = Z +
√ 2Z est un sous-groupe dense de R, distinct de R
(1/2 ∈ G =⇒ 2 ∈ Q !) bien que Z et 2Z soient fermés dans R.
Í Gf est toujours non vide (0 ∈ Gf ) le reste suit tout aussi facilement.
Î Avec la première question, il est équivalent de montrer que
On a ainsi, si (x, y) ∈ R2 , f (x, y) = f (x + y, y). Autrement dit, u 7→ f (u, y), qui est a
priori 1-périodique, est également y-périodique. Si y est irrationnel, l’argument utilisé dans
a prouve que cette fonction est constante :
2
ê Soit f une fonction continue et Z -périodique de R dans C vérifiant f = f ◦ A avec
enfin
2 1
A= .
1 1
L’endomorphisme A a deux valeurs propres :
√ √
3− 5 3+ 5
µ= et λ=
2 2
de sorte que |µ| < 1 < |λ|. Soient u (resp. v) un vecteur propre de A associé à µ (resp. λ).
Si X = xu + yv où (x, y) est dans R2 on a , pour n ∈ N :
Mais f est continue sur R2 et Z2 -périodique donc uniformément continue sur R2 (vérification
en fin de solution). Il s’ensuit que :
(car xµn u → 0 quand n → +∞). Par suite : f (X) = f (xu). Ceci montre que f (xu + yv) ne
dépend pas de y. Mais en utilisant (4) pour n dans Z \ N, et en faisant tendre n vers −∞,
il vient f (X) = f (yv). Autrement dit f (xu + yv) ne dépend pas de x. En fin de compte, f
est constante sur R2 . ê
Soient enfin x et y dans R2 tels que ||x − y||∞ 6 δ. Soit m dans Z2 tel que x0 = x − m soit
dans [−1/2, 1/2]2 . Alors y 0 = y − m vérifie ||y 0 − x0 ||∞ 6 δ, donc (x0 , y 0 ) ∈ K 2 et :
Exercice 105 (Le théorème de Riesz dans un espace de Hilbert : c’est facile !)
Ê H est de dimension infinie : soit E := {en , n ∈ N} une famille orthonormale, elle est
visiblement bornée. Montrons qu’elle est fermée : soit x ∈ E : il existe une suite (eϕ(k) )k ⊂ E
√ converge vers x. On a donc limk keϕ(k+1) − eϕ(k) k = 0, mais pour tout i 6= j : kei − ej k =
qui
2. La seule alternative est alors que ϕ soit constante à partir d’un certain rang k0 i.e.
x = eϕ(k0 ) ∈ E qui est donc fermé. √
Comme pour la question précédente E n’est pas compact vu le recouvrement ouvert n∈N B(en , 22 ).
S
q
i Remarque : C’est bien sûr la version « Hilbert » du célèbre théorème de Riesz dont la
preuve est un peu plus délicate. Le cadre particulier des espaces de Hilbert permet cette très
simple et élégante approche.
Bien sûr que non ! il suffit de considérer pour le premier cas l’application f : Ω = R? → R
définie par
(
−1 si x > 0,
f (x) =
+1 six < 0.
1
et pour le second la fonction g : Ω = R? → R où g(x) = − . q
x
i Remarque : dans les deux cas c’est la non connexité de Ω qui fait capoter ce résultat
archi-classique.
Z 1
?
h pn , T p0 i = tn T ? p0 (t)dt
0
Z 1
= T (tn )p0 (t)dt
0
Z 1
= ntn−1 dt = 1
0
r
? ? 1
1 = |h pn , T p0 i| ≤ kpn k · kT p0 k ≤ kT ? p0 k < 1 pour n assez grand,
2n + 1
d’où la contradiction. q
non vide du connexe Mn (C), la seule alternative est F = Mn (C) : égalité absurde si n ≥ 2
et trivialle si n = 1.
Ê ê Si A est nilpotente, elle est semblable à une matrice triangulaire T = ((tij )) stricte
(i.e. tij = 0 pour i ≥ j). Il est bien entendu suffisant de prouver le résultat pour T . Pour
tout q ∈ N soit D(q) = diag(q n , q n−1 , . . . , q 2 , q) et notons
on a alors
(
uij = q i−j tij , ∀ n ≥ j > i ≥ 1
uij = 0, si i ≥ j
α
|uij | ≤ , 1 ≤ i, j ≤ n
q
ê Réciproquement soit k.k une norme sur Cn et |k.|k la norme subordonnée sur Mn (C).Pour
tout A ∈ Mn (C) et λ ∈ C valeur propre de A on a (classiquement) :
|λ| ≤ |kA|k.
Si bien que s’il existe une suite (Bk )k de matrices semblables à A et de limite nulle, les
matrices A et Bk ayant même spectre on démontre sans peine que
Exercice 111 (Topologie dans Mn (C) : les classes de conjugaison ) [45], [18].
Si A ∈ Mn (C) on définit la classe de conjugaison de A par
SA = P −1 AP, P ∈ GLn (R) .
Ë (i) ⇒ (ii). Soit A ∈ Mn (C) une matrice diagonale et B ∈ SA . Comme A est diagonali-
sable, son polynôme minimal est scindé à racines simples et est annulé par A, ie πA (A) = O.
En outre, M ∈ SA ⇒ ∀ k ∈ N : M k = P Ak P −1 ⇒ πA (M ) = P · πA (A) · P −1 = O. Ainsi
πA (M ) = O, ∀ M ∈ SA
et par continuité3 de M 7→ πA (M )
πA (M ) = O, ∀ M ∈ SA ,
ainsi πA (B) = O : la matrice B ∈ SA est donc annulée par un polynôme scindé à racine
simples et est donc diagonalisable. Pour s’assurer que B ∈ SA , il est maintenant suffisant
de montrer que les matrices A et B ont mêmes valeurs propres et mêmes dimension de sous
espaces propres.
Soient λ ∈ spec(A), Eλ = {x ∈ Cn : Ax = λx} le sous-espace propre associé et Fλ = {x ∈
Cn : Bx = λx} et posons nλ = dim Eλ , mλ = dim Fλ . Soit M ∈ SA , M étant semblable à
A : rang(M − λIn ) = n − nλ qui implique que les mineurs d’ordre n − nλ + 1 de M − λIn
sont nuls. Ces mineurs dépendant polynomialement et donc continuement des coefficients,
les mineurs d’ordre n − nλ + 1 de B sont aussi nuls, soit :
n − mλ = rang(B − λIn ) ≤ n − nλ
i.e.
mλ ≥ nλ , ∀λ ∈ spec(A).
En outre B étant diagonalisable
X X X
mλ = n = nλ ≤ mλ
λ∈C λ∈spec(A) λ∈spec(A)
3Attention ! par contre l’application A 7→ π n’est pas continue, on deux trouvera deux preuves dans ce document.
A
qui implique nλ = mλ , ∀ λ ∈ spec(A) qui assure à son tour que B est semblable à A :
B ∈ SA soit SA = SA .
(ii) ⇐ (i). Supposons maintenant par contraposée A non diagonalisable. On peut écrire A
sous la forme A = D + N avec D diagonalisable, N nilpotente non nulle et N D = DN . À
conjugaison prés, on peut supposer
D = diag(λ1 Im1 , . . . , λk Imk )
λ1 , . . . , λk étant les valeurs propres distinctes de A, m1 , . . . , mk leur multiplicité. Comme
N D = DN , N est de la forme
N = diag(N1 , . . . , Nk ), Ni ∈ Mmi (C), 1 ≤ i ≤ k,
et en effectuant les produits par blocs dans N D = DN on voit que l’on peut4 supposer les
Ni triangulaires supérieures strictes. Alors, comme dans la preuve de la première question
D(q)−1 DD(q)) = D et lim D(q)−1 N D(q)) = 0
q
qui implique
D = lim D(q)−1 (D + N )D(q) = lim D(q)−1 AD(q)
q q
Mais aussi, comme nous l’avons remarqué plus haut πA (B) = O : la matrice B annulée
par un polynôme scindé à racines simples est diagonalisable. Les matrices A et B ont donc
mêmes valeurs propres (comptées avec leur multiplicités) et sont diagonalisables : elles sont
semblables et B ∈ SA .
Ì ê Si A = λIn , λ ∈ C alors SA = {A} qui est bien bornée. Maintenant, si A n’est pas
une matrice scalaire l’endomorphisme f canoniquement associé à A n’est pas une homothétie
et un exercice classique d’algèbre linéaire5 assure de l’existence d’un vecteur v ∈ Cn tel que
la famille {v, f (v) = Av} soit libre. Considérons alors la base Bλ := {v, λAv, e3 , · · · , en } où
λ > 0 et soit Aλ ∈ A la matrice de A dans cette base. En observant la première colonne
de Aλ il vient kAλ k∞ ≥ λ−1 −→ +∞ lorsque λ tends vers 0 et SA n’est pas bornée6 dans
Mn (C).
4Choisir une base adaptée à la décomposition précédente dans laquelle chaque matrice N est triangulaire, la matrice D
i
reste inchangée vu sa forme et le choix de la nouvelle base.
5Consultez votre ouvrage favori...
6Qu’importe la norme, elles sont toutes équivalentes...
ê Une classe de similitude n’est jamais ouverte et est même d’intérieur vide car tous les
éléments d’une telle classe ayant même trace, elle est incluse dans un hyperplan affine de
Mn (C) donc d’intérieur vide.
Pour la connexité par arcs, ce n’est pas encore clair.... o 8
Ê En est un sous espace vectoriel normé de dimension finie de E il est donc complet
(consultez votre manuel favori) et donc fermé dans E (dans un espace métrique, toute partie
complète est fermée).
soit
kα0 a0 k
kαk ak k ≤ ,
3k
P
La série n αk ak est donc absolument convergente, et comme E est complet, elle converge.
Ì ................. o
f ainsi prolongée est visiblement continue et par convexité du cube [0, 1]d elle reste à valeurs
dans le cube, soit f ([0, 1]) = [0, 1]d .
Ë Munissons l’ensemble [−1, 1]Z des suites y = (yn )n∈Z vérifiant |yn | ≤ 1, ∀ n ∈ Z, de
la topologie produit ; par le théorème de Tychonoff c’est un espace compact. Comme pro-
duit
P dénombrable d’espace métrisable, K est aussi métrisable (par exemple par d(y, x) =
−|n|
n∈Z 2 |y n − x n |, la compacité peut alors d’ailleurs de démontrer par un procédé d’ex-
traction diagonal...).
Avec Alexandroff-Hausdorff, il existe donc une surjection continue ψ de C sur [−1, 1]Z :
t ∈ C 7→ ψ(t) := (ψn (t))n∈Z ; [−1, 1]Z étant muni de la topologie produit, les applications
coordonnées ψn : C → [−1, 1] sont continues.
On suppose ici que C ⊂ [0, 1/2] (par exemple l’image du Cantor standart par l’homéomor-
phisme f (x) = x/2) de telle sorte que
\
(C + n) (C + m) = ∅, ∀ m 6= m dans Z,
S
ce qui permet de définir la fonction f sur A = n∈Z (C + n) par
f (t + n) = ψn (t), t ∈ C, n ∈ Z.
Vu A et ψ, f est bien définie et continue sur A et comme dans la question précedente (R\A est
réunion dénombrable d’intervalles ouverts ; on peut aussi bien entendu invoquer le théorème
d’extension de Tietze) on prolonge f sur R en une fonction continue notée encore f à valeurs
dans [−1, 1]. La fonction f possède la propriété recquise : soit y = (yn )n ∈ [−1, 1]Z , il existe
t0 ∈ C tel que ψ(t0 ) = y i.e. f (t0 + n) = ψn (t0 ) = yn , ∀ n ∈ Z.
i Remarques : ê On peut bien entendu remplacer [−1, 1]Z par [−a, a]Z avec a > 0.
Toutefois il n’est pas envisageable de contruire une application continue sur R qui interpole
toutes les suites doublement (i.e. indicées dans Z) bornées et même toute les suite constantes :
en effet si tel était le cas on aurait
∀ α ∈ R, ∃ t ∈ R : f (t + n) = α, ∀n ∈ Z
qui implique f ([0, 1]) = R contredisant la continuité de f . Q
ê Soit {Mn }n∈Z une suite arbitraire d’entiers positifs ; en remplacant [−1, 1]Z par n∈Z [−Mn , Mn ]
la démontration précédente permet de travailler avec des suites y = (yn )n vérifiant |yn | ≤
Mn , ∀ n ∈ Z. Il est en particulier possible de construire une fonction continue qui interpole
toutes les suites (indicées dans N) bornées. Précisément : « Il existe f ∈ C (R, R) telle que
pour toute suite bornée y = (yn )n∈N il existe t ∈ R vérifiant f (t + n) = yn , ∀ n ∈ N » . En
effet, considèrons la fonction f obtenue avec la suite Mn = n si n ≥ 0 et Mn = 0 sinon. Avec
ce choix, pour toute suite bornée y = (yn )n∈N il existe k ∈ N tel que |yn | ≤ k, ∀ n ∈ N et il
existe alors t ∈ R tel que f (s + m) = 0 si m < k et f (s + m) = ym−k pour m ≥ k, le choix
t = s + k convient.
Ì Pour montrer que tout espace de Banach séparable X est isométrique7 de C ([0, 1]), on
procède par étape :
ê Première étape : Tout espace de Banach séparable est isométrique à un sous-espace
de C (K) où K est une partie compacte convexe métrisable d’un espace vectoriel topologique
E.
Désignons par X ? le dual topologique de X. Tout élément x ∈ X peut être considéré comme
une forme linéaire sur X ? par
(8) x(y ? ) := y ? (x), ∀ y ∈ X ?.
Si l’on muni X ? de la topologie σ(X ? , X) (où faible? , c’est la topologie la plus faible sur X ?
rendant continue les éléments de X considérés comme fonctionnelles par (8)), pour cette
topologie, la boule unité K de X ? (la boule unité de l’espace de Banach X ? ) est une partie
7X est isométrique à un sous-espace Y d’un Banach, s’il existe une application linéaire continue T : X → Y vérifiant
kT xkY = kxkX pour tout x ∈ X.
compacte, convexe et métrisable ([46] pages ? ? ?). Nous pouvons maintenant définir une
isométrie J de X dans C (K) par
J est clairement linéaire, que J(x) ∈ C (K) résulte de la définition de la topologie σ(X ? , X),
vérifions enfin que J est bien une isométrie. Pour tout k ∈ K, x ∈ X
L’inégalité contraire est une conséquence d’un corollaire du théorème de Hahn-Banach ([46]
page ....) : pour tout x ∈ X il existe kx ∈ K telle que kx (x) = kxkX . Alors
Puisque K est un espace compact convexe métrisable, il existe (c’est la seconde remarque
de Ê) une surjection continue ψ : [0, 1] → K. L’opérateur de composition défini sur C (K)
par
T (f )(t) = f (ψ(t)), t ∈ [0, 1]
est un opérateur linéaire de C (K) dans C ([0, 1]), c’est une isométrie car
par surjectivité de ψ. o
i Remarques : Dans l’article cité dans l’énoncé on trouvera d’autres applications, par
exemple
ê Un ensemble convexe « universel » : Pour tout d ∈ N? , il existe un compact convexe
K ⊂ Rd+2 tel que tout compact convexe de Rd soit isométrique à l’une des faces de K.
ê (Rudin, 1973) Il existe une suite uniformément bornée d’applications strictement posi-
tives (fn )n continues sur [0, 1] (on peut même prendre des polynômes) vérifiant
fn (x) → 0 pour tout x ∈ [0, 1].
Pour toute suite non bornée (λn )n de réels positifs, il existe x ∈ [0, 1] telle que
limsupn λn fn (x) = 0.
Ê Sinon, l’adhérence de A dans X est d’intérieur non vide : il existe donc un ouvert O de
˚X
X tel que A 6= ∅ et par conséquent, tout fermé de X contenant A contient O. Soit F un
fermé de Y contenant A : il existe un fermé L de X tel que F = L ∩ Y . A ⊂ F = L ∩ Y
implique A ⊂ L et par suite O ⊂ L. Ainsi, pour tout fermé F de Y contenant A : O ∩Y ⊂ F ,
donc
Y
O∩F ⊂A .
Il ne reste plus qu’à vérifier que l’ouvert O ∩ F est non vide : si c’est le cas, comme O est un
X X
ouvert non vide de A on aurait O ∩ Y = ∅ et A ⊂ Y =⇒ O ∩ A = ∅ soit O ⊂ (X \ A )
ce qui est absurde !
Ë Soit A ⊂ Y une partie maigre de Y . A est donc contenue dans un réunion dénombrable
de fermés (de Y ) d’intérieur vide (dans Y ) i.e.
[ Y
A⊂ Fn , Fn = Fn , F˚n = ∅.
n
X
Avec Ê les Fn sont rares dans X et comme A ⊂ ∩n Fn est maigre dans X comme réunion
dénombrable d’ensembles rares (dans X).
Í Soit X un espace de Baire séparé et sans points isolés, les singletons sont alors fermés
d’intérieur vide et X ne peut être dénombrable (sinon X = ∪n {an }....). o
ê Par sa définition, Oε est un ouvert de [0, 1] ; contenant la suite {an }, il est aussi dense.
Comme « dans un espace de Baire le complémentaire de toute partie maigre
est encore de Baire » (voir l’exercice précédent) : pour montrer que A est de Baire,
il sera suffisant de montrer que son complémentaire (dans [0, 1]) est maigre (i.e. réunion
dénombrable d’ensembles rares). Or
!
S [ \ S
[0, 1] \ A = p≥1 [0, 1] \ O1/p = [0, 1] \ On (1/p) := p≥1 Fp
p≥1 n≥1
| {z }
fermé de [0, 1] car ∩ de fermés
les fermés Fp = [0, 1] \ O1/p sont visiblement d’intérieur vide (sinon l’ouvert O1/p éviterai un
ouvert ce qui est absurde puisqu’il est dense car contenant la suit dense {an }) : A est bien
un espace de Baire.
ê Pour montrer que A n’est pas dénombrable on s’appuie sur le résultat suivant « un
espace de Baire séparé et sans points isolés est non dénombrable » (voir l’exercice
précédent). Montrons donc que A est séparé et sans points isolés.
ê Soient x 6= y dans A = ∩p O1/p et posons δ = |x − y|. Le diamètre de On (1/p) vaut 2/p2n
qui est < δ dès que p est suffisament grand et ceci pour tout n ∈ N. Pour un tel choix de
p, il existera nx ∈ N tel que x ∈ Onx (1/p), alors y 6∈ Onx (1/p) ; mais comme y ∈ A il existe
ny 6= nx ∈ N tel que y ∈ Ony (1/p) : A est bien séparé.
ê A contenant la suite dense {an } est sans points isolés.
ê A est enfin de mesure de Lebesgue nulle car pour tout ε > 0
X X
λ(A) ≤ λ(Oε ) ≤ λ(0n (ε)) = 2.2−n ε = 2ε.
n≥0 n≥0
D’où le résultat. o
Sinon, il existe (a, b) ∈ R2 tel que |f (a, b)| = c > 0. L’application y 7→ f (a, y) étant par
hypothèse continue sur R donc au point b, il existe ε > 0 tel que
(8) |f (a, y)| ≥ c/2, ∀ y ∈]b − ε, b + ε[.
Si on pose pour tout k ∈ N?
Ek = {y ∈]b − ε, b + ε[ : |f (x, y)| ≥ c/4, ∀ x ∈]a − 1/k, a + 1/k[} ,
on aura, avec (8)
[
]b − ε, b + ε[= Ek .
k≥1
Par le théorème de Baire, l’un au moins des ensembles Ek , disons Em est non rare (i.e.
◦ ◦
Em 6= ∅) et il existe un intervalle ]α, β[⊂ Em . Il n’est maintenant pas difficile de vérifier que
(4) |f (x, y)| ≥ c/4, ∀ (x, y) ∈]a − 1/m, a + 1/m[×]α, β[
ce qui fourni la contradiction désirée.
◦
Pour vérifier (4), soit (x, y) ∈]a − 1/m, a + 1/m[×]α, β[. Comme y ∈]α, β[⊂ Em ⊂ Em , il
existe une suite (yn )n dans Em convergente vers y et la continuité de x 7→ f (x, y) implique
lim (x, yn ) = (x, y) & (x, yn ) ∈]a − 1/m, a + 1/m[×]α, β[ =⇒ (|f (x, y)| ≥ c/4) .
n→∞
soit (4). o
Dans les espaces vectoriels normés (R[X], N1 ), (R[X], N1 ) étudier la continuité des
applications
( (
R[X] −→ R[X] R[X] −→ R[X]
T : LQ :
P 7−→ T (P ) = P (X + 1) P 7−→ L(P ) = P Q
i Que N1 , N2 soient deux normes sur R[X] ne mérite ici aucune précision (ne pas oublier
tout de même de justifier que N2 (P ) 6= +∞....).
Ê On a pour tout n ∈ N?
n
X
n n n
N1 (X ) = 1 et N1 (T (X )) = N1 ((X + 1) ) = Cnk = 2n
k=0
l’existence d’une constante C > 0 telle que N1 (T (P )) ≤ CN1 (P ) est donc sans espoir : T
est un endomorphisme discontinu de (R[X], N1 ).
N2 (T (P )) = sup P (x + 1)e−|x|
x∈R
= sup P (y)e−|y−1| où y = x + 1,
y∈R
où l’inégalité est justifiée car |y − 1| ≥ |y| − 1 et donc −|y − 1| ≤ 1 − |y|. T est donc un
endomorphisme continu de (R[X], N2 ) de norme inférieure ou égale à e.
d
X
(1) LQ (P ) = bk LX k (P )
k=0
i.e. ∀ n ∈ N, LX n ∈ Lc ((R[X], N1 )). De là, (1) et (2) donnent, pour tout polynôme P
d
X
N1 (LQ (P )) ≤ |bk |N1 (LX k (P )) = N1 (Q)N1 (P ).
k=0
n∈N:
N2 (LQ (X n )) supx∈R (b0 + b1 x + · · · + bd xd )xn e−|x|
=
N2 (X n ) nn e−n
(b0 + b1 n + · · · + bd nd )nn e−n
≥ ∼ |bd |nd −→ +∞ car d ≥ 1.
nn e−n n→∞ n→∞
LQ est donc discontinue pour la norme N2 dés que Q est non constant. Si Q est constant,
LQ = λIR[X] : elle est continue. o
On peut commencer par remarquer que ψ(e) 6= 0 et ker(ψ) fermé assurent que dist(e, ker(ψ)) >
0.
Pour x ∈ ker(ψ) nous avons
|ψ(e)| = |ψ(e − x)| ≤ |||ψ||| ke − xk,
soit, en passant à la borne inférieure lorsque x décrit ker(ψ)
|ψ(e)| ≤ |||ψ|||dist(e, ker(ψ)).
Pour obtenir l’inégalité inverse, par définition de la norme |||ψ|||, il est équivalent d’établir
|ψ(e)|
(8) ∀ y ∈ E, |ψ(y)| ≤ kyk.
dist(e, ker(ψ))
Cette formule évidente si ψ(y) = 0, est aussi homogène en y : il est donc suffisant (quitte
à remplacer y par yψ(e)/ψ(y) ) de l’établir pour y ∈ E vérifiant ψ(y) = ψ(e). Dans ce cas,
y − e ∈ ker(ψ) qui implique (classique) dist(e, ker(ψ)) = dist(y, ker(ψ)) ≥ kyk, et
kyk kyk
|ψ(y)| ≤ |ψ(y)| = |ψ(y)|
dist(y, ker(ψ)) dist(e, ker(ψ))
d’où (8), ce qui fallait démontrer. o
i L (Rd ) est de dimension finie (d2 ), il est donc suffisant de montrer que L est fermé
bornée dans L (Rd ).
ê L est borné : B f (0, a) ⊂ K car K est fermé ; comme K est aussi borné, il existe b > 0
tel que K ⊂ B f (0, b). Par conséquent
d x
∀ x ∈ R \ {0} : u a ∈ K ⊂ B f (0, b)
kxk
où encore
d a a
∀ x ∈ R \ {0} : ku(x)k ≤ kxk ⇒ |||u||| ≤ .
b b
f a
i.e. L ⊂ BL(R d ) (0, ).
b
ê L est fermé : Soit (un )n une suite dans L qui converge vers u ∈ L(Rd ). Pour tout x ∈ K
et tout n ∈ N, un (x) ∈ K ; par conséquent u(x) = limn un (x) ∈ K puisque K est fermé :
u ∈ L.
ê Si K est d’intérieur vide, la proprièté peut être fausse : par exemple, si K ⊂ H = Rd−1 ×{0}
(hyperplan de Rd ) toute affinité d’hyperplan fixe H et de vecteur directeur ed = (0, . . . , 0, 1)
conserve K et est donc dans L, mais l’ensemble de ces affinités n’est visiblement pas bornée
(considérer un definie par un (ei ) = ei , (1 ≤ i ≤ d − 1) et un (ed ) = ned ). o
Ê Soient f ∈ C2π et ε > 0. Par continuité uniforme de f sur R, il existe δ > 0 tel que
|x − y| < δ implique |f (x) − f (y)| ≤ ε. Pour n ∈ N? , considérons la subdivision σn de pâs
constant 2π/n de l’intervalle [0, 2π] ; alors 2π/n ≤ δ assure que l’application 2π-périodique
continue fn , égale à f en chaque point de la subdivision et affine par morceaux sur [0, 2π]
vérifie kf − fn k∞ ≤ ε. En outre, chaque application fn est continue sur R et de classe C 1 par
morceaux : la suite des sommes partielles de Fourier (Sk (fn ))k est donc (avec le théorème
de Dirichlet) une suite (de polynômes trigonométriques) qui converge uniformément vers fn .
Comme
kf − Sk (fn )k∞ ≤ kf − fn k∞ + kfn − Sk (fn )k∞
le résultat suit.
Ë Les sommes partielle de Fourier de toute application f ∈ C2π convergent au sens de Césaro
uniformément sur R vers f (c’est le théorème de Fejèr). D’où le résultat.
Soit ε > 0 par continuité uniforme de f sur R, il existe δ ∈]0, π[ tel que |x − y| < δ implique
|f (x) − f (y)| ≤ ε. Avec un tel choix comme un est paire et d’intégrale égale à 1 sur [0, 2π]
Z δ Z −δ Z π
|fn (x) − f (x)| ≤ ε un (t)dt + + 2kf k∞ un (t)dt
−δ −π δ
Z π Z π Z π
≤ε un (t)dt + 4kf k∞ un (t)dt = ε + 4kf k∞ un (t)dt (4)
−π δ δ
(car (1 + cos(δ))/2 ∈]0, 1[ assure que le second terme tends vers 0 avec n). En resumé nous
avons
∀ ε, ∃ nε : n ≥ nε sup |fn (x) − f (x)| ≤ ε.
x∈R
La suite de polynômes trigonométriques (fn )n est donc bien uniformément convergente sur
R vers f o
CONTINUITÉ
Exercice 121 (Les algèbres C 0 ([0, 1], R) et C 1 ([0, 1], R) sont-elles isomorphes ?)
[10]-2006.
Existe-t-il un isomorphisme d’algèbres entre C 0 ([0, 1], R) et C 1 ([0, 1], R) ?
Comme dans l’exercice précédent, les fonctions inversible de C (Rd ) sont celle qui ne s’an-
nulent pas sur K et φ unitaire assure que f inversible équivaut à φ(f ) où φ(f −1 ) inversible.
137
138/408 Petit Bestiaire d’Exercices pour l’Oral de l’Agrégation Interne Patrice Lassère
Notons φ un tel morphisme et soit f ∈ C (Rd ). Il existe x0 ∈ K tel que |f (x0 )| = supx∈K |f (x)| =
kf k∞ et quitte à considérer −f on peut supposer que kf k∞ = f (x0 ). L’application g :=
f − f (x0 )1 (1 est la fonction constante égale à 1 sur K) est par conséquent non inversible
dans C (Rd ), il en donc de même de son image φ(g) = φ(f ) − f (x0 )1. Ainsi, il existe x1 ∈ K
tel que φ(g)(x1 ) = 0 i.e. φ(f )(x1 ) = f (x0 ) qui implique kφ(f )k∞ ≥ kf k∞ . L’inégalité inverse
s’obtient de la même manière en considérant cette fois φ−1 . o
Il suffit de remarquer que pour toute application non constante f ∈ C 0 (R, R), la famille
(f n )n est libre. En effet, une relation de liaison λ0 + λ1 f + · · · + λd f d = 0 assure que le
polynôme P = λ0 + λ1 X + · · · + λd X d s’annule sur l’image de f qui est un intervalle non
réduit à un point d’après le théorème des valeurs intermédiaires et les hypothèses sur f : P
est donc le polynôme nul. La seule sous-algèbre de dimension finie dans C 0 (R, R) est celle
des fonctions constantes, elle est de dimension 1. o
Sans perdre de généralité supposons f strictement croissante. Soit c ∈]a, b[, on a donc :
sup f (x) := f (c− ) ≤ f (c) ≤ f (c+ ) = inf f (x).
a≤x<c c<x≤b
Supposons un instant que f (c) < f (c+ ). Par définition de f (c+ ) il existe une suite (xn )n ⊂
]c, b] qui vérifie (
limn→+∞ xn = c,
limn→+∞ f (xn ) = f (c+ ).
f étant croissante
f (xn ) ≥ f (c+ ) > f (c), ∀ n ∈ N
mais la propriété des valeurs intermédiaires nous assure qu’il existe
d ∈]c, b[ tel que f (d) = f (c+ ).
de sorte que
(4) inf f (x) ≥ inf f (x) = f (d).
c<x<d c<x≤b
les formules (4) et (8) sont contradictoires et par conséquent f (c+ ) = f (c). On procède de
même pour f (c− ) et les extrémités a, b : f est bien continue. q
Exercice 125 (Baireries ) Soit f : ]0, +∞[ une application continue telle que
x
∀ x > 0, lim f = 0.
n→+∞ n
A-t-on
lim f (x) = 0 ?
x→0+
On peut donc appliquer le théorème de Baire : un au moins des Fk , disons Fk0 possède un
intérieur non vide. Il existe donc a > 0, δ > 0, k0 ∈ N tels que
]a − δ, a + δ[⊂ Fk0
quitte à diminuer δ on peut toujours supposer que a ≤ kδ0 .
Soit alors 0 < x < δ et n = xa , alors a − δ ≤ a − x ≤ nx < a < a + δ et n ≥ k si bien que
nx ∈ Fk0 qui implique nx
|f (x)| = f ≤ ε,
n
En résumé, pour tout ε > 0, il existe δ > 0 tel que 0 < x < δ implique |f (x)| ≤ ε i.e.
lim f (x) = 0,
x→0+
f (x) f (h)
6=
0= = 1.
x h
Ñ Du même tonneau : les fonctions mid-convexes. Une fonction f : R → R est dite
mid-convexe si
x+y 1
f ≤ f (x) + f (y) , ∀ x, y ∈ R.
2 2
Toute fonction convexe est mid-convexe et il n’est pas difficile de démontrer qu’une fonction
mid-convexe continue est convexe. L’existence de fonctions mid-convexes qui ne soient pas
convexes dépend encore une fois de l’axiome du choix. L’axiome du choix étant admis,
considérons une base de Hamel (bi )i de R et soit encore une fois f définie sur R par
X X
∀x = ak b k : f (x) = ai f (bi )
i∈Ix i∈Ix
f est mid-convexe mais en choisissant des valeurs convenables pour f (bi ) il n’est pas difficile
de faire en sorte que f ne soit pas convexe. q
Ê Toute application convexe est bien entendu mid-convexe2 ; pour la réciproque, classique
mais plus délicate, on propose deux solutions :
1avec cette construction il est même clair que toute application g : H → R se prolonge en une fonction additive sur R
2Ou encore J-convexe en hommage à J.L.W.V.Jensen qui introduit cette notion en 1906.
ê Première solution : C’est la plus classique, mais aussi peut être la plus délicate à pré-
senter pour un oral, elle consiste à prouver (8) sur une partie dense de I puis de la prolonger
à tout l’intervalle par continuité. On procède par étapes en démontrant successivement
2n 2n
!
1 X 1 X
(1) f n
xj ≤ n f (xj ), ∀ n ∈ N? , xj ∈ I.
2 j=1 2 j=1
n
! n
1X 1X
(2) f xj ≤ f (xj ), ∀ n ∈ N? , xj ∈ I.
n j=1 n j=1
m m m m
(3) f n x + 1 − n y ≤ n f (x) + 1 − n f (y), ∀ m, n ∈ N? , x, y ∈ I.
2 2 2 2
Par densité3 de { 2mn , m, n ∈ N? } dans [0, 1], la continuité de f sur I permet d’étendre (3) à
tout p ∈ [0, 1] i.e.
f (px + (1 − p)y) ≤ pf (x) + (1 − p)f (y), ∀ x, y ∈ I, p ∈ [0, 1].
f est bien convexe sur I.
ê Seconde solution : Elle (voir [16]) repose sur le théorème des valeurs intermédiaires.
Si f n’est pas convexe il existe a < c < b tels que le point (c, f (c)) soit au dessus de la
corde reliant les points (a, f (a)) et (b, f (b)) ; et quitte à retrancher à f la fonction affine
(donc convexe donc ne modifiant pas la quantité f x+y 2
− 21 f (x) − 21 f (y)....) x 7→ f (a) +
f (b)−f (a)
b−a
(x − a), on peut supposer que f (a) = f (b) = 0 et dans ce cas f (c) > 0.
Alors, l’ensemble { x ∈ [a, c] : f (x) = 0} est non vide (il contient a) majoré et fermé (f est
continue) : il admet donc un plus grand élément u qui est strictement plus petit que c car
f (c) > 0. Par le théorème des valeurs intermédiaires et la définition de u, f est strictement
positive sur ]u, c]. De la même manière, on démontre l’existence d’un point v ∈]c, b[ tel que
f soit strictement positive sur [c, v[. Mais alors f ( u+v
2
) > 0 alors que f (u) = f (v) = 0 ce qui
exclu la réalisation de (8) sur l’intervalle.
Ë Avec l’axiome du choix, on peut construire des fonctions mid-convexes non convexes,
ce sont des objets pathologiques nulle part continus donc le graphe est dense dans R2 , nous
les avons deja rencontrés (et construits) dans l’exercice précédent, question Ñ. o
)
Montrer que les solutions continues sur R et non identiquement nulles de l’équation
fonctionnelle
p
(8) f ( x2 + y 2 ) = f (x)f (y), ∀ x, y ∈ R
2
sont de la forme f (x) = f (1)x .
3La densité est immédiate : pour tout intervalle ]a, b[⊂ [0, 1] considérer n ≥ 1 tel que b − a > 1
, alors, par le principe des
2n
tiroirs, un multiple de 21n se trouve forcément dans ]a, b[.
i Remarques : ê Une telle fonction n’est donc pas injective et par conséquent S 1
n’est homéomorphe à aucune partie de R (pour un tel résultat on peut plus classiquement
si un tel homéomorphisme existe considérer sa restriction à S 1 moins un point (qui reste
connexe !) mais dont l’image ne le reste plus...).
ê On peut aussi considérer (c’est en fait la même preuve..) la fonction g : θ ∈ [0, π] 7→
g(θ) = f (eiθ ) − f (ei(θ+π) ), vu que g(0) = −g(π) il ne reste plus qu’à appliquer le théorème
des valeurs intermédiaires pour conclure.
ê En guise « d’application », sur tout méridien terrestre il existe donc à chaque instant
deux point antipodaux en lesquels la température est la même. On peut aussi démontrer
qu’à chaque instant il existe sur terre deux point antipodaux pour lesquels température et
pression sont identiques.
ê Une autre belle application est le « problème de gâteau » : pour tout gâteau connexe
borné (dans R2 ...) il existe deux perpendiculaires qui le divisent en quatre parts égales. Voir
l’exercice ci-dessous.
g : I −→ J & f : J −→ R
on suppose g continue sur I et f ◦ g continue sur I ; montrer que f est continue
sur g(I).
Soit donc y ∈ g(I), et supposons f discontinue en y : il existe ε > 0 et une suite (yn )n ⊂ g(I)
tels que
(il faut ici se garder de croire que la suite (xn )n est nécessairement convergente ou même
admet une sous-suite convergente, mais les pré-images g −1 ({yn }) nous laissent suffisament de
place pour construire une nouvelle suite elle convergente et ceci par l’intervention judicieuse
du théorème des valeurs intermédiaires).
Quitte à considérer une sous-suite supposons (yn )n monotone et même décroissante (même
raisonnement si la suite est croissante). Nous avons donc dans g(I)
α ≤ x0
(on peut bien entendu avoir l’inégalité contraire, mais le raisonnement est le même) Les deux
formules précédentes et le théorème des valeurs intermédiaires assurent pour tout n ∈ N
l’existence d’un réel x0n ∈ [α, x0 ] vérifiant g(x0n ) = g(xn ) = yn . De cette nouvelle suite
inclue dans le compact [α, x0 ] nous sommes donc assurés de pouvoir extraire un sous-suite
convergente (xnk )k de limite l ∈ [α, x0 ] ⊂ I.
Par continuité de g sur I
Supposons qu’une telle fonction existe et soient x1 6= x2 tels que f (x1 ) = f (x2 ) = b. Alors
pour tout x ∈ R \ {x1 , x2 }, f (x) 6= b et par suite on a ou bien f (x) > b pour tout x ∈]x1 , x2 [
ou bien f (x) < b pour tout x ∈]x1 , x2 [. Dans le premier cas, il existe un unique x0 ∈]x1 , x2 [
tel que c = f (x0 ) = max{f (x), x ∈ [x1 , x2 ]}. En effet, sinon f va prendre sur [x1 , x2 ]
au moins trois fois certaines valeurs (faire un dessin si le max est atteint en deux points
distincts, c’est le TVI). Ainsi il doit exister exactement un réel x00 en dehors de [x1 , x2 ] tel
que f (x00 ) = f (x0 ) = c > b. Mais alors, toujours par le théorème des valeurs intermédiaires
tous les réels de ]b, c[ seront atteints au moins trois fois. Contradiction. On procède de manière
analogue si f < b sur ]x1 , x2 [. q
Supposons que f soit discontinue en un point x ∈ R. Il existe alors une suite (xn )n conver-
gente vers x telle que (f (xn ))n ne converge pas vers f (x). Il existe donc ε > 0, une suite
d’entiers nk > k vérifiant
|f (xnk ) − f (x)| ≥ ε.
Ainsi pour tout entier k on a f (xnk ) ≥ f (x) + ε > f (x) ou bien f (xnk ) ≤ f (x) + ε < f (x).
L’une au moins de ces deux inégalités est réalisée pour une infinité de k, sans perdre de
généralité et quitte à extraire une sous-suite supposons que la première soit vérifiée pour
tout k. Considérons alors q ∈ Q tel que f (x) + ε > q > f (x). Nous avons alors f (xnk ) >
q > f (x), ∀ k ∈ N et f vérifiant la propriété des valeurs intermédiaires, il existe pour tout
k ∈ N, yk ∈ (x, xnk ) tel que f (yk ) = q, ce qui ne veut rien dire d’autre que la suite (yk )k
convergente vers x est incluse dans f −1 ({q}) fermé de R : x ∈ f −1 ({q}) i.e. f (x) = q ce qui
est bien entendu absurde. q
(
x2 sin( x1 ) si x ∈ R?
f (x) =
0 si x = 0.
ê On peut aussi contruire ([5],pages 79-80) une fonction nulle part continue mais vérifiant
la propriété des valeurs intermédiaires sur tout intervalle de R, un exemple toutefois bien
délicat et fortement déconseillé pour l’oral...
ê pour en savoir plus la lecture de l’article de J.B. Hiriart-Urruty « Que manque-t-il à une
fonction vérifiant la propriété des valeurs intermédiaires pour être continue ? » ([10], ref.
exacte ? ?) est fortement conseillée.
Ê Il faut commencer par remarquer que f n’est pas continue sur R. En effet, f continue et
bijective sur R sera strictement monotone, par exemple strictement croissante ; dans ce cas
en considérant x0 ∈ R tel que f (x0 ) = 0 on aurait f (x) > 0 pour x > x0 et f (x) < 0 pour
x < 0 ce qui est bien sûr absurde puisque f (R) = R?+ . f n’est donc pas continue sur R.
Ë Supposons maintenant que f ne possède qu’un nombre fini de points de discontinuité
x1 < x2 < · · · < xn . f est alors strictement monotone sur chaque intervalle ]−∞, x1 [, ]x1 , x2 [, . . . , ]xn , +∞[
et par le théorème des valeurs intermédiaires f (] − ∞, x1 [), f (]x1 , x2 [), . . . , f (]xn , +∞[) sont
des intervalles ouverts deux à deux disjoints, donc
n−1
!
[
R?+ \ f (] − ∞, x1 [) ∪ f (]xi , xi+1 [) ∪ f (]xn , +∞[)
i=1
n−1
!
[
R\ ] − ∞, x1 [∪ ]xi , xi+1 [∪]xn , +∞[ = {x1 , . . . , xn }
i=1
Que f soit bijective, c’est clair, on a tout fait pour. f est aussi continue à l’origine car
Enfin, puisque
1
f −1 ( ) = 2n + 1,
2n + 1
f −1 est bien discontinue à l’origine. q
i Remarques : ê Cet exemple est un garde-fou contre la tentation d’affirmer que l’appli-
cation réciproque d’une application continue en un point est continue en l’image de ce point :
le théorème standart du cours impose à f d’être bijective et continue sur tout un intervalle
ouvert I pour pouvoir affirmer que f −1 sera continue sur f (I). En outre la construction est
facile à mémoriser : pour que f −1 soit discontinue à l’origine il suffit de construire une suite
ne tendant pas vers zéro telle que son image par f tende vers zéro ; ayant envoyé tous les
nombres pairs sur les entiers nous pouvons faire ce que nous voulons des impairs qui sera
notre suite, reste plus qu’à bricoler un peu pour conserver la bijectivité...
ê Quitte à modifier légérement f (considérer f 3 à la place de f ), on peut même produire
un exemple où f est dérivable à l’origine (f 3 est dérivable en x = 0 car |f (x)| ≤ |x| =⇒
f 3 (x) = o(x2 ) à l’origine).
Z et Q étant tous deux dénombrables, une telle bijection existe. Soit a ∈ Z, pour tout ε > 0
si 0 < η < 1 : (z ∈ Z & |z − a| < η) =⇒ z = a et par suite |f (z) − f (a)| = 0 < ε. f est donc
continue sur Z. T
Par contre si b ∈ Q et 0 < ε < 1 il existe pour tout η > 0 un rationnnel c ∈ Q (]a − η, a + η[\{a}).
alors f −1 étant injective |f −1 (a) − f −1 (c)| ≥ 1 > ε, i.e. f −1 est discontinue au point a. q
i Remarque : C’est cette fois-ci la non-connexité de l’intervalle de départ (ici Z) qui rend
possible la non-continuité de l’application réciproque.
exp(f (z)) = z, ∀z ∈ C? ?
∃ N ∈ N : g ≡ 2iN π
et par suite l’application de R dans C définie par
On vérifie facilement que f est dérivable en x = 0 avec f 0 (0) = 0, elle est donc dérivable sur
R et un calcul élémentaire montre f 0 (x) ∈ [0, 1[. Avec le théorème des valeurs intermédiaires,
si x 6= y il existe c ∈ (x, y) tel que
f réduit donc strictement les distances, mais bien entendu, l’équation f (x) = x est sans
solution. q
i Remarque : le théorème du point fixe ne s’applique pas ici, en effet, f n’est pas contrac-
tante (i.e. ∃ 0 < k < 1 : ∀ x 6= y ∈ R : |f (x) − f (y)| ≤ k|x − y|) elle l’est localement mais
pas globalement (car f 0 (x) → 1− lorsque x → +∞).
Ê Supposons f continue au point a ∈ A : pour tout ε > 0 il existe δ > 0 tel que f (x) ∈
Bδ2 (f (a), ε/2) pour tout x ∈ Bδ1 (a, δ) ∩ A. Par conséquent d2 (f (x), f (y)) < ε pour tout
x, y ∈ Bδ1 (a, δ) ∩ A soit of (a) = 0.
Réciproquement si of (a) = 0, alors étant donné ε > 0 il existe δε > 0 tel que 0 < δ < δε
implique diamf(A ∩ Bδ1 (a, δ)) < ε i.e. d1 (x, a) < δ implique d2 (f (a), f (x)) ≤ diamf(A ∩
Bδ1 (a, δ)) < ε.
et posons alors
1
fA (x) = gA (x) 1Q (x) − .
2
gA et donc fA est bien définie sur R ; nous allons vérifier que l’ensemble des points de
discontinuité de fA est précisément A.
ê Soit x ∈ Å, on peut alors construire deux suites (x ) ∈ A ˚ ∩ Q, (y ) ∈ Å ∩ (R \ Q)
n n n n
toutes deux convergentes vers x. Vu sa définition, gA (x) > 0 si x ∈ A, et vu celle de fA :
fA (xn ) > 0, fA (yn ) < 0 ∀ n ∈ N si bien que fA continue au point x implique fA (x) = 0 ce
qui est absurde puisque x ∈ A : fA est donc discontinue sur l’intérieur de A.
On procède identiquement si x ∈ A ∩ ∂A : fA (x) 6= 0 mais on peut approcher x par une suite
(zn )n ⊂ R \ A soit fA (zn ) = 0 d’où la discontinuité.
A = Å ∪ (∂A ∩ A) la fonction fA est bien discontinue sur A.
ê Sur R \ A : fA ≡ 0. Soit (xn )n ⊂ R une suite convergente vers x ∈ R \ A, si (xk )k ⊂ A
alors pour tout n ∈ N il existe un entier kn tel que k ≥ kn =⇒ xk 6∈ Fn (en effet, sinon, il
existerai une infinité de xk dans au moins un Fn soit x ∈ F n = Fn ⊂ A ! ). Ainsi :
1 1 1
∀ n ∈ N, ∃kn ∈ N : k ≥ kn =⇒ gA (xk ) ≤ n+1 + n+2 + · · · = n
2 2 2
qui assure que lim gA (xk ) = 0 = gA (x) : gA est bien continue au point x et donc sur R \ A.
k→+∞
Í Non, par exemple toute fonction définie sur un espace métrique discret est continue.
Î On considère une suite (fn )n d’applications continues sur X et simplement convergente
sur X vers f . Posons pour m ∈ N, ε > 0
Aε˚
[
Aε (m) = {x ∈ X : |f (x) − fm (x)| ≤ ε}, A(ε) = (m).
m≥1
Nous allons vérifier que C := ∩n≥1 A1/n est l’ensemble des points de discontinuité de f :
Supposons pour commencer que f soit continue en un point x ∈ X. Puisque f (x) =
limn fn (x), il existe m ∈ N tel que
ε
|f (x) − fm (x)| ≤ .
3
Par continuité de f et fm en x il existe une boule B(x, r) telle que
ε ε
∀ y ∈ B(x, r) : |f (y) − f (x)| ≤ et |fm (y) − fm (x)| ≤ .
3 3
si bien que [ [ [
X \ A(1/k) ⊂ Fm (1/k) \ (Fm (1/k))◦ .
k≥1 k≥1 m≥1
X \ C est inclu dans une réunion dénombrable d’ensembles maigre : C contient donc une
intersection d’ouverts denses, par le théorème de Baire, C est dense dans X.
Ï Si une telle fonction existe, R \ Q serait un Fσ d’intérieur vide, donc maigre et par suite
R = (Q) ∪ (R \ Q) est lui aussi maigre comme réunion de deux ensembles maigres ce qui est
absurde au vu du théorème de Baire. o
dans la question Ì et on trouvera dans l’exercice ci-dessous un autre exemple beaucoup plus
classique.
1
Ê Soit x ∈ Q? , il existe q ∈ N? tel que f (x) = q
> 0 mais aussi une suite (xn )n d’irrationels
de limite x soit
1
lim f (xn ) = 0 6= = f (x)
n q
i Remarques : La seconde étape résulte aussi (très bon exercice sur les suites et sous-
suites) du fait suivant (en fait le même) : « si une suite de rationnels (pn /qn )n converge vers
un irrationnel alors limn qn = +∞ ».
i Nous venons donc de démontrer que toute fonction continue au sens de Cesàro en au
moins un point vérifie l’équation fonctionnelle de Cauchy : f (a)+f (b) = f (a+b), ∀ a, b ∈ R,
une équation que nous avons étudiée dans un autre exercice (voir .........). Nous savons en
particulier que f sera de la forme f (x) = ax + b dès qu’elle sera continue en au moins un
point ; mais ici c’est au sens de Cesàro que f est continue et il n’est donc pour le moment
pas possible de conclure... Les deux prochaines questions vont justement établir l’hypothèses
manquante à savoir la continuité de f à l’origine.
ê L’existence de la suite (yn )n revient à résoudre les équations
y1 + y2 + · · · + yn = nxn , ∀ n ∈ N? .
Soit
yn = nxn − (n − 1)xn−1 , n ∈ N?
(avec la convention x0 = 0). o
ê Comme
y1 + y2 + · · · + yn
lim = lim xn = 0
n→∞ n n→∞
i La continuité au sens de Cesàro est donc une propriété beaucoup plus contraignante
que la continuité usuelle ; ce résultat n’était à priori, absolument pas prévisible. En effet, les
relations éventuelles entre continuité ordinaire et continuité au sens de Cesàro ne sont pas
évidentes : il y a plus de suites (C)-convergentes, mais d’un autre coté, la (C)-convergence
est plus faible que la convergence usuelle ; autrement dit, une fonction continue au sens de
Cesàro doit sur beaucoup plus de suites faire quelque chose moins fort que la continuité
ordinaire.
Rx
Exercice 143 (L’équation fonctionnelle f 2 (x) = 0
(f 2 (t) + f 02 (t)) dt + 2007. )
[28]
Déterminer les fonctions f ∈ C 1 (R) vérifiant
Z x
2
f 2 (t) + f 02 (t) dt + 2007,
(8) f (x) = x ∈ R.
0
Les deux fonctions de part et d’autre de l’égalité (8) seront égales si elles ont même dérivée
et coïncident à l’origine. Dérivons (8), on tombe sur
2f (x)f 0 (x) = f 2 (x) + f 02 (x), x ∈ R,
ou encore (f − f 0 )2 (x) = 0, x ∈ R, soit f = f 0 et finalement f (x) = Cex , x ∈ R. Enfin
l’évaluation à l’origine donne√f 2 (0) = 2007 ; les solutions de l’équation fonctionnelle (8) sont
les deux fonctions f (x) = ± 2007ex . o
Ë Sans perdre de généralité, supposons a > b > c et soit f une solution de l’équation, comme
dans la première question on peut écrire pour tout x ∈ R
bx cx
f (x) = −f −f
a a
2 2
bx bcx cx
=f 2
+ 2f 2
+f
a a a2
3 2 2 3
bx b cx bc x cx
= −f 3
− 3f 3
− 3f 3
−f
a a a a3
n k n−k
n
X
k b c x
= (−1) Cn f n
∀ n ∈ N,
k=0
a
où la dernière égalité résulte d’une récurrence élémentaire sur n. f étant de classe C ∞ , on
peut dériver cette dernière expression à tout ordre :
n k n−k N k n−k
(N ) n
X
k b c (N ) b c x
(F) ∀ x ∈ R, ∀ N, n ∈ R : f (x) = (−1) Cn n
f n
.
k=0
a a
Fixons x dans R, comme a > b > c > 0 on a |bk cn−k x/an | < |bn x/an | ≤ |x| pour tout n ∈ N
et k ∈ {0, 1, . . . , n}. Donc, avec (F) nous avons pour tout x ∈ R, n, N ∈ N :
n k n−k N
(N )
X
k b c
|f (x)| ≤ Cn n
kf (N ) k[−x,x]
k=0
a
n nN N n
(N )
X
k b (N ) 2b
≤ kf k[−x,x] Cn = kf k[−x,x]
k=0
a aN
Fixons N suffisament grand pour que 0 < 2bN /aN < 1 et x ∈ R, comme pour tout n ∈ N
N n
(N ) (N ) 2b
|f (x)| ≤ kf k[−x,x] ,
aN
on en déduit en faisant tendre n vers +∞ que f (N ) (x) = 0 pour tout réel x : f est donc un
polynôme f (x) = ad xd + · · · + a1 x + a0 . L’équation fonctionnelle s’écrit alors
ad (ad + bd + cd )xd + · · · + a1 (a + b + c) + 3a0 = 0, ∀ x ∈ R.
a, b, c étant strictement positifs : la seule alternative est ad = · · · = a1 = a0 = 0. f est
donc identiquement nulle. Réciproquement la fonction identiquement nulle vérifie l’équation
fonctionnelle, c’est donc l’unique solution. o
Soit f une solution éventuelle. Pour a < b deux rationnels, n ∈ N? et i ∈ {0, 1, . . . , n} posons
ai = a + i b−a
n
. Il est clair que les réels ai sont rationnels et l’inégalité triangulaire nous donne
n−1 n−1
X X (b − a)2
|f (a) − f (b)| ≤ |f (ai ) − f (ai−1 )| ≤ 7 |ai − ai−1 |2 = 7 .
i=0 i=0
n
En faisant tendre maintenant n vers +∞ on en déduit que f (a) = f (b) : f est donc constante
sur Q.
Montrons que est constante sur R : On a déja f (q) = r pour tout q ∈ Q ; pour x ∈ R
considérons une suite de rationnels (qn )n qui converge vers x, alors
|f (x) − r| = |f (x) − f (qn )| ≤ 7|x − qn |2 −→ 0.
n→+∞
i.e. f (x) = r : f est bien constante. Réciproquement, il est facile de vérifier que les fonctions
constantes sont solutions du problème. o
DÉRIVABILITÉ
1 désignant la fonction constante x 7→ 1(x) = 1 nous avons D(1.1) = D(1) = D(1) + D(1),
soit D(1) = 0 et par suite D est nulle sur toutes les applications constantes. Comme pour
toute f ∈ C 0 (R, R), f = (f − f (0)) + f (0) ⇒ D(f ) = D(f − f (0)) il est suffisant de
se concentrer sur la restriction de D sur les applications nulles
√ à l’origine ; pour une telle
application D(f 2 ) = 0 si bien que pour f ≥ 0 : D(f ) = D(( f )2 ) = 0. Dans le cas général,
comme il est toujours possible d’écrire f comme différence de deux fonctions continues,
positives et nulles à l’origine (f = max{f (x), 0} − max{−f (x), 0}) on a encore D(f ) = 0 et
la seule forme linéaire qui convienne est la forme identiquement nulle. o
Exercice 147 (Deux fonctions f, g dérivables telles que f 0 g 0 ne soit pas une
dérivéee ) [49]
Montrer qu’il existe deux fonction dérivables f, g : R → R telles que f 0 g 0 ne soit
pas une dérivée.
161
162/408 Petit Bestiaire d’Exercices pour l’Oral de l’Agrégation Interne Patrice Lassère
g est continue sur R, c’est donc une dérivée. Par conséquent, h := g − f 0 est une dérivée : il
existe donc une application dérivable H : R → R telle que
(
cos(1/x) si x 6= 0
H 0 (x) = h(x) = g(x) − f 0 (x) =
0 si x = 0.
Considérons alors
1 + cos(2/x)
2 si x 6= 0
h (x) = 2
0 si x = 0,
nous allons vérifier que h2 n’est pas une dérivée et répond donc à notre problème. Supposons
qu’il existe une application dérivable D : R → R telle que D0 = h2 , en notant H̃(x) =
H(x/2) nous avons
cos(2/x)
0 si x 6= 0
H̃ (x) = 2
0 si x = 0,
qui implique
1
0 + H̃ 0 (x) si x 6= 0
D (x) = 2
H̃ 0 (x) si x = 0,
ou encore
0 1 si x 6= 0
D0 (x) − H̃ 0 (x) = D(x) − H̃(x) = 2
0 si x = 0.
Mais il est bien connu (cf autre exo) qu’une dérivée ne peut avoir une discontinuité de
première espèce : contradiction et l’application h2 est bien sans primitive. q
Si l’une des quatre applications f (i) , 0 ≤ i ≤ 3 n’est pas de signe constant le problème est
trivial. Supposons donc f, f 0 , f 00 , f 000 de signe constant, alors f et f 00 sont de même signe.
Pour cela, si f 00 > 0 la formule de Taylor-Lagrange nous donne pour tout x ∈ R
x2 00
f (x) = f (0) + xf (0) + f (cx ) > f (0) + xf 0 (0)
0
2
et f (0) + xf 0 (0) est certainement positif pour x suffisament grand et du signe de f 0 (0) : f
étant supposée de signe constant f > 0 sur R(on procède de manière analogue si f 00 < 0).
Ainsi f (x)f 00 (x) > 0, ∀ x ∈ R. Ce même raisonnement vaut pour le couple f 0 , f 000 et le
résultat est démontré. q
i Remarque : L’hypothèse C 3 est en fait superflue : trois fois dérivable suffit si l’on se
souviens qu’une dérivée possède toujours la propriété des valeurs intermédiaires.
ê F est clairement continue sur [a, b], dérivable sur ]a, b[ avec
f 0 (x) g 0 (x) h0 (x)
F 0 (x) = det f (a) g(a) h(a)
f (b) g(b) h(b)
f 0 (c) 1 0
F 0 (c) = det f (a) a 1 = 0
f (b) b 1
0
f (c) g 0 (c) 0
F 0 (c) = det f (a) g(a) 1 = 0
f (b) g(b) 1
nous donne
hn−1 (n−1) hn
f (x + h) = f (x) + hf 0 (x) + · · · + f (x) + f (n) (x + θ(h)h).
(n − 1)! n!
(n+1)
Si de plus f (x) existe et est différent de zéro,montrer que
1
lim θ(h) = .
h→0 n+1
Par Taylor-Young
par conséquent Iϕ ∪ Iψ est un intervalle non vide de R (la réunion de deux connexes non
disjoints est toujours connexe). Ainsi, pour tout réel λ ∈ (f 0 (x), f 0 (y)) il existe t ∈]x, y[
tel que ϕ(t) = λ ou ψ(t) = λ, si par exemple ϕ(t) = λ (idem avec ψ) le théorème des
accroissements finis nous assure qu’il existe ζt ∈]x, y[ tel que
f (x) − f (t)
λ = ϕ(t) = = f 0 (ζt ).
x−t
finalement
f (x) − f (y)
ϕ : (x, y) ∈ C 7→ ϕ(x, y) = ,
x−y
est visiblement continue, ϕ(C ) est donc connexe dans R : c’est un intervalle, et par le
théorème des accroissements finis ϕ(C ) ⊂ f 0 (I). Inversement, vu la forme de I on a, pour
tout x ∈ I
ϕ : x ∈ I 7→ ϕ(x) = f (x) − λx ∈ R.
ϕ est dérivable sur I et il existe α, β ∈]a, b[ tels que ϕ(α) = ϕ(β) (sinon ϕ est injective
continue sur ]a, b[ est strictement monotone, ϕ est dérivable : ϕ0 est de signe constant ce qui
est absurde puisque ϕ0 (a) = f 0 (a) − λ < 0 et ϕ0 (b) = f 0 (b) − λ > 0...) on conclut alors avec
le théorème de Rolle. q
On montre facilement que f est deux fois dérivable en x = 0 avec f 0 (0) = f 00 (0) = 0. En
outre x = 0 est bien point d’inflexion de f car f est > 0 sur R?+ et < 0 sur R?− alors que
la tangente à l’origine au graphe de f est l’axe des abcisses. Toutefois, après un petit calcul
on a au voisinage de l’origine f 00 (x) = −x sin(x−1 ) + o(x) qui n’est bien sûr pas de signe
constant sur aucun voisinage à droite (et à gauche) de zéro. q
i Remarque : si f est deux fois dérivable au voisinage d’un point a et si sa dérivée seconde
s’y annule en changeant de signe alors f admet un point d’inflexion en a. La réciproque est
donc fausse.
−π π
On se ramène au cas classique en posant si x ∈] , [, g(x) = f (tan(x)) et g(x) = l pour
2 2
π
x = ± . On conclut en appliquant le théorème de Rolle à g. q
2
Il faut donc montrer que ∀ ε > 0, ∃ η > 0 : ∃ζ ∈]a, a + η[ tel que |f 0 (a) − f 0 (ζ)| ≤ ε. f étant
dérivable (à droite)
f (x) − f (a) 0
(8) ∀ ε > 0, ∃ η > 0 : ∀ x ∈]a, a + η[ : − f (a) ≤ ε,
x−a
mais pour un tel x le théorème des accroissements finis assure de l’existence d’un ζ ∈]a, x[
tel que f (x) − f (a) = (x − a)f 0 (ζ). Il ne reste plus qu’à reporter dans (8) pour conclure. q
Soit a < x < c, appliquons le théorème des accroissements finis à f sur [x, c] : il existe
x < ηx < c tel que
f (x) − f (c)
= f 0 (ηx ).
x−c
Mais
(x < ηx < c) =⇒ lim ηx = c
x→c−
f (x) − f (c)
lim = lim f 0 (ηx ) = lim f (x) = l.
x→c− x−c x→c− x→c−
0
f est donc dérivable à gauche au point c avec fg (c) = l, on fait de même à droite. q
Si f n’est pas constante, on peut trouver a ∈ R tel que f 0 (a) 6= 0 et la formule de Taylor-
Lagrange nous donne pour
x2 00
x ∈ R, ∃ ζx ∈ (a, x) : f (x + a) = f (a) + xf 0 (a) + f (ζx ) ≥ f (a) + xf 0 (a)
2
x−a x−b
(F) ∀ x > b > a, f (x) ≥ f (b) − f (a).
b−a b−a
Si f (b) > f (a) il existe δ > 0 tel que f (b) = f (a) + δ et (F) devient pour tout x > b :
x−a x−a a−b
f (x) ≥ f (b) − + f (a)
b−a b−a b−a
x−a
= (f (b) − f (a)) + f (a)
b−a
x−a
≥ δ + f (a) := h(x)
b−a
la fonction h est non majorée sur ]b, +∞[, il en est donc de même pour f d’où la contradiction.
On procède de manière analogue si f (b) < f (a) en établissant pour x < a < b avec δ :=
f (a) − f (b)
b−x
f (x) ≥ δ + f (b)
b−a
ê Ce résultat ne subsiste plus si on remplace R par un intervalle de la forme (a, +∞[
(resp. ] − ∞, a)), il suffit par exemple de considérer f (x) = e−x (resp. f (x) = ex ).
Pour la continuité et la dérivabilité c’est classique. En outre pour tout n ∈ N f (x) = o(xn )
à l’origine : elle admet donc un développement limité à l’ordre n en ce point (la partie
principale étant le polynôme nul...). q
Lemme : Soit f une telle fonction et P ∈ R[X] un polynôme de degré d sans zéros
sur ] − 1, 1[. Alors pour tout n ∈ N on a
Z((f P )(n+d) ) ≥ Z(f (n) ) + d.
Preuve du lemme : Par une récurrence élémentaire, il est suffisant de prouver que pour
tout n ∈ N et tout α réel hors de ] − 1, 1[ on a en posant g(x) = (x − α)f (x) :
Z(g (n+1) ) ≥ Z(f (n) ) + 1.
Or
Nous sommes maintenant en mesure de résoudre l’exercice proposé. Soit p un entier tel que
p
3 f(0) f(0)
< min ,
4 f(1/2) f(−1/2)
et posons g(x) = (1 − x2 )−p f (x) avec g(−1) = g(1) = 0 ; par le lemme de division g est C ∞
sur R et par construction
g(−1) < g(−1/2), g(−1/2) > g(0), g(0) < g(1/2) et g(1/2) > g(1).
Il en résulte que g 0 s’annule au moins trois fois sur ] − 1, 1[ ; le lemme permet alors d’affirmer
que
Z(f (2p+1) ) ≥ 2p + 3.
i Remarques : ê Cette solution nous laisse sur notre fin, en effet on ne comprends
pas plus après sa lecture pourquoi ce mystérieux zéro supplémentaire va apparaitre et ne met
pas vraiment non plus en évidence le role de la compacité du support de f qui est essentielle
2
(la fonction f (x) = e−x vérifie pour tout n ∈ N : limx→±∞ f (n) (x) = 0 mais on a Z(f (n) ) = n
). Il serait vraiment trés intéressant d’avoir une autre preuve expliquant l’apparition de ce
zéro supplémentaire.
ê En observant un peu plus en détail cette solution, il n’est alors pas difficile de montrer
que pour tout entier k ≥ 1 les applications f (n) admettront au moins n+k zéros dans ]−a, a[.
ê Le lemme de division est un outil essentiel pour beaucoup d’exercices d’analyse. Il dit
en substance que (reprendre l’enoncé du Zuily et pourquoi la solution...........)
Nous allons montrer que f est dérivable en x0 . x0 ∈]0, 1[\ (∪n In ) implique que x0 6= an pour
tout n ∈ N, par conséquent f (x0 ) = 0.
Soit x ∈]0, 1[, s’il existe ∈ N tel que x = an alors
f (x) − f (x0 )
= f (an ) ≤ bn = 1
x − x0 |an − x0 | c n bn cn
sinon
f (x) − f (x0 )
x − x0 = 0.
Puisque cn → ∞ ces deux inégalités assurent que pour tout ε > 0 il n’existe qu’un nombre
fini de réels x ∈]0, 1[\{x0 } tels que
f (x) − f (x0 )
x − x0 ≥ ε.
k n−p
Ckp
X 2
≤ n! |λn |k−p sup |ϕ(k−p) (λn x)|
p=0
(n − p)! |λ n | λn x∈[−2,2]
k
|an |Mn X Ckp 2n−p
≤ (8)
|λn |n−k p=0 (n − p)!
!
où Mn := max sup |ϕ(k−p) (λn x)| . D’autre part, avec les majorations grossières
0≤k≤n−1 λn x∈[−2,2]
pour 0 ≤ p ≤ k ≤ n − 1 :
2n−p
Ckp ≤ k! ≤ (n − 1)!, ≤ 2n
(n − p)!
supposons en outre |λn | ≥ 1, ∀ n ∈ N, alors 1/|λn |n−k ≤ 1/|λn |, et finalement (8) devient
k
(k) |an |Mn X |an |Mn n
sup fn (x) ≤
n−k
k!2n ≤ 2 n!.
x∈R |λn | p=0
|λn |
de cette dernière inégalité, nous avons
(4n ) |λn | ≥ max (1, Mn |an |4n n!)
qui implique
sup fn(k) (x) ≤ 2−n ,
(6) ∀ 0 ≤ k ≤ n − 1.
x∈R
Maintenant, comme ϕ est constante égale à 1 sur [−1, 1], ses dérivées d’ordre supérieur à 1
sont identiquement nulles sur ] − 1, 1[ donc en particulier à l’origine. De même
(
n! si n = k
(xn )(k) x=0 =
0 sinon.
De là, pour évaluer f (k) (0), seul un terme n’est pas nul dans l’expression précédente, il reste
précisément :
ak n (k)
f (k) (0) = (x ) ϕ(λn x)x=0 = ak ,
k!
et f possède bien les propriétés désirées. o
Nous avons
n n
!
n
X d X
(−1)k+1 Cnk k n = − lim (−1)k Cnk ekx
k=1
x→0 dxn k=0
n
d
= − lim n
((1 − ex )n )
dx
x→0
n
dn x2 x3
= − lim n −x − − − ...
x→0 dx 2! 3!
dn xn+1
n+1 n
= (−1) lim x +n + ...
x→0 dxn 2
= (−1)n+1 n!
o
INTÉGRATION
R1
ê On procède par récurrence sur n ∈ N. Si n = 0, I0 = 0 ex dx = e − 1 et l’assertion est
donc vraie pour n = 0 avec a0 = −1 = −b0 . Supposons la propriété vrai au rang n avec
In = an + ebn , alors
Z 1 Z 1
x n+1
x n+1 1
In+1 = e x dx = e x 0
− ex (n + 1)xn dx
0 0
= e − (n + 1)In = e − (n + 1)(an + ebn ) = −(n + 1)an + e(1 − (n + 1)bn )
soit la propriété au rang n + 1, C.Q.F.D.
ê Supposons que e = p/q avec p, q ∈ N? , alors
qan + pbn
0 < In = an + ebn = ,
q
qui implique
qan + pbn ≥ 1
et par conséquent
1
(1) In ≥ , ∀ n ∈ N.
q
D’un autre coté, on a la majoration immédiate
Z 1
e
(2) 0 < In ≤ exn dx = , ∀ n ∈ N.
0 n+1
(1) et (2) donnent
1 e
0< ≤ , ∀n ∈ N
q n+1
177
178/408 Petit Bestiaire d’Exercices pour l’Oral de l’Agrégation Interne Patrice Lassère
R∞ 2
Exercice 166 (Calcul de l’intégrale de Gauss 0 e−t dt (1) )
Z ∞ −xt2
e
On considère l’application f (x) := dt.
0 1 + t2
ê Montrer que f ∈ C 1 (R?+ ) ∩ C 0 (R+ ).
ê En déduire que Z f∞ est solution√d’une équation différentielle.
2 π
ê Montrer que e−t dt = (on pourra introduire la fonction auxilliaire
0 2
g(t) = e−t f (t)...).
ê Notons
∞ 2 ∞
e−xt
Z Z
f (x) := dt = g(x, t)dt
0 1 + t2 0
2
e−xt
où g(x, t) = 1+t2
; c’est une fonction C ∞ sur R+ × R+ qui vérifie
1
∀ x ∈ R+ , ∈ L1 (R+ ).
|g(x, t)| ≤
1 + t2
f est donc (par convergence dominée) continue sur R+ . De même pour la dérivabilité, nous
avons pour a > 0
2 −xt2 2 −at2
∂g te t e
∀ x ≥ a > 0 : (x, t) ≤ − ≤ .
∂x 1 + t 1 + t2
2
Toujours par convergence dominée, g est de classe C 1 sur [a, +∞[ et ceci pour tout a > 0 :
f ∈ C 1 (R?+ ). En résumé
Z ∞ 2 −xt2
te
(8) f ∈ C (R+ ) ∩ C (R+ ) et ∀ x ∈ R+ : f (x) = −
0 1 ? ? 0
dt.
0 1 + t2
2
ê Notons I = R+ e−t dt. Avec (8) nous avons pour tout x > 0
R
Z ∞ 2 −xt2 Z ∞
0 te −xt2 I
f (x) = − dt = f (x) − e dt =√ f (x) − √ .
0 1 + t2 0 u=t x x
L’application g(x) = e−x f (x) vérifie
I
g 0 (x) = e−x (f 0 (x) − f (x)) = −e−x √ .
x
Il existe donc une constante C > 0 telle que
x
e−t
Z
∀x ∈ R?+ : g(x) = C − I √ dt.
0 t
En outre
−x π −x
(1) |g(x)| ≤ e |f (x)| ≤ e =⇒ lim g(x) = 0
2 x→∞
et
√
x x
e−t
Z Z
2
(2). g(x) = C − I √ dt =√ C − 2I e−u du −→ C − 2I 2
0 t u= t 0 x→∞
qui implique
lim f (x) = 2I 2
x→0
et par continuité de f à l’origine
Z ∞
dt π
lim f (x) = f (0) = 2
= ,
x→0 0 1+t 2
√
π
soit I = . o
2
R∞ sin(t)
Exercice 167 (Calcul de l’intégrale de Cauchy 0 t
dt (1) )
R∞
On considère l’application f (x) := 0 sin(t)
t
e−xt dt.
ê Montrer que f ∈ C 1 (R?+ ).
ê En déduire une forme explicite de f sur R?+ .
ê Montrer que f est R ∞continue à πl’origine.
ê En déduire que 0 sin(t) t
dt = 2 .
R∞ R ∞ sin(t)
ê Écrivons f (x) = 0 g(x, t)dt où g(x, t) = e−xt sin(t)t
. Pour x = 0, f (0) = 0 t
dt et
1 −xt
nous retrouvons l’intégrale (convergente ) de Cauchy ; pour x > 0, comme |g(x, t)| ≤ e ∈
1
L (R+ ), f est encore bien définie : f est finalement définie sur R+ .
Soit a > 0, nous avons
∂g
|g(x, t)| ≤ e−at ∈ L1 (R+ ) et (x, t) = | − sin(t)e−xt | ≤ e−at ∈ L1 (R+ ).
∂x
De ces deux inégalités, le théorème de continuité et dérivabilité des intégrales à paramètres
assure que f ∈ C 1 (R?+ ) et
Z ∞
0
?
∀ x ∈ R+ , f (x) = − sin(t)e−xt dt.
0
i Remarque : Il faut se garder, malgré les questions suivantes, de vouloir par ces théo-
rèmes de domination obtenir la continuité de f à l’origine : en effet f est à l’origine définie
1Voir l’exercice ? ? ? ?
par l’intégrale de Cauchy qui est notoirement non absolument convergente et une domina-
tion de g dans un voisinage de l’origine impliquerai assurément l’absolue convergence. C’est
pourquoi d’ailleurs les dominations n’ont lieu que sur [a, +∞[...
ê L’expression de f 0 (x) que nous venons d’obtenir nous permet un calcul explicite : soit
x>0
Z ∞
1 ∞ it
Z
0 −xt
e − e−it e−xt dt
f (x) = − sin(t)e dt = −
0 2i 0
1 ∞ t(i−x)
Z
− e−t(i+x) dt
=− e
2i 0
t(i−x) ∞ −t(i+x) ∞
1 e e
=− +
2i i−x 0 i+x 0
1 1 1 1
=− − − =−
2i i−x i+x 1 + x2
−t(i+x) −xt
(les deux termes « entre crochets » sont nuls à l’infini car par exemple | e i+x
|= √e
x2 +1
→0
lorsque t tends vers +∞....). En intégrant cette formule, il vient
La constante C n’est pas difficile à déterminer, en effet la formule ci-dessus implique que
π
lim f (x) = − +C
x→+∞ 2
et pour tout x > 0
Z ∞
1
|f (x)| ≤ e−xt dt = → 0
0 x x→+∞
soit
π π
− +C =0 et C = .
2 2
Résumons nous :
π
−arctan(x) + , si x > 0
(8) f (x) = R ∞ sin(t) 2
0
dt, si x = 0.
t
et la fonction ( u
G e−u si x 6= 0,
H(x, u) = x
0 si x = 0.
est continue sur R+ × R?+ (la continuité en (0, u) découle de limt→∞ G(t) = 0) ; elle est aussi
dominée par
|H(x, u)| ≤ e−u ∈ L1 (R+ ).
Donc par convergence dominée
∞ ∞
Z Z
(4) lim |f (x) − f (0)| = lim − H(x, u)du = − lim H(x, u)du = 0.
x→0+ x→0 0 0 x→0
R +∞ sin(t)
Exercice 168 (Encore un calcul de l’intégrale de Cauchy 0 t
dt (2) )
Montrer que Z +∞
sin(t) π
dt = .
0 t 2
puis
!
Z π/2 Z π/2
sin((2n + 1)x) sin((2n + 1)x)
(8) lim dx − dx = 0,
n→∞ 0 x 0 sin(x)
R +∞ sin(t)
Exercice 169 (Toujours un calcul de l’intégrale de Cauchy 0 t
dt (3) )
Sachant que pour tout a > 0
Z ∞ Z ∞ −at
sin(t) e
dt = 2
dt,
0 t+a 0 t +1
montrer que
Z ∞
sin(t) π
dt = .
0 t 2
e−at
−at
1 e 1
lim 2 = 2 = f (t), ∀ t ∈ R+ , et t2 + 1 ≤ f (t) ∈ L (R+ )
a→0+ t + 1 t +1
donc, par convergence dominée
∞ ∞
e−at
Z Z
1 π
lim = = .
a→0+ 0 t2 + 1 0 t2 +1 2
Pour le terme de gauche, une convergence dominée est sans espoir car il est notoire (cf.
exercice page 193) que
sin(t)
t 7→ 6∈ L1 (R+ ).
t
ê On peut tout de même justifier l’inversion des deux limites « à la main » par exemple
en remarquant que
∞ ∞ Z ∞
Z Z Z 1
sin(t) sin(t) a| sin(t)| a| sin(t)|
dt − dt ≤ dt + dt
0 t+a 0 t 0 t(t + a) 1 t(t + a)
le premier terme du second membre tend vers zéro avec a par convergence dominée car
l’intégrande converge simplement vers la fonction nulle sur [0, 1] avec la domination
a| sin(t)| | sin(t)|
≤ ∈ L1 ([0, 1]).
t(t + a) t
∞ ∞ Z ∞
Z Z Z ε
sin(t) sin(t) a| sin(t)| a| sin(t)|
dt − dt ≤ dt + dt
0 t+a 0 t 0 t(t + a) ε t(t + a)
Z ε Z ∞
sin(t) a
≤ dt + dt
0 t ε t2
a
≤ε+ ∀ a, ε > 0
√ ε √
≤2 a ∀ a > 0 (on a fait ε = a)
d’où le résultat. q
∞ ∞
e−at
Z Z
sin(t)
dt = 2
dt, ∀ a ∈ R?+ .
0 t+a 0 t +1
ê Ces intégrales impropres sont clairement convergentes pour tout x ∈ R+ ; posons pour
(x, t) ∈ R+ × R+ : f (x, t) = sin(xt)/t + x, g(x, t) = e−tx /t2 + 1. Les dominations
1
|g(x, t)| ≤ , ∀ t ∈ R+ ,
1 + t2
−at
∂g(x, t)
∀ t ∈ R+ , ≤ te ∈ L1 (R+ ), ∀ x ≥ a > 0,
∂x 1 + t2
∂ g(x, t) t2 e−at
2
∀ t ∈ R+ , ≤ ∈ L1 (R+ ), ∀ x ≥ a > 0,
∂x2 1 + t2
assurent par convergence dominée que g est continue sur R+ et de classe C 2 sur R?+ avec
Z ∞ −xt Z ∞ 2 −xt
0 te 00 te
g (x) = − 2
dt, g (x) = 2+1
dt, ∀ x ∈ R?+ .
0 t + 1 0 t
On en déduit immédiatement que g 00 (x) + g(x) = 1/x sur R?+ .
Pour f c’est un peu plus délicat car l’application t 7→ f (x, t) est notoirement non absolu-
ment intégrable sur R+ et toute domination est veine, on commence donc par une intégration
par parties pour obtenir une expression plus exploitable de f .
Z ∞ ∞ Z ∞ Z ∞
sin(t) 1 − cos(t) 1 − cos(t) 1 − cos(t)
f (x) = dt = + dt = dt
0 t+x t+x 0 0 (t + x)2 0 (t + x)2
(afin d’alléger les calculs on a choisi 1 − cos(t) comme primitive de sin(t) choix qui annule le
« terme entre crochets »). De là, si h(x, t) = 1 − cos(t)/(t + x)2 et pour x ≥ a > 0
∂h(x, t) 2(1 − cos(t)) 2
∀ t ∈ R+ , = −
3
≤
3
∈ L1 (R+ ), ∀ x ≥ a > 0,
∂x (t + x) (t + a)
2
∂ h(x, t) 6(1 − cos(t)) 12
∀ t ∈ R+ , 2
=
4
≤ ∈ L1 (R+ ), ∀ x ≥ a > 0,
∂x (t + x) (t + a)3
ces dominations impliquent que f ∈ C 2 (R?+ ) avec
Z ∞
00 6(1 − cos(t))
f (x) = dt, ∀ x ∈ R?+
0 (t + x)4
R∞
Exercice 171Z(Calcul de l’intégrale de Cauchy 0 sin(t) t
dt (5) )
∞
sin(xt)
Soit F (x) = dt. Préciser le domaine de définition de F , étudier la
0 t(t2 + 1)
continuite et l’existence des dérivées premières et secondes ; exprimer F (x) en fonc-
R ∞ sin(t)
tion de C := 0 dt et en déduire la valeur de C.
t
L’intégrale définissant F est clairement convergente pour tout x ∈ R : F est définie sur R et
sin(xt)
est impaire. Posons f (x, t) = t(t2 +1) .
|2t| 2t2
≤ 2 2 + ∈ L1 (R)
a (t + 1)2 a(t2 + 1)2
on peut donc dériver sous l’intégrale : F est deux fois dérivable sur R? et
Z ∞
2t2 cos(xt)
00 2t sin(xt)
F (x) = − 2· 2 2
+ 2 + 1)2
dt, ∀ x ∈ R? .
0 x (t + 1) x(t
Cette expression est un peu chargée, faisons une intégration par parties :
Z ∞
sin(xt) 2t2 cos(xt)
00 2t
F (x) = − 2 · + dt
0 (t + 1)2 x2 x(t2 + 1)2
∞ Z ∞
sin(xt) t cos(xt) x cos(xt) cos(xt) − xt sin(xt)
= 2 2 − − − dt
x (t + 1) x(t2 + 1) 0 0 x2 (t2 + 1) x(t2 + 1)
Z ∞
t sin(xt)
=− dt.
0 t2 + 1
Il est intéressant à ce stade d’observer que nous retrouvons finalement la formule
Z ∞ 2
00 ∂
F (x) = 2
f (x, t)dt, x ∈ R? ,
0 ∂x
mais pour justifier une dérivation sous l’intégrale une transformation de F 0 (voir (8)) à été
nécessaire ; remarquez aussi que l’existence de F 00 (0) reste ouverte. Nous avons donc :
Z ∞
0 cos(xt)
F (x) = dt, ∀ x ∈ R,
0 t2 + 1
Z ∞
00 t sin(xt)
F (x) = − 2
dt, ∀ x ∈ R? .
0 t +1
soit a + b = −C ; de même, F 0 continue à l’origine avec F 0 (0) = π/2 donne a − b = π/2 i.e.
2a = π/2 − C, 2b = −C − π/2 et finalement
π
(4) F (x) = sh(x) − Cch(x) + C, ∀ x ∈ R?+ .
2
ê Il reste à évaluer C. Pour cela, montrons que lim F (x) = C. Soit x > 0,
x→+∞
∞
2t2 cos(xt)
Z
00 2t sin(xt)
F (x) − C = F (x) = − 2· 2 + dt
0 x (t + 1)2 x(t2 + 1)2
(on a encore ici besoin de la première expression de F 00 pour conclure facilement) pour
x ≥ a > 0, on a la domination
2
2t2
2t sin(xt) 2t cos(xt) 2t
− ·
x2 (t2 + 1)2 + ≤ + ∈ L1 (R+ ).
x(t2 + 1)2 a2 (t2 + 1)2 a(t2 + 1)2
∞
2t2 cos(xt)
Z
2t sin(xt)
lim (F (x) − C) = lim − 2· 2 + dt = 0
x→+∞ 0 x→+∞ x (t + 1)2 x(t2 + 1)2
π
lim F (x) = C et F (x) ∼ − C ex + C
x→+∞ +∞ 2
qui donnent
Z ∞
sin(t) π
C= dt = .
0 t 2
C.Q.F.D. o
R∞
Exercice 172 (Calcul de l’intégrale de Cauchy 0 sin(t) t
dt (5), )
Montrer que pour tout x ∈ R
Z π/2 Z x
−x cos(t) π sin(t)
e cos(x sin(t))dt = − dt.
0 2 0 t
Z ∞
sin(t) π
En déduire que dt = .
0 t 2
ê Soit x ∈ R, on a
Z π/2 !
Z π/2
−x cos(t)
e cos(x sin(t))dt = Re e−x cos(t) eix sin(t) dt
0 0
!
Z π/2
−xe−it
= Re e dt
0
∞
!
π/2
(−x)n e−int
Z X
= Re dt
0 n=0
n!
∞ π/2
(−x)n
Z
π X
Re e−int dt
= +
2 n=1 n! 0
∞ π/2
(−x)n sin(nt)
π X
= +
2 n=1 n! n 0
∞
π (−x)n sin(nπ/2)
X
= +
2 n=1 n! n
∞
π X (−x)2k+1 sin((2k + 1)π/2)
= +
2 k=0 (2k + 1)! 2k + 1
∞
π X (−1)k x2k+1
= −
2 k=0 (2k + 1)(2k + 1)!
∞ Z
π X x (−1)k t2k+1 dt
= −
2 k=0 0 (2k + 1)! t
Z xX ∞
π (−1)k t2k+1 dt
= −
2 0 k=0 (2k + 1)! t
Z x
π sin(t)
= − dt.
2 0 t
RP PR
Les deux échanges = sont justifiés par la normale convergence des deux séries
entières sur le domaine d’intégration (leur rayon de convergence étant infini).
ê Une convergence dominée élémentaire2 (e−x cos(t) cos(x sin(t)) ≤ 1 ∈ L1 ([0, π/2])) im-
plique
Z π/2
lim e−x cos(t) cos(x sin(t))dt = 0,
x→+∞ 0
soit, vu la formule établie au dessus
Z x
π sin(t)
lim − dt = 0
x→+∞ 2 0 t
n2
Exercice 173 (Étude de la suite (un = n2 − nk=1
P
)
(n+k)2 n
) [10].
Convergence et limite de la suite de terme général
n
n X n2
un = − .
2 k=1 (n + k)2
Ecrivons
n
! n
!
Z 1
1 1X 1 1X k
un = n − =n f (t)dt − f( )
2 n k=1 1 + k 2 n k=1 n
0
n
Rx
où f (t) = (1 + t)−2 . En posant pour 0 < x < 1, F (x) = 0
f (t)dt, on peut écrire cette
dernière expression sous la forme :
n
! n−1 Z k+1/n n
!
Z 1
1X k X 1X 0 k
un = n f (t)dt − f( ) = n f (t)dt − F ( )dt
0 n k=1 n k=0 k/n n k=1
n
n−1 Z k+1/n n−1
!
X 1X 0 k
= f (0) − f (1) + n f (t)dt − F( )
k=0 k/n
n k=0 n
n−1
!
X k+1 k 1 0 k
= f (0) − f (1) + n F( ) − F( ) − F ( )
k=0
n n n n
k+1 k 1 k 1 k k+1
F( ) − F ( ) − F 0 ( ) = 2 F 00 (ζn,k ) où ζn,k ∈] , [
n n n n 2n n n
et finalement
n−1 Z 1
1 1 X 00 1 3
lim un = f (0) − f (1) + lim F (ζn,k ) = f (0) − f (1) + F ”(t)dt =
n→+∞ 2 n→+∞ n k=0 2 0 8
la dernière limite étant justifiée puisque l’on y reconnaît la somme de Riemann de la fonction
continue F 00 associée à la subdivision { nk }nk=0 en les points ζk,n ∈] nk , k+1
n
[ k
. q
Ë Pour tout n ∈ N : √
t e = tn exp(− 2 ) ∈ L1 (R)
t
n −ωt
2
et l’intégrale In est bien convergente ; en outre, si n ≥ 1 une intégration par parties donne
n −ωt ∞ n ∞ n−1 −ωt
Z
−1 n
In = ω −t e 0
+ t e dt = In−1
ω 0 ω
soit
n!
In =, n ∈ N.
ω n+1
Et puisque pour n ≥ 1 : ω 4(n+1) = (−1)n+1 au vu du calcul précédent
I4n+3 ∈ R, ∀ n ∈ N
sa partie imaginaire est donc toujours nulle i.e.
Z ∞
√ √
4n+3 t 2 t 2
0 = im(I4n+3 ) = t exp(− ) sin( )dt
0 2 2
Z ∞
= xn sin(x1/4 ) exp(−x1/4 )dx (poser x = t4 /4)
Z0 ∞
= xn f (x)dx n ∈ N
0
d’où le contrexemple désiré.
D’autre part
Z a Z a Z a
2n+1 2n+1
(f (t) + f (−t)) t dt = f (t)t dt − f (−t)(−t)2n+1 dt = 0, ∀ n ∈ N.
−a −a −a
L’application continue sur [−a, a], g(t) = f (t) − f (−t) vérifie donc
Z a
f (t)tn dt = 0, ∀ n ∈ N,
−a
compacité de ce dernier intervalle, il existe λ > 0 tel que P 0 (t) ≥ λ > 0. Ainsi, en posant
M = sup[0,1] |f (t)|, on a pour tout k ∈ N :
Z a Z a
1 a 0
Z
n
n
P (t)f (t)dt ≤ M
P (t)dt ≤ P (t)P n (t)dt
0 0 λ 0
n+1 a
1 P (t) 1 − (1 − ab)n+1 1
= = ≤
λ n+1 0 λ(n + 1) λ(n + 1)
et en procédant de même sur [b, 1] on peut donc affirmer que
Z a Z 1
n
(8) lim P (t)f (t)dt = 0, & lim P n (t)f (t)dt = 0.
n→∞ 0 n→∞ b
R +∞ R +∞
Exercice 175 (Étude de Iα = 0 sin(t) tα
dt, Jα = 2 tαsin(t) +sin(t)
dt & Sα =
P (−1)n
n≥1 nα +(−1)n , α ∈ R)
Pour 2 > α > 0 étudier la convergence et l’absolue convergence des intégrales
impropres ou séries
Z +∞ Z +∞ X (−1)n
sin(t) sin(t)
Iα = dt & J α = dt & S α =
0 tα 2 tα + sin(t) n≥1
nα + (−1)n
Ê Notons f (t) = sin(t)t−α . Iα est une intégrale impropre à l’origine et à l’infini. À l’origine,
f est équivalente à t−α+1 fonction positive, de type Riemann, intégrable en t = 0 puisque
−α+1 < −1. En outre |f (t)| ≤ t−α fonction intégrable à l’infini pour α > 1 : la convergence
absolue de Iα est donc établie pour 2 > α > 1.
Reste donc le cas 0 < α ≤ 1 ; vu ce qui précède (tout se passant bien à l’origine) il sera
suffisant d’étudier la convergence sur [1, +∞[. Pour cela on fait une intégration par parties
(avec la convention habituelle qu’elle sera réellement correcte si deux termes parmi les trois
existent, sinon, travailler sur [1, A] puis faire tendre A vers +∞...)
Z +∞ +∞ Z +∞
sin(t) cos(t) α cos(t)
α
dt = α
+ dt
1 t t 1 1 tα+1
et +∞
cos(t) ≤ α ∈ L1 ([1, +∞[)
α cos(t)
= − cos(1),
tα 1 tα+1 tα+1
assurent alors la convergence de Iα pour 0 < α ≤ 1.
Pour la convergence absolue en +∞ si 0 < α ≤ 1, le plus simple est de remarquer que
Z (2n+1)π
n Z 2kπ+ 3π n Z 2kπ+ 3π √ n √
sin(t) X 4 sin(t) X 4 2 X π 2
dt ≥ dt ≥ α dt =
0
tα π
k=0 2kπ+ 4
tα
k=0 2kπ+ π
4
2 ((2k + 1)π) k=0
4 ((2k + 1)π)α
cette dernière quantité tendant vers l’infini avec n comme somme partielle d’une série de
terme général équivalent à Ck −α donc divergente...
Ë Le problème est toujours à l’infini où notre fonction n’est pas de signe constant, donc
pas question d’utiliser les équivalents, le dernier recours (le développement asymptotique)
montre ici toute sa puissance
n n
nx n!
Z
t
À Montrer que 1− tx−1 dt = , x > 0.
0 n x(x + 1) . . . (x + n)
Á Montrer que n
Z n
t
Γ(x) = lim 1− tx−1 dt, x > 0.
n→∞ 0 n
 En déduire que
nx n!
Γ(x) = lim , x > 0.
n→∞ x(x + 1) . . . (x + n)
i On suppose ici acquis (classique mais excellent exercice sur les intégrales à paramètres)
le fait que Γ ∈ C ∞ (R?+ ) et que l’on peut dériver sous l’intégrale.
Á Si f est deux fois dérivable sur [0, 1] et si f 00 est bornée et intégrable sur [0, 1],
montrer que
Z 1 n !
f 0 (1) − f 0 (0)
1 X 2k − 1
lim n2 f (t)dt − f = .
n→∞ 0 n k=1 n 24
Ë Pour la seconde limite, la technique est la même mais il faut bien entendu pousser plus
loin le développement : avec Taylor-Lagrange, nous avons
2
2i − 1 0 2i − 1 2i − 1 1 00 2i − 1
f (x) − f =f x− + f (ζi (x)) x − .
2n 2n 2n 2 2n
Et par conséquent
Z 1 n ! n Z i
2 1 X 2k − 1 2
X n 2i − 1
n f (t)dt − f =n f (t) − f dt
0 n k=1 n i=1
i−1
n
2n
n Z i
2
X n
0 2i − 1 2i − 1
=n f x− dx
i=1
i−1
n
2n 2n
n Z i 2
n2 X n 00
2i − 1
+ f (ζi (x)) x − dx.
2 i=1 i−1 n
2n
i
2i − 1 2i − 1
Z
n
0
f x− dx
i−1
n
2n 2n
n n Z i 2 n
n2 X n 00
1 X 2i − 1 1 X
mi ≤ f (ζi (x)) x − dx ≤ Mi
24n i=1 2 i=1 i−1
n
2n 24n i=1
n n−1
X X λj
λj xj = λn xn + (1 − λn ) xj .
j=1 j=1
1 − λn
Posons
X X λj X λj
λ= λj , x= xj , y= xj
j∈S j∈S
λ j6∈S
1 − λ
et
X λj X λj
x= xj < xn0 = xn0 = y
j∈S
λ j∈S
λ
x est donc different de y : par stricte convexité de f nous avons
f (λ1 x1 + · · · + λn xn ) = f (λx + (1 − λ)y) < λf (x) + (1 − λ)f (y).
Maintenant, avec l’inégalité de Jensen
! !
X λj X λj
λf (x) + (1 − λ)f (y) = λf xj + (1 − λ)f xj
j∈S
λ j6∈S
1 − λ
X λj X λj
≤p f (xj ) + (1 − λ) f (xj )
j∈S
λ j6∈S
1−λ
n
X
= λj f (xj )
j=1
1/p R
b p
Exercice 179 (Limite en 0, ±∞ de |f (t)| dt ) [27] a
Montrer que si f est continue sur [a, b], alors
Z b 1/p
p
lim |f (t)| dt = max |f (x)|.
p→+∞ a x∈[a,b]
si de plus f est sans zéros dans [a, b] déterminer les limites suivantes
Z b 1/p Z b 1/p
p p
lim |f (t)| dt & lim |f (t)| dt .
p→0 a p→−∞ a
Z b 1/p Z b 1/p
p p
(F) |f (t)| dt ≤ M dt = M (b − a)1/p ,
a a
De l’autre côté, par continuité de f et étant donné ε > 0, il existe [α, β] ⊂ [a, b] tel que
|f (x)| ≥ M − 2ε sur [α, β]. Alors
Z b 1/p Z β 1/p
p p ε
|f (t)| dt ≥ |f (t)| dt ≥ (M − )(β − α)1/p
a α 2
Ainsi, vu (F), on a pour tout ε > 0 :
Z b 1/p
ε 1/p p
(M − )(β − α) ≤ |f (t)| dt ≤ M (b − a)1/p
2 a
d’où le résultat en faisant tendre p vers +∞ (ε > 0 étant arbitraire).
Z b 1/p
p
lim |f (t)| dt = min |f (x)|.
p→−∞ a x∈[a,b]
R∞
Exercice 180 (Étude de la suite ( 0 n log (1 + n−α f α (t)) dt)n ) ([10], 1999/00).
1 + tα ≤ (1 + t)α ,
ainsi
(
0 si α > 1
lim In = R ∞
n→∞
0
f (t)dt si α = 1.
q
a2nk − b2nk
Z b Z b
b−a ank bnk
= + cos(2nk x)dx + 2 sin(2nk x)dx
2 2(a2nk + b2nk ) a ank + b2nk a
b−a a2nk − b2nk an b n
:= + 2 2
Ik + 2 k k2 Jk
2 2(ank + bnk ) ank + bnk
mais bien entendu
lim Ik = lim Jk = 0
k→∞ k→∞
et 2
ank − b2nk 1
ank bnk 1
2(a2 + b2 ) ≤ 2
et ≤
a2 + b 2 2
nk nk nk nk
donc 2
ank − b2nk
ank bnk 1
lim 2
Ik
+ J k
≤ (|Ik | + |Jk |) = 0
k→∞ 2(an + b2 nk ) a2 + b 2
nk nk
2
k
soit
b
b−a
Z
lim fnk (x)dx =
>0
k→∞ a 2
contredisant (F) : l’affaire est donc entendue (on procède de même si c’est la suite (bn )n ne
converge pas vers zéro). q
R x2 dt
Exercice 182 (Étude de x 7→ x log(t)
)
On définit F : ]1, +∞[→ R par
Z x2
dt
F (x) = .
x log(t)
Montrer que F est injective et préciser son image F (]1, +∞[).
soit
Z 2 log(x) Z 2 log(x)
log(x) du du
x log(2) = e < F (x) < e2 log(x) = x2 log(2)
log(x) u log(x) u
qui assure F (1− ) = log(2) : ainsi F (]1, +∞[) =] log(2), +∞[. o
Il est « clair » que l(x) = f (x) pour au moins une valeur de x (sinon, ou pourrait déplacer le
graphe de l verticalement ce qui réduirait la valeur de l’intégrale). Le graphe de l est donc
tangent à celui de f et il existe c ∈ [a, b] tel que
équivaut à maximiser
Z b Z b
l(t)dt = (f 0 (c)t + f (c) − cf (c))dt a≤c≤b
a a
0
f (c) 2
= (b − a2 ) + (b − a)(f (c) − cf 0 (c))
2 0
f (c) 0
= (b − a) (a + b) + f (c) − cf (c) .
2
Il s’agit donc de maximiser sur l’intervalle [a, b] l’application
f 0 (c)
g(c) = (a + b) + f (c) − cf 0 (c).
2
f 00 (c)
Or, g 0 (c) = (a + b − 2c), mais f 00 > 0 : g 0 est donc du signe de c 7→ a + b − 2c. Ainsi
2
g est croissante sur [a, (a + b)/2] et décroissante sur [(a + b)/2, b] son maximum est donc
atteint au point médian (a + b)/2. L’inf de notre problème est donc réalisé par la tangente
à la courbe au point (a + b)/2 :
a+b 0 a+b a+b
l(x) = f +f x− .
2 2 2
q
où l’intégration par partie dans l’intégrale généralisée est légitime car deux termes parmi
les trois existent, le terme « en crochets » étant nul à l’origine vu (8). Maintenant de cette
inégalité et toujours par Cauchy-Schwarz
Z A Z x 2 Z A R x
1 0
f (t)dt
f (t)dt dx ≤ 2 f (x) × dx
0 x2 0 0 x
Z A 1/2 Z A Z x 2 !1/2
1
≤2 f 2 (t)dt f (t)dt dx
0 0 x2 0
Z A Z A Z x 2 Z A
2 1
(4) g (t)dt = f (t)dt dx ≤ 4 f 2 (t)dt.
0 0 x2 0 0
• Montrons que la constante 4 dans (4) est optimale. Pour cela, si elle est vérifiée pour un
C < 4 on l’applique à f (t) = t−α , −1/2 < α < 1/2
A x 2 A
A1−2α A1−2α
Z Z Z
dx dt dt
dx = ≤C = C ·
0 x2 0 tα (1 − 2α)(1 − α)2 0 tα 1 − 2α
qui donne
1
≤ C,
(1 − α)2
et en faisant tendre α vers 1/2 on obtient C ≥ 4 : 4 est bien optimale dans (4).
Z A Z A Z ∞
2 2
0≤ g (t)dt ≤ 4 f (t)dt ≤ 4 f 2 (t)dt < ∞,
0 0 0
Z ∞ Z ∞
2
g (t)dt ≤ 4 f 2 (t)dt.
0 0
La constante 4 est bien entendu encore optimale (reprendre le raisonnement précedent avec
des fonctions continues sur R?+ nulles sur [2, +∞[ et égales à t−α sur ]0, 1] avec |α| < 1/2).o
Ë Sachant que3
1 X 2z
πcotan(πz) = + , ∀ z ∈ C \ Z,
z n≥1 n2 − z 2
montrer que Z +∞
π πz
f (t)dt = tan( ), ∀ z ∈ C, |re(z)| < 1.
0 2 2
Ì En développant f en série entière (pour la variable z) montrer que
Z +∞
π sh(zt) X
1 − 2−2n−2 ζ(2n + 2)z 2n+1 ,
tan(πz/2) = dt = 2 ∀ |z| < 1.
2 0 sh(t) ‘≥0
Ê Comme limt→0+ f (t) = z = f (0), f est continue à l’origine et donc sur R+ . En outre
|σ| < 1 implique
e − e−zt |σt|
zt
|f (t)| =
≤ e ∼ 2e(|σ|−1)t ∈ L1 (R+ )
2sh(t) |sh(t)| +∞
ainsi f ∈ L1 (R+ ).
Pour t > 0, on peut écrire
2e−t −t
X
−2nt
X
(z−2n−1)t −(z+2n+1)t
X
f (t) = sh(zt) = 2e sh(zt) e = e − e := un (t),
1 − e−2t n≥0 n≥0 n≥0
et les fonctions un sont intégrables sur R+ car (toujours car |σ| < 1)
(bien remarquer que |σ| < 1 assure que (2n + 1)2 − z 2 6= 0, ∀ n ∈ N). Ainsi, formellement
Z +∞ Z +∞ X X Z +∞ X 2z
f (t)dt = un (t)dt = un (t)dt =
0 0 n≥0 n≥0 0 n≥0
(2n + 1)2 − z 2
RP PR
il ne reste donc plus qu’à justifier l’échange = ci-dessus. Pour cela, il faut être
délicat car la condition suffisante « la série n R+ |un (t)|dt converge » n’est pas ici réalisée4
P R
Ë Il suffit d’un peu de patience et d’attention. Avec la question précédente nous avons
pour |re(z)| < 1 et z 6= 0 (si z = 0 il n’y a rien à démontrer)
Z +∞ X 2z X 2z X 2z
f (t)dt = 2 2
= 2 2
−
0 n≥0
(2n + 1) − z n≥0
n −z n≥0
(2n)2 − z 2
2 X 2z 2 X 2z
=− + 2 2
+ −
z n≥1 n − z z n≥1 (2n)2 − z 2
1 X 2z 1 X z/2
=− − + +
z n≥1 z 2 − n2 z n≥1 (z/2)2 − n2
!
1 2 X z
= −πcotan(πz) + +
2 z n≥1 (z/2)2 − n2
π
= −πcotan(πz) + cotan(πz/2)
2
2
π cos (πz/2) − cos(πz)
= ·
2 cos(πz/2) sin(πz/2)
π sin2 (πz/2) π
= · = tan(πz/2).
2 cos(πz/2) sin(πz/2) 2
Ì Pour t ≥ 0 on a
sh(zt) X t2n+1 z 2n+1 X
f (t) = = := hn (t).
sh(t) n≥0
sh(t)(2n + 1)! n≥0
Soit Z ∞
|hn (t)|dt ≤ 2|z|2n+1 ζ(2)
0
R P
qui est pour |z| < 1 le terme général d’une série convergente, ceci justifie l’échange R+ n hn =
P R
n R+ hn , et finalement
Z ∞
sh(zt) X
2 1 − 2−(2n+2) ζ(2n + 2) · z 2n+1 , ∀ |z| < 1.
dt =
0 sh(t) n≥0
π
c’est le développement en série entière de la fonction D(0, 1) 3 z 7→ tan(πz/2). o
2
i Commentaire : Il existe bien sûr des méthodes plus élémentaires pour déterminer
le développement en série entière de la fonction
Z ∞ tangente à l’origine, c’est ici juste une
sh(zt) π
conséquence de la formule de Ramanujan dt = tan(πz/2)
0 sh(t) 2
Il est considéré comme acquis5 que la fontion Γ est de classe C ∞ sur R?+ avec
Z ∞
(k)
Γ (x) = logk (t)e−t tx−1 dt, ∀ k ∈ N.
0
n
t
n log(t) 1 − t pour t ∈ [0, n],
fn (x) = log(t) 1 − 1[0,n] (t) = n
n
0 si x > n.
La suite (fn )n est simplement convergente sur R+ vers t 7→ log(t)e−t et l’inégalité classique
(1 − t)a ≤ e−at , a > 0, 0 ≤ t ≤ 1, assure la domination
Z n n Z Z
t
lim 1− log(t)dt = lim fn (t)dt = log(t)e−t dt = Γ0 (1).
n 0 n n R+ R+
5Mais attention, vérifier les hypothèses de dérivation sous l’intégrale n’est pas compétement immédiat et doit être parfai-
tement maitrisé... !
où
N N
X n n X 1 1
JN = = − .
k=0
k(n + k + 1) n + 1 k=0 k n+k+1
Pour N > n on peut écrire
n 1 1 1 1 1 1 1 1 1
JN = + ··· + + + + ··· + − − ··· − − − ··· −
n+1 1 n−1 n n+1 N n+1 N N +1 N +n+1
n 1
1 + · · · + − 1 1
= + · · · +
n+1 n N +1 N +n+1
| {z }
n+1 termes
R √ R 1 R 1
Exercice 187 ( R |f (t)|dt ≤ 8 R |tf (t)|2 dt 4 R |f (t)|2 dt 4 . ) [43]
Soit f ∈ L1 (R), montrer que
41 Z 41
√ Z
Z
|f (t)|dt ≤ 8 |tf (t)|2 dt |f (t)|2 dt .
R R R
i Il n’y a bien sûr pas de raisons que f ∈ L1 (R) assure la convergence des deux intégrales
dans le terme de droite de l’inégalité. Dans
R le cas où au moins l’une de ces deux intégrales
diverge on convient que l’inégalité s’écrit R |f (t)|dt ≤ +∞ et il n’y a rien à démontrer. On
peut donc dorénavant, supposer que ces trois intégrales convergent.
Soit x ∈ R?+ , pour faire apparaitre un terme du type tf (t), il convient d’isoler l’origine :
Z Z Z
1
|f (t)|dt = |f (t)|dt + |tf (t)|dt
R [−x,x] R\[−x,x] |t|
sZ r sZ
√ 2
≤ 2x |f (t)|2 dt + |tf (t)|2 dt
[−x,x] x R\[−x,x]
√
r
2B
≤ 2Ax + := ϕ(x), ∀ x ∈ R?+
x
R R
avec A = R |f (t)|2 dt, B = R |tf (t)|2 dt, et où on a appliqué l’inégalité de Cauchy-Schwarz
dans les deux intégrales pour obtenir la seconde inégalité. Ainsi
Z
|f (t)|dt ≤ inf? ϕ(x)
R x∈R+
et déterminer cet infimum ne pose aucun problème puisque ϕ tendsp vers +∞ p lorsque x
tends vers 0 et √
+∞ et que sa dérivée ne s’annule qu’au point x = B/A soit ϕ( B/A) =
inf x∈R?+ ϕ(x) = 8A1/4 B 1/4 , soit l’inégalité demandée. o
Vu les hypothèses
Z b
(f (t) − m)(M − f (t))dt ≥ 0.
a
Soit
Z b Z b Z b
2
(M + m) f (t)dt − f (t)dt − mM (b − a) = − f 2 (t)dt − mM (b − a) ≥ 0,
a a a
i.e. Z b
f 2 (t)dt ≤ −mM (b − a).
a
Q.E.D. o
puis
1 1
f 0 (x) = < , ∀ x > 1.
x2 + f 2 (x) 1 + x2
Ainsi, pour x > 1
Z x
f (x) = f (1) + f (t)dt
1
Z x Z ∞
dt dt π
<1+ <1+ =1+ .
1 1 + t2 1 1+t 2 4
Ê
R x f étant continue sur [0, 1] à valeurs strictement positives Rl’application F : [0, 1] 3 x 7→
1
0
f (t)dt est continue, strictement croissante de [0, 1] sur [0, 0 f (t)dt] : par le théorème des
valeurs intermédiaires, pour tout n ∈ N, il existe an,0 , . . . , an,n dans [0, 1] tels que
Z 1
k
F (an,k ) = f (t)dt, k = 0, . . . , n.
n 0
n Z 1 Z R 1 f (u)du
an,0 + · · · + an,n 1 X −1 k 0
lim = lim F f (t)dt = F −1 (t)dt.
n→∞ n+1 n→∞ n + 1
k=0
n 0 0
0
soit Z 1
p+1 p
a 3 ≤ f (x)dx, ∀ p > 1.
0
Comme p > 1 est arbitraire, le résultat suit en faisant tendre p vers 1. o
R +∞ sin(t)
Exercice 194 (Calcul de l’intégrale de Cauchy 0 t
dt (8) )
−xy
Z +∞intégrant f (x, t) = e sin(x) sur [, T ] × [0, +∞[, 0 < < T, calculer
En
sin(t)
dt.
0 t
6Si f ∈ Lp (I), g ∈ Lq (I) alors f g ∈ L1 (I) et R |f (x)g(x)|dx ≤ (R |f (x)|p dx)1/p (R |f (x)|q dx)1/q où p > 1, q > 1
I I I
conjugués, et donner une référence....
l’application ci-dessus du théorème de Fubini est justifiée par |f (x, y)| ≤ e−xy et
Z T Z ∞ Z T xy ∞ Z T
−xy e dx T
e dy dx = − dx = = log < ∞.
ε 0 ε x 0 ε x ε
pour tous 0 < ε < T .
Maintenant, observons que pour 0 < ε ≤ y
|e−yε (cos ε + y sin ε)| ≤ 1 + yεe−yε ≤ 1 + e−1 ,
de même, pour T ≥ 1
|e−yT (cos T + y sin T )| ≤ e−yT (1 + y) ≤ e−y (1 + y).
Ainsi pour 0 < ε ≤ y ≤ T et T ≤ 1
max{(1 + e−1 , e−y (1 + y)}
|gε,T (y)| ≤ ∈ L1 (R+ ).
y2 + 1
Il est donc légitime d’invoquer le théorème de la convergence dominée pour écrire
Z ∞ Z ∞
dy π
lim lim gε,T (y)dy = 2
= ,
ε→0+ T →+∞ 0 0 y +1 2
d’autre part, comme Z T Z ∞
sin(x)
dx = gε,T (y)dy
ε x 0
nous avons finalement
Z ∞ Z ∞
sin(x) π
dx = lim lim gε,T (y)dy = .
0 x ε→0+ T →+∞ 0 2
o
SUITES ET SÉRIES
Considérons une suite (an )n de nombres complexes vérifiant la propriété (4). Posons pour
tout entier k
Sk = {n ∈ N : k < |an | ≤ k + 1}.
Les disques fermés de centre an et de rayon 1/2 sont par hypothèse deux à deux disjoints et
pour tout n ∈ Sk
1 3
D(an , 1/2) ⊂ {z ∈ C : k − ≤ |z| ≤ k + } = C(0, k − 1/2, k + 3/2).
2 2
(si k = 0, interpréter le second terme comme le disque D(0, 3/2)) soit, en sommant les aires
" 2 2 #
π 3 1
card(Sk ) ≤ π k+ − k− = 2π(2k + 1)
4 2 2
si k ∈ N? et
π 9
card(S0 ) ≤π
4 4
si k = 0. Ainsi
card(S0 ) ≤ 9 et card(Sk ) ≤ 8(2k + 1), ∀ k ∈ N? .
Par conséquent, pour tout k ∈ N?
X 1 card(Sk ) 8(2k + 1) 24
3
≤ 3
≤ 3
≤ 2
n∈Sk
|an | k k k
219
220/408 Petit Bestiaire d’Exercices pour l’Oral de l’Agrégation Interne Patrice Lassère
où la sommation par paquets dans le second terme est légitime puisque la série est à termes
positifs et (Sk )k≥0 une partition de N. q
Exercice 196 (Convergence d’une série par sommation par paquets ) [6]
α(n), (n ∈ N) désignant le nombres de 0 dans l’écriture décimale de n en base 3.
Montrer que la série entière
∞
X xα(n)
n=0
n3
converge si, et seulement si |x| < 25.
L’entier n ∈ N? , admet exactement k + 1 chiffres dans son écriture en base 3 si, et seulement
si 3k ≤ n < 3k+1 . Ainsi, si pour x > 0 on pose
3k+1
X−1 3k+1
X−1
xα(n)
Sk = et Tk = xα(n) ,
n3
n=3k n=3k
et
∞ ∞ k
X Tk X x+2
3k
=2
k=0
3 k=0
27
série convergente si, et seulement si |(x + 2)/27| < 1 i.e. (pour x > 0) si et seulement si
x < 25. o
p∈P
p
diverge.
1
P
Notons p1 , p2 , . . . la suite croissante des nombres premiers. Si la série p∈P p converge, il
existe un entier k tel que
X 1 1
<
i≥k+1
pi 2
et par suite
X N N
(8) ∀ N ∈ N, < .
i≥k+1
pi 2
Nous dirons que p1 , p2 , . . . , pk sont les petits nombres premiers, les autres pk+1 , pk+2 , . . . seront
les grands nombres premiers. Pour N ∈ N, Nb sera le nombre d’entiers n ≤ N admettant au
moins un grand nombre premier comme diviseur et Ns lui désignera le nombre d’entiers n ≤
N admettant uniquement des petits nombres premiers comme diviseur. Nous allons montrer
que l’hypothèse de convergence de la série assure, pour un entier N convenablement choisi,
l’inégalité Nb + Ns < N d’où la contradiction désirée puisque bien entendu N = Nb + Ns .
N
Pour estimer Nb , remarquons que E est le nombre d’entiers n ≤ N multiples de p1 , la
p1
formule (8) donne
X N N
(4) Nb ≤ E < .
i≥k+1
p i 2
Considérons maintenant un entier n ≤ N n’admettant que des petits diviseurs premiers, et
écrivons le sous la forme n = an b2n où an est la partie « non carrée ». Chaque an se décompose
k
donc en un produit √ de différents
√ petits nombres√ premiers : il y a donc 2 choix possibles pour
an . Comme bn ≤ n ≤ N , il reste au plus N choix possibles pour bn soit
√
(F) Ns ≤ 2k N .
√ √
On choisit enfin N suffisamment grand pour que 2k N ≤ N/2 (i.e. 2k+1 ≤ N ), par exemple
N = 22k+2 ; la formule (4) étant vraie pour tout N elle donne avec (F) : Nb + Ns < N d’où
la contradiction. o
i Remarque : La divergence de la série p∈P p1 est établie pour la première fois par
P
L.Euler en 1748. La preuve ci-dessus est celle de P.Erdös (1938). Un corollaire immédiat est
que l’ensemble des nombres premiers est infini.
ê Pour s’assurer que j x2j ⊂]0, A2 [ on peut remarquer (la série est bien entendu conver-
P
m
!2 m
X X X
xj = x2j + 2 xj xk
j=0 j=0 0≤j<k≤m
m m
!2
X X X
x2j ≤ xj −2 xj xk ≤ A2 − 2x0 x1 , ∀m ∈ N
j=0 j=0 0≤j<k≤m
X
x2j ≤ A2 − 2x0 x< A2 .
j≥0
ê Il reste maintenant à s’assurer que toutes les valeurs de ]0, A2 [ vont être atteintes : à cet
effet, on considère la suite géométrique (xj )j de raison d variable (xj+1 /xj = d). Alors
m
!2
X x0 X x20 1−d X 1−d 2
xj = et x2j = 2
= xj = A.
j≥0
1−d j≥0
1−d 1+d j=0
1+d
On reprend in-extenso des extraits du remarquable ouvrage de P. Eymard & J.P. Lafon,
[14].
Ê Elle est archi-classique et se trouve dans votre livre de chevet. Des intégrales de Wallis :
1.3.5 . . . (2n − 1) π
si m = 2n,
Z π/2
2.4.6 . . . (2n) 2
Im = sinm (t)dt =
0
2.4.6 . . . (2n)
si m = 2n + 1
3.5.7 . . . (2n + 1)
on tire
π I2n
(4) = pn
2 I2n+1
avec
n
22 .42 .62 . . . (2n)2 Y 4k 2
pn = = .
(1.3)(3.5) . . . (2n − 1)(2n + 1) k=1 4k 2 − 1
ou encore
1 1 (2n)!
(F) √ ∼ 2n .
πn n→∞ 2 (n!)2
Par conséquent, la probabilité d’obtenir après 2n tirages d’un pièce de monnaie une nombre
égal de pile et de face est
n
cas favorables C2n 1 (2n)! 1
= 2n = 2n 2
∼ √ .
cas possibles 2 2 (n!) n→∞ πn
ce qu’il nous fallait établir.
√
Ì On effectue dans l’intégrale de Gauss le changement de variables t = nx
Z ∞ Z ∞
2
√
G := exp (−t )dt = n exp (−nx2 )dx.
0 0
Des inégalités (exercice !)
2 2 1
0 ≤ 1 − x2 ≤ e−x , 0 ≤ x ≤ 1 et e−x ≤ , x ≥ 0,
1 + x2
on déduit
√
Z 1 √
Z ∞
2 n dx
n (1 − x ) dx ≤ G ≤ n
0 0 (1 + x2 )n
et après les changements de variable x = sin(u) dans l’intégrale de gauche et x = tan(u)
dans celle de droite on voit apparaitre nos intégrales de Wallis
√ √
nI2n+1 ≤ G ≤ nI2n−2
soit
√
22n (n!)2 n √ (2n)! 2n π
≤ G ≤ n 2n 2
(2n)! 2n + 1 2 (n!) 2n − 1 2
√
π
et quand n tends vers l’infini, les deux extrémités de cette formule tendent vers d’après
2
(F), soit
Z ∞ √
2 π
G := exp (−t )dt = .
0 2
Í La preuve se décline en deux étapes : on commence par montrer qu’il existe une constante
C > 0 telle que
√ n n
n! ∼ C n
n→∞ e
√
puis avec la formule de Wallis on vérifie que C = 2π.
√ n n
Pour déterminer la valeur de la constante C on reporte l’équivalent n! ∼ C n dans
n→∞ e
(F) ce qui nous donne
2n
√ 2n
C 2n r
1 (2n)! e 1 2
√ ∼ ∼ =
πn n→∞ 22n (n!)2 n→∞ 2n 2 n 2n C n
2 C n
e
√
soit C = 2π, la formule de Stirling est démontrée.
Il est bon de rappeler qu’il existe une constante (constante d’Euler) γ > 0 telle que
n
X 1
(F) xn = = log(n) + γ + εn avec lim εn = 0.
k=1
k n→∞
Considérons alors
n
X (−1)k+1
yn =
k=1
k
on a
1 1 1 1
y2n = 1 − + − ··· + +
2 3 2n − 1 2n
1 1 1 1 1 1 1
= 1 + + + ··· + − 1 + + ··· +
3 5 2n − 1 2 2 3 n
1 1 1 1 1 1 1 1 1 1
= 1 + + + ··· + + 1 + + ··· + − 1 + + ··· +
3 5 2n − 1 2 2 3 n 2 3 n
1 1 1 1 1 1 1
= 1 + + + ··· + + − 1 + + ··· +
2 3 2n − 1 2n 2 3 n
= x2n − xn = log(2n) + γ + ε2n − log(n) − γ − εn
= log(2) + ε2n − εn −→ log(2) lorsque n → ∞
P
la série n≥1 yn est donc convergente et de somme log(2).
Groupons maintenant les termes en prenant alternativement deux termes positifs puis un
terme négatif, par exemple pour les premiers termes :
1 1 1 1 1 1 1 1
1, , − , , , − , , , − ...
3 2 5 7 4 9 11 6
si on désigne par zn la somme des n premiers termes de cette nouvelle suite
1 1 1 1 1 1
z3n = 1 + + + · · · + − + + ··· +
3 5 4n − 1 2 4 2n
1 1 1 1 1 1 1 1 1
= 1 + + + ··· + − + + ··· + − + + ··· +
2 3 4n 2 4 4n 2 4 2n
1
= x4n − (x2n + xn )
2
1 1 3
= log(4n) − (log(2n) + log(n)) + ε4n − (ε2n + εn ) −→ log(2) lorsque n → ∞
2 2 2
et la modification de l’ordre des termes a bien modifié la somme de la série. q
i Remarque : ...................................
Ë Passons à l’exemple : la suite (un )n est bien définie, décroissante minorée : elle est donc
convergente vers la solution l ∈ [0, 1] de sin(l) = l, soit l = 0. Pour voir si la méthode
précédente s’applique écrivons
i Remarque : L’exemple précédent est P assez classique, pour changer vous pouvez par
exemple montrer que si (an )n ⊂ R+ , Sn := nk=1 a2k et limn an Sn = 1 alors an ∼ n−1/3 .
n→∞
On a
X 1 1 1
n!rn = n! = + + ...
k≥n+1
k! n + 1 (n + 1)(n + 2)
i.e.
1
n!rn > .
n+1
On en déduit aussi
1 1 1
n!rn < + + · · · =
n + 1 (n + 1)2 n
soit la double inégalité. De là on tire immédiatement
(F) lim n n!rn = 1,
n→∞
puis,
sin(2πn!rn )
(8) n sin(2πen!) = n sin(2πn!rn ) = 2πrn n!n → 2π
2πn!rn
la limite résultant de (F) et (8) via sin(x) ∼ x. D’un autre côté, si e ∈ Q : sin(2πen!) = 0
x→0
d’où le résultat. q
i Remarques : ê On peut donner une variante (plus classique, plus rapide) basée sur
les mêmes inégalités : supposons que e = pq avec q > 1, vu ce qui précède
q
X 1 1
0 < q! e − = q!rq <
k!
k=0
q
autrement dit
q
!
X 1
q! e − ∈ ]0, 1[
k=0
k!
ce qui est visiblement absurde car
q q
! !
X 1 p X 1
q! e − = q! − ∈ N.
k=0
k! q k=0 k!
ê Le même argument montre que pour toute suite (εn )n ∈ {0, 1}N (non identiquement
P εn
nulle à partir d’un certain rang), le réel n≥0 est irrationel.
n!
ê Voir aussi page 177 pour une autre preuve de l’irrationalité de e.
ê Montrer que π est irrationnel est plus délicat, une démonstration est donnée dans l’exer-
cice suivant.
La divergence de la série harmonique (vers +∞) assure l’existence de la suite (an )n . On peut
remarquer qu’une récurrence donne facilement pour tout n > 1
1 1 1 1 1 1
+ + ··· + <1 & + + ··· + >1
n n+1 2n − 1 n n+1 3n − 2
i.e. 2n − 1 < an < 3n − 2, donc si ann n converge, sa limite sera dans l’intervalle [2, 3]. Mais,
vu la définition de an :
1 1 1 1
+ 1< + ··· + <1+
n n+1 an an
en comparant avec une intégrale
Z an
1 1 1 dt 1 1 1
1− < + ··· + < < + + ··· + ≤1
n n+1 an n t n n+1 an − 1
i.e.
1 a
n
1− < log <1
n n
le résultat suit en prenant l’exponentielle. q
X1 1
1 1
1 1
1 1
1 1
1 1
r := = 1+ + + + + + ··· > + + + + + + · · · = r !!
n≥1
n 2 3 4 5 6 2 2 4 4 6 6
Exercice 207
Pn (Divergence de la série harmonique (suite)) ([10],1993/94).
1
Soit sn = k=1 k montrer que lim s2n − sn = log(2). Conclusion ?
n
On peut écrire
2n n n n
X 1 X 1 1X 1 1X k 1
s2n − sn = = = k
= f où f (t) = ,
k=n+1
k k=1
n+k n k=1 1 + n
n k=1 n 1+t
on reconnaît là une somme de Riemann sur [0, 1] associée à la fonction f ∈ C 0 ([0, 1]) par
conséquent
Z 1
lim s2n − sn = f (t)dt = log(2).
n→∞ 0
La suite (sn )n ne vérifie pas le critère de Cauchy, la série harmonique est donc divergente.q
i Remarque : l’intérêt de cet exercice est bien entendu d’utiliser les sommes de Riemann,
car la divergence de la série harmonique via le critère de Cauchy s’obtient classiquement
(voir aussi l’exercice précédent) par
1 1 1 1 1
s2n − sn = + ··· + ≥ + ··· + = .
n+1 2n 2n 2n 2
Supposons (xn )n divergente. Pour une suite bornée de réels divergente il n’y a qu’une alter-
native : elle possède deux sous-suites (xsn )n , (xtn )n qui convergent vers deux limites distinctes
l1 , l2 . Vu les hypothèses
il existe donc k1 ∈ N tel que ial1 = ial2 + 2ik1 π i.e. a(l1 − l2 ) = 2k1 π. De même avec la
seconde suite il existe k2 ∈ N : b(l1 − l2 ) = 2k2 π. De ces deux relations on déduit aussitôt
a k1
(car l1 − l2 6= 0) que = ce qui est bien sur absurde. q
b k2
2
Exercice 209 (Divergence de la série n≥1 sinn(n) ) [33]
P
[ π π 3π 5π
[− + 2kπ, + 2kπ] ∪ [ + 2kπ, + 2kπ]
k∈Z
4 4 4 4
réunion disjointe d’intervalles de longueur < 2 donc ne contenant pas trois entiers consécutifs ;
ils sont aussi séparés par des intervalles de longueur strictement plus grande que 1, donc
contenant au moins un entier : l’assertion est donc claire.
Notre série étant à termes positifs, elle sera de même nature que celle de terme général vn =
u3n + u3n+1 + u3n+2 et vu ce qui précède l’un des trois réels sin2 (3n), sin2 (3n + 1), sin2 (3n + 2)
est supérieur ou égal à 1/2 donc
1 1
vn ≥ ∼
2(3n + 2) n→∞ 6n
les an étant positifs, il suffit de montrer que les sommes partielles se dominent mutuellement,
pour cela, posons
1K.Knopp, « Theory and Application of infinite series », Dover (1990), pages 120-121.
1 Z 1−η Z 1
Z
n
n n
n
t (f (t) − f (1))dt ≤ n
t |f (t) − f (1)|dt + t |f (t) − f (1)|dt
0 0 1−η
2kf k∞ (1 − η)n+1
ε
≤ n + ≤ 2ε pour n ≥ Nε
n+1 n+1
i.e.
Z 1 Z 1
n nf (1)
lim n t f (t)dt = lim n tn f (1)dt = lim = f (1).
n→∞ 0 n→∞ 0 n→∞ n + 1
q
On procède par contraposée : xn = xm pour n > m assure que Tx est fini et alors bien
entendu fermé ; on peut donc sans perte de généralité supposer les xn deux à deux distincts.
Tx = {xn : n ∈ N} fermé dans [a, b] est compact : il existe une sous-suite convergente
(xnk )k de limite a ∈ Tx et par continuité de f : xnk +1 = f (xnk ) → f (a). Ainsi, sauf peut
être pour un nombre fini, tous les éléments de Tx sont des points d’accumulation. On peut
donc supposer, quitte à supprimer le nombre fini de points isolés, que tous les points sont
d’accumulation. On peut......................................................... o
P suite croissante (SN )N est bornée,Pdonc convergente, il en est donc de même de la série
La
n≥1 (b1 + · · · + bn )(an − an−1 ) = − n un = − limN SN . o
P∞ 1
Exercice 214 (Calcul de n=0 F (2n ) ) [34], 1984/7.
Montrer que √
∞
X 1 7− 5
= ,
n=0
F (2n ) 2
où (F (n))n est la suite de Fibonacci :
F (0) = 0, F (1) = 1, F (n) = F (n − 1) + F (n − 2), n ∈ N \ {0, 1}.
L’indication est claire : si la série converge, par le critère de Cauchy elle ne peut admettre de
blocs de termes consécutifs de somme arbitrairement grande. Observons donc les quantités
X cos(log(log(n)))
Sl = .
k∈N
log(n)
l
(on peut remarquer que la croissance de k 7→ log2 (k) assure que Nk est bien constitué
d’éléments consécutifs) On peut aussi écrire
Nl = { k ∈ N : exp(exp(2πl − π/3)) ≤ k ≤ exp(exp(2πl))}
et il en résulte
](Nl ) ≥ exp(exp(2πl)) − exp(exp(2πl − π/3)) − 1
= exp(exp(2πl)) − exp(α exp(2πl)) − 1
où 0 < α = exp(−π/3) < 1. Ainsi, pour k ∈ Nl on a la minoration
cos(log2 (k)) cos(−π/3) 1
≥ = ,
log(k) exp(2πl) 2 exp(2πl)
d’où
exp(exp(2πl)) − exp(α exp(2πl)) − 1
Sl ≥ .
2 exp(2πl)
Pour conclure il n’y a plus qu’à remarquer que
exp(x) − exp(αx) − 1 exp(x)
lim = lim [1 − exp(x(α − 1)) − exp(−x)] = +∞
x→+∞ 2x x→+∞ 2x
(le terme entre crochets tendant vers 1 car 0 < α < 1). o
√
i Bien remarquer que [n] est toujours bien défini car l’équation n = k + 1/2, k, n ∈ N
n’a pas de solutions.
Comme souvent dans ce genre d’exercice l’astuce consiste à sommer paquets (ce qui est
parfaitement légitime puisque la série est à termes positifs...sous reserve de convergence...)
√
[n] = k ⇔ k − 1/2 < n < k + 1/2
⇔ k − k + 1/4 < n < k 2 + k + 1/4
2
⇔ k2 − k + 1 ≤ n ≤ k2 + k (car n ∈ N)
∞ ∞ 2 +k
kX
!
X 2[n] + 2−[n] X
= (2k + 2−k )2−n
n=1
2n k=1 n=k2 −k+1
∞ 2k−1
−k2 +k−1
X X
−k
= k
(2 + 2 )2 2−n
k=1 n=0
∞
X 2 +k−1 1 − 2−2k+2
= (2k + 2−k )2−k ·
k=1
1 − 2−1
∞
2 +k 2 −k
X
= (2k + 2−k )(2−k − 2−k )
k=1
X∞
= (2−k(k−2) − 2−k(k+2) )
k=1
∞
X ∞
X
= 2−k(k−2) − 2−k(k+2)
k=1 k=1
∞
X ∞
X
= 2−k(k−2) − 2−l(l−2) (l = k + 2)
k=1 l=3
2
X
= 2−k(k−2) = 2 + 1 + 3.
k=1
C.Q.F.D. o
Ê Le terme général du produit de Cauchy des deux séries (convergentes car alternées) est
n n
X X (−1)k+1 (−1)n−k+1
cn = ak bn−k = √ ·√
k=0 k=0
k+1 n−k+1
n
X 1
= (−1)n √ √ ,
k=0
k+1 n−k+1
donc n n
X 1 X 1
|cn | = √ √ ≥ =1
k=0
k + 1 n − k + 1 k=0
n + 1
P P
d’où la grossière divergence du produit de Cauchy des deux séries n an et n bn .
P
Exercice 218 (e = 1/k!, une preuve élémentaire ) CMJ, 1-1994.
Montrer que pour tout x ∈]0, 1] : 1 + ex > ex > 1 + x. En déduire que pour tout
n ∈ N et x ∈]0, 1] :
x2 xn exn+1 x2 xn+1
1+x+ + ··· + + > ex > 1 + x + + ··· +
2! n! (n + 1)! 2! (n + 1)!
∞
X 1
et enfin que = e.
k=0
k!
i Remarque : Bien entendu on peut encore mieux exploiter l’inégalité précédente que l’on
peut réecrire sous la forme :
n
e x
X xk 1
>e − > , ∀ n ∈ N,
(n + 1)! k=0
k! (n + 1)!
soit
∞
x
X xk
e = , ∀ x ∈]0, 1].
k=0
k!
P
Exercice 219 (Divergence « douce » de k 1/k log(k) log(log(k)) par le TAF )
CMJ, 5-1986.
Utiliser le théorème des accroissements finis pour Pétablir la divergence de la sé-
rie de terme général 1/k log(k) log(log(k)). Estimer m≤k≤n 1/k log(k) log(log(k)),
conclusion ?
241
242/408 Petit Bestiaire d’Exercices pour l’Oral de l’Agrégation Interne Patrice Lassère
f est donc bornée par 10 sur [0, 1] (bien remarqer que nous n’avons pas utilisé la continuité
des applications fn ).
Ì Montrons qu’en général il n’y a pas convergence uniforme sur [0, 1]. Pour cela, considérons
une suite strictement croissante x0 = 0 < x1 < x2 < · · · < 1 de limite 1 et les applications
fn ∈ C ([0, 1]) définies de la manière suivante :
0 ≤ fn ≤ 1,
fn ≡ 0 sur [0, 1] \ [xn , xn+1 ] et fn (yn ) = 1 où yn = (xn + xn+1 )/2,
fn est affine sur [xn , yn ] et [yn , xn+1 ].
Il n’est pas difficile de vérifier que la suite (fn )n ainsi définie vérifie (8), en effet les supports
des applications fn étant mutuellement disjoints
(
0 si n 6= m
|fn (x)fm (x)| = 2
≤ 2−|n−m| , ∀ n, m ∈ N.
|fn (x)| ≤ 1 si n = m
Maintenant, comme (toujours par disjonction des supports)
X
f (x) = fk (x) = fn (x) où x ∈ [xn , xn+1 ],
k≥0
en particulier
f (yn ) = 1, ∀ n ∈ N.
Ainsi
f (0) = 0, lim yn = 1 et lim f (yn ) = 1,
n n
f est donc discontinue au point 1 et la convergence ne peut être uniforme sur [0, 1]. o
i Remarque : Dans l’exemple précédent, f est discontinue au point 1 en lequel toutes
les applications fn s’annulent ; il est en fait assezPfacile de vérifier qu’en un point x ∈ [0, 1]
où il existe un entier n tel que fn (x) 6= 0 la série n fn est normalement convergente sur un
voisinage de x et f est par suite continue au point x.
l’échange « intégrale/série » étant justifié par la normale convergence de la série entière sur
[0, x] ⊂ [0, 1[ , ainsi
xm+1 X xm+1 xm+1
X
log(f (x)) = = −
m≥0
(m + 1)(m + 2) m≥0 m + 1 m + 2
1 X xm+1
1 1
=1+ 1− =1+ 1− log
x m≥0 m + 1 x 1−x
implique
∀ ε > 0, ∃ nε : m, n ≥ nε =⇒ sup kfn (x) − fm (x)kE ≤ ε
x∈Y
Le critère de Cauchy uniforme est donc bien vérifié sur D(0, 1), soit Ë. q
i Remarque : Le candidat à l’agrégation externe peut régler l’implication « délicate » (Ì
=⇒ Ë) trés simplement en invoquant le principe du maximum.
Ì Si (fn )n converge continuement vers f sur A, montrer que f est continue sur
A (même si les fn ne sont pas continues !)
Í Montrer que toute suite (fn )n uniformément convergente sur A vers une fonc-
tion f ∈ C (A, K) converge continuement sur A. La réciproque est-elle vraie ?
Î Soit (fn )n une suite de fonctions définies sur une partie compacte K. Montrer
que les propriétés suivantes sont équivalentes.
ê La suite (fn )n est uniformément convergente vers f ∈ C 0 (K).
ê La suite (fn )n est continuement convergente sur K vers f .
Ì Avec la première question, si (fn )n converge continuement vers f sur A, elle converge
simplement sur A vers f . Montrons que f est continue sur A : soit x ∈ A, (xn )n ⊂ A une
suite convergente vers x. Pour tout ε > 0, par la convergence de (fn (x1 ))n vers f (x1 ) il existe
n1 (qui à priori dépend de x1 ) tel que
ε
|fn1 (x1 ) − f (x1 )| ≤ .
2
De même, il existe n2 > n1 tel que
ε
|fn2 (x2 ) − f (x2 )| ≤ .
2
En réitérant ce processus, on construit une suite strictement croissante d’entiers vérifiant
ε
(1) |fnk (xk ) − f (xk )| ≤ ∀ k ∈ N.
2
Mais avec la question précédente limk→∞ fnk (xk ) = f (x), si bien qu’il existe aussi un entier
k0 tel que
ε
(2) |fnk (xk ) − f (x)| ≤ , ∀ k ≥ k0 .
2
Finalement (1) et (2) assurent que
|f (xk ) − f (x)| ≤ |fnk (xk ) − f (xk )| + |fnk (xk ) − f (x)| ≤ ε ∀ k ≥ k0
et f est continue au point x, elle est donc continue sur A.
Í ê Supposons maintenant que la suite (fn )n converge uniformément sur A vers une
fonction continue f (les fonctions fn n’étant pas continues, l’hypothèse de continuité sur f
est essentielle vu la question précédente) et montrons que la convergence est continue sur A.
Soit donc (xn )n ⊂ A une suite convergente vers x ∈ A. Soit ε > 0, par convergence uniforme
sur A nous avons
ε
(3) |fn (xn ) − f (xn )| ≤ sup |fn (y) − f (y)| ≤ , ∀ n ≥ n0 .
y∈A 2
Et par continuité de f
ε
(4) |f (xn ) − f (x)| ≤ , ∀ n ≥ n1 .
2
tn f (t) et n n
P P
Exercice 225 (Étude des séries de fonctions n≥0 n≥0 (−1) t f (t)
) ([10] 1993/94 et 1997/98.)
Ê Notons pour n ∈ N
(
[0, 1] −→ R
fn (t) =
t 7−→ tn f (t)
ê Remarquons qu’il y a bien sûr simple convergence sur [0, 1[ de somme f1+t(t)
. En outre il y
aura convergence en t = 1 si, et seulement si, f (1) = 0. En résumé la série est simplement
convergente sur [0, 1] si, et seulement si, f (1) = 0 et
(
f (t)
X
n si t ∈ [0, 1[
t f (t) = 1+t
n≥0
0 si t = 1.
ê Supposons
P f (1) = 0 et désignons par Rn le reste d’ordre n de la série simplement conver-
gente n≥0 fn . On a donc :
(
tn+1 f (t)
k
= −tn+1 f (t)−f (1)
P
Rn (t) = k≥n+1 t f (t) = 1−t t−1
, t ∈ [0, 1[
Rn (1) = 0
Supposons f dérivable en t = 1 avec f 0 (1) = 0 et soit ε > 0, il existe 0 < η < 1 tel que
f (t) − f (1)
∀ t ∈ [1 − η, 1[, ≤ ε,
t−1
de sorte que
soit
n ≥ N =⇒ |Rn (t)| ≤ ε
soit
f (t) − f (1)
(3) ∀ t ∈ [0, 1] : ≤ εt−(N +1)
t−1
et comme limt→1− t−(N +1) = 1 il existe 0 < η < 1 tel que
t = 1 et f 0 (1) = 0
i.e. f est dérivable enP
En résumé, la série n≥0 tn f (t) converge uniformément sur [0, 1] si, et seulement si f est
dérivable au point t = 1 avec f (1) = f 0 (1) = 0.
P n n f (t)
Ë ê Pour 0 ≤ t < 1 la série est bien évidement convergente avec n≥0 (−1) t f (t) = 1+t
.
Pour t = 1 la série converge si et seulement si, f (1) = 0.
ê Étudions maintenant la convergence uniforme sur [0, 1[. Le reste de la série est
X (−t)n+1 f (t)
Rn (t) := (−1)k tk f (t) = ,
k≥n+1
1+t
1
si bien que ( 1+tvariant sur [0, 1[ entre 1/2 et 1) la série converge uniformément sur [0, 1[ si,
et seulement si, la suite (tn+1 f (t))n tend uniformément vers zéro sur [0, 1[. Supposons que f
tende vers 0 en 1 : pour tout ε > 0, il existe 0 < η < 1 tel que
max{ε, M (1 − η)n+1 }
|Rn | ≤ ≤ 2ε, pour n assez grand
1+t
d’où la convergence uniforme sur [0, 1[ et donc sur [0, 1] si on pose f (1) = 0.
ê Supposons maintenant la convergence uniforme sur [0, 1[, en particulier pour tout ε > 0
il existe un entier N ∈ N tel que
lim f (t) = 0
t→1−
ê En résumé, la série converge uniformément sur [0, 1[ si, et seulement si limt→1− f (t) = 0.
Si par ailleurs f (1) = 0 la convergence est alors uniforme sur [0, 1]. q
Soit (Pn )n une telle suite et f sa limite. La suite (Pn )n satisfait donc au critère de Cauchy
uniforme sur R, en particulier avec ε = 1
(∃N ∈ N : n ≥ N & p ∈ N ) =⇒ sup |Pn (x) − Pn+p (x)| ≤ 1
x∈R
∀ p ∈ N : ∃ Cp ∈ R : PN − PN +p = Cp (8)
et puisque
On passe alors à la limite sur p dans (8) pour tout réel x ∈ R arbitraire :
soit
f (x) = l + PN (x), x ∈ R.
q
qui impliquent
(4) ∀ k ∈ N, ∀ n ∈ Z : |cn (f (k) )| = |n|k |cn (f )|.
Soit, vu les hypothèses sur f et ses dérivées :
(8) Z π Z π
1 1
f (k) (t)e−int dt = f (k) (t)e−int dt ≤ 1.
∀ k ∈ N, ∀ n ∈ Z : |cn (f (k) )| =
2π −π 2π −π
Supposons maintenant qu’il existe n0 ∈ Z tel que
|n0 | ≥ 2 et |cn0 (f )| > 0
alors vu (4) :
lim |cn0 (f (k) )| = lim n0 k |cn0 (f )| = +∞
k→+∞ k→+∞
contredisant (8), ainsi
cn (f ) = 0, ∀ n ∈ Z \ {−1, 0, 1}
et la série de Fourier de f est de la forme
c−1 (f )e−ix + c0 (f ) + c1 (f )eix = a cos(x) + b sin(x) + c où a, b, c ∈ R.
f étant C ∞ et 2π-périodique sur R, les théorèmes classiques de convergence nous assurent
que
f (x) = a cos(x) + b sin(x) + c, ∀ x ∈] − π, π[.
Seconde étape : Il ne reste plus qu’à exploiter les trois dernières hypothèses
f 0 (0) = 1, kf k∞ ≤ 1 et kf 0 k∞ ≤ 1
En effet, f 0 (0) = 1 implique b = 1 et, au voisinage de zéro
f 0 (x) = a sin(x) − cos(x) = −1 + ax + o(x)
qui sera, si a 6= 0, strictement plus petit que −1 lorsque x tendra vers zéro suivant le signe
contraire de a : c’est contraire à l’hypothèse et donc a = 0. Regardant au voisinage de π2 , on
obtient c = 0, soit finalement
f (x) = sin(x) ∀ x ∈ R.
Q.E.D. o
i Remarque : Sans la 2π-périodicité, on pourrai bien sûr envisager de réitérer le même
raisonnement sur la fonction g 2π-périodique sur R et égale à f sur ] − π, π] pour en déduire
que g puis f (car f est clairement développable en série entière sur R et égale à g sur ]−π, π] )
coïncide avec la fonction sinus sur ]−π, π] ; mais malheureusement une obstruction apparait :
g n’étant même plus continue (à priori) en les points de πZ, les formules ∀ k ∈ N, ∀ n ∈ Z :
cn (g (k) ) = (in)k cn (g) ne sont plus valables. En effet elle s’obtiennent aprés k intégrations par
parties dans cn (g (k) ), les termes « entre crochets » disparaissant grâce au caractère continu
(et la 2π périodicité) de g en −π et π ; de ce fait, dans notre cas, la non continuité de g et
à fortiori de ses dérivées en ces points va faire apparaitre à droite un polynôme en n qui
(semble) rendre vain tout espoir de généralisation par cette méthode....
Ë Le cas général On va proposer deux solutions (ces deux solutions sont tirées du vo-
lume 116-3 de la RMS, [10]), l’une s’appuyant sur la théorie des fonctions holomorphes,
l’autre sur l’analyse fonctionnelle. Commencons par fixer quelques notations communes aux
deux solutions.
Pour toute fonction bornée sur R, on pose kf k = supx∈R |f (x)|. L’espace vectoriel
EC := {f ∈ C ∞ (R, C) : ∃ c > 0 ∀ n ∈ N, ∀ x ∈ R |f (n) (x)| ≤ c}
(on définit de même l’espace réel EC ) sera normé par N (f ) = sup kf (n) k ; on désignera par ER
n∈N
l’espace vectoriel réel ER ⊂ EC des applications à valeurs réelles. Enfin, B désignera l’espace
des fonctions entières f vérifiant
∃c > 0 : ∀ z ∈ C, |f (z)| ≤ ce|Im(z)| ,
il est normé par NB (f ) = sup |f (z)|e−|Im(z)| .
z∈C
Résultats préliminaires :
Lemme 3.
Pour toute application f ∈ E , il existe une unique fonction entière f˜ dont la restriction
à R est f . De plus, f˜ ∈ B et NB (f˜) ≤ N (f ).
Pour tout t ∈ R (resp. τ ∈ C) et f ∈ E (resp. B) les translatés x 7→ ft (x) = f (x + t)
(resp. fτ ) restent dans E (resp. B).
De toute suite (fn )n bornée de (E , N ) il existe une application f ∈ E et une sous-suite
(j)
(fnk )k telles que pour tout entier j ∈ N la suite (fnk )k converge uniformément sur tout
compact de R vers f (j) (i.e. converge dans1 C ∞ (R)).
Preuve : Les hypothèses sur f assurent qu’elle est la restriction à la droite réelle de la série
entière
X f (n) (0)
f˜(z) := zn
n≥0
n!
de rayon de convergence infini. En outre pour tout x + iy ∈ C
X f (n) (x) X
|y|
|f˜(x + iy)| ≤ = e|y| N (f )
n
(iy) ≤
n! n!
n≥0 n≥0
soit f ∈ B et NB (f˜) ≤ N (f ).
La seconde assertion est immédiate et la troisième se déduit du fait que toute partie bornée
de (E , N ) est bornée dans l’espace de Montel C ∞ (R) .
Quitte à remplacer chaque fk par une de ses translatés (via le lemme, mais il faut être
tout de même délicat car le « sup » peut être atteint « à l’infini »...) on peut supposer que
limk Q(fk ) = ρ. La troisième assertion du lemme assure alors de l’existence d’une sous-suite
(j)
(fnk )k et d’une application f ∈ ER telles que pour tout entier j ∈ N la suite (fnk )k converge
uniformément sur tout compact de R vers f (j) . Clairement, f ∈ X qui est non vide.
K ρ = 1.
Supposons par l’absurde que ρ > 1. Dans ce cas, pour tout f ∈ X : f (0)f 0 (0) 6= 0 (ne pas
oublier que dans ce cas Q(f ) = f 2 (0) + f 02 (0) > 1 et max{|f (0)|, |f 0 (0)|} ≤ 1...) ; on peut
donc, en invoquant la seconde étape, en déduire que
∀ f ∈ X, n ∈ N : f (n) ∈ X et f n+2 (0) = −f (n) (0).
Toutes les applications f étant développables en série entière, X est donc inclu dans le plan
engendré par les fonctions sin et cos, mais sur ce plan Q et N 2 coïncident ce qui contredit
l’assertion ρ > 1.
Seconde solution :
+∞
α
X α(α − 1) . . . (α − n + 1)
(F) (1 + x) = 1 + xn
n=1
n!
en déduire pour |x| < 1 que
+∞
1 X (2n − 3)!!
(8) |x| = 1 − (1 − x2 ) − (1 − x2 )n
2 n=2
(2n)!!
 Montrer que la suite de polynômes (Pn )n définie par
1
x − Pn2 (x) , n ∈ N
P0 (x) = 0,
Pn+1 (x) = Pn (x) +
√ 2
converge uniformément sur [0, 1] vers g(x) = x. En déduire une suite de poly-
nômes qui converge uniformément sur [−1, 1] vers f (x) = |x|.
Ê x ∈ [−1, 1], on pose donc x = cos(t), t ∈ [−π, π]. La fonction g(t) = f (cos(t)) = | cos(t)|
est 2π-périodique continue et C 1 par morceaux : elle est donc développable en série de Fourier
et sa série de Fourier converge uniformément sur R. Par parité
N
!
X a0
| cos(t)| = lim an cos(nt) + uniformément en t ∈ [−π, π].
N →∞
n=1
2
Il est alors facile de vérifier que pour tout n ∈ N, cos(nt) est un polynôme en cos(t) (écrire
cos(nt) = re ((cos(t) + i sin(t))n ) et dans ce dernier terme les sinus apparaissent sous une
k
puissance paire, il ne reste plus qu’à écrire sin2k (t) = (1 − cos2 (t)) ...). Ainsi | cos(t)| est
limite uniforme sur [−π, π] de polynômes en cos(t), il ne reste plus qu’à remplacer cos(t) par
x pour conclure.
Ë On peut obtenir la formule (F) avec les séries entières en écrivant (1+x)α = exp (α log(1 + x)),
on peut tout aussi bien le faire en montrant dans la formule de Taylor-Lagrange appliquée
à (1 + x)α à l’ordre N , que le reste rN (x) tend vers zéro lorsque N tend vers l’infini et ceci
pour tout x ∈] − 1, 1[ en effet
N
X α(α − 1) . . . (α − n + 1) α(α − 1) . . . (α − N ) N +1
(1 + x)α = 1 + xn + x (1 + xθx )α−N −1
n=1
n! (N + 1)!
N
X α(α − 1) . . . (α − n + 1)
=1+ xn + rN (x)
n=1
n!
où 0 < θx < 1. Or, pour tout x ∈] − 1, 1[
α(α − 1) . . . (α − N ) N +1
lim x =0
N →∞ (N + 1)!
de sorte que pour s’assurer que limN rN (x) = 0, il suffit de montrer que la suite (1 + xθx )α−N −1 N
d’où (F) pour x ∈ [0, 1[ et donc sur ] − 1, 1[ puisque le domaine de convergence d’une série
entière est toujours une boule.
Pour obtenir (8) il n’y a plus qu’à écrire
p
|x| = 1 − (1 − x2 )
et appliquer (F) avec α = 1/2.
Ì La troisième question est plus classique, consultez votre livre de chevet favori.
i Remarques : ê On peut s’étonner d’un tel engoument pour approcher la valeur absolue :
c’est un « tic historique » probablement du à la preuve trés ingénieuse que le jeune H.Lebesgue
(23 ans) donne en 1898 du célèbre théorème de Weierstrass (toute fonction continue sur un
intervalle [a, b] est limite uniforme de polynômes), pour cela, il commence par observer qu’il
est facile d’approcher une fonction continue par une application continue affine par morceaux,
qu’un tel objet est combinaison linéaire de translations de |x| ; les polynômes étant invariant
par translation, il suffit donc d’approcher uniformément |x| sur tout voisinage de l’origine.
Ce que fit Lebesgue en exhibant justement celle de la seconde série (ce fut sa première
publication...).
ê Dans la seconde question, la théorie des séries entières assure la convergence uniforme
(et même normale) seulement sur tout [−a, a] ⊂] − 1, 1[. En fait comme pour les autres
exemples, la convergence est uniforme sur [−1, 1], elle résulte de...................................... q
Que f soit de classe C ∞ et que l’on puisse dériver sous la somme relève du théorème de
Weierstrass, en effet les séries des dérivées de tout ordre sont normalement convergente sur
R car l’exponentielle « l’emporte » sur la puissance et les fonctions sin et cos sont bornées
sur R...
X
f ∈ C ∞ (R) et ∀ k ∈ N, x ∈ R : f (k) (x) = (k)
fm (x)
m≥0
en particulier
X
f (2n) (0) = (−1)n e−m (m2 )2n .
m≥0
Donc, sous réserve de convergence, la série de Taylor de f à l’origine est donnée par (les
dérivées impaires de f sont nulles)
!
X f (n) (0) X f (2n) (0) X (−1)n x2n X
xn = x2n = e−m (m2 )2n .
n≥0
n! n≥0
(2n)! n≥0
(2n)! m≥0
en particulier
1+cos(x)
Exercice 230 (Développement en série de Fourier de f (x) = 4−2 cos(x)
, série
entière ) ([10],1997/98).
1 + cos(x)
Développer en série de Fourier la fonction f (x) = .
4 − 2 cos(x)
f est clairement C ∞ sur R, paire, 2π-périodique, elle admet donc un développement en série
∞
X
de Fourier de la forme f (x) = an cos(nx) et la convergence est uniforme sur R (Dirichlet).
n=0
Il est sage de se persuader qu’un calcul direct des coefficients de Fourier
Z 2π
1 1 + cos(x)
an = cos(nx)dx
2π 0 4 − 2 cos(x)
est plus qu’incertain, voire déconseillé, voici donc deux méthodes qui donnent ce développe-
ment par des chemins détournés.
Ê Première méthode :
1 + cos(x) 2 + eix + e−ix 2eix + e2ix + 1 ix X 2 + 2X + 1
f (x) = = = = F (e ) où F (X) = −
4 − 2 cos(x) 2(4 − eix − e−ix ) 2(4eix − e2ix − 1) 2(X 2 − 4X + 1)
√
Les pôles de F sont α = 2 − 3 et α−1 et on peut (après avoir décomposé en éléments
simples ) développer F en puissance relatives de X, (on retrouve le développement en série
de Laurent de la fonction méromorphe F sur C sur la couronne C(0, α, α−1 ) pour ceux qui
s’en souviennent)
√ √ !
1 α 3 3
F (X) = − 1− −
2 X(1 − Xα ) 1 − αX
√
1 3 X |n| n
=− + α X , α < |X| < α−1
2 2 n∈Z
le cercle unité étant clairement inclus dans la couronne {α < |X| < α−1 } on peut faire
X = eix dans le développement précédent
√ √ √
1 + cos(x) ix 1 3 X |n| inx 3−1 3 X√ √ n
f (x) = = F (e ) = − + α e = + 3 2 − 2 cos(nx), ∀ x ∈ R.
4 − 2 cos(x) 2 2 n∈Z 2 2 n≥1
Nous avons donc trouvé un développement en série trigonométrique de f sur R, visiblement
normallement convergente sur R : c’est le développement en série de Fourier de f (la conver-
gence uniforme sur un intervalle de longueur 2π permet de s’en assurer (échange justifié de
somme et d’intégrale))
∞
X 1 + cos(x)
Ë Seconde méthode : nous savons que f (x) = an cos(nx) = qu’on peut
n=0
4 − 2 cos(x)
aussi écrire
∞
X
(4 − 2 cos(x)) an cos(nx) = 1 + cos(x)
n=0
la convergence étant normale sur R en réordonnant, on trouve
X
(4a0 − a1 − 1) + (4a1 − a0 − a2 − 1) cos(x) + (4an − an−1 − an+1 ) cos(nx) = 0
n≥2
cette série trigonométrique étant normalement convergente sur R ses coefficients sont nuls
(c’est du cours) donc
4a0 − a1
=1
(F) 4a1 − a0 − a2 =1
4a − a
n n−1 − an+1 = 0, ∀ n ≥ 2.
2
on retrouve un système classique à résoudre √ n caractéristique est r − 4r + 1 = 0
√ n : son équation
la solution générale est an = λ 2 + 3 + µ 2 − 3 . Comme limn an = 0, λ est nul et
√ n
a√n = a1 2 − 3 , n ≥ 1, enfin avec les deux premières équations de (F) il vient a0 =
3−1
√ √
2
, a 1 = 3 2 − 3 et finalement
√ √
3−1 3 X√ √ n
f (x) = + 3 2 − 2 cos(nx).
2 2 n≥1
Exercice 231 (Inégalité de Bernstein via les séries de Fourier) A.Pommellet, [10]-6,
1992/93.
n n n n
X X πλk 2Λ X πλk X
P (t) = ak eiλk t = ak ei 2Λ π
t
= ak ei 2Λ
u
= ak eiλ̃k u = P̃ (u)
k=−n k=−n k=−n k=−n
π
kP k∞ = kP̃ k∞ , et kP 0 k∞ = kP̃ 0 k∞
2Λ
π π π
kP̃ 0 k∞ ≤ kP̃ k∞ , soit kP 0 k∞ ≤ kP k∞
2 2Λ 2
i.e.
(8) kP 0 k∞ ≤ Λ kP k∞
Il ne reste donc plus qu’à établir (8) pour tout polynôme trigonométrique généralisé vérifiant
Λ = π2 .
X 4(−1)l
Ψ(t) = sin ((2l + 1)t) , ∀ t ∈ R.
l≥0
π(2l + 1)2
n
X
Í Soit donc P (t) = ak eiλk t un polynôme trigonométrique généralisé vérifiant Λ = π2 .
k=−n
La grande astuce consiste à remarquer qu’alors
π π π
Λ= =⇒ (λk = Ψ(λk ), −n ≤ k ≤ n) puisque sur [− , ] : Ψ(t) = t
2 2 2
on a alors :
X n
|P 0 (t)| = ak iλk eiλk t
k=−n
X n
iλk t
= iak Ψ(λk )e
k=−n
!
n l
X X 4(−1)
iλk t
= iak sin ((2l + 1)λ ) e
2 k
π(2l + 1)
k=−n l≥0
n l
X X 2(−1) i(t+2l+1)λk i(t−2l−1)λk
= ak e −e
π(2l + 1)2
l≥0 k=−n
X 2
≤ (|P (t + 2l + 1) |+| P (t − 2l − 1)|)
l≥0
π(2l + 1)2
4kP k∞ X 1
≤
π l≥0
(2l + 1)2
π
=kP k∞ , ∀ t ∈ R,
2
P 1 2 2
= π8 qu’on déduit de
P1
(dans la dernière inégalité on a utilisé (2l+1)2 l2
= π6 ...) soit
kP 0 k∞ ≤ π2 kP k∞ d’où (8) si Λ = π2 , et d’où (8) pour tout polynôme trigonométrique
généralisé vu la question Ë, d’où l’inégalité de Bernstein par la question Ê. q
Exercice 232 (Une fonction continue non dérivable à l’origine mais dévelop-
pable en série de Fourier (1)) √
Soit f : R → R la fonction paire, 2π-périodique égale à x sur [0, π].
Ê Y a-t-il dans le cours un théorème permettant d’affirmer que f est déve-
loppable en série de Fourier ?
Z x
Ë Soit G la fonction définie sur R par G(x) = sin(t2 )dt, montrer que pour
0
tout x > 0
Z x
1 − cos(x2 ) 1 − cos(t2 )
G(x) = + dt.
2x 0 2t2
En déduire que la limite lim G(x) existe, est finie et strictement positive.
x→+∞
2 π
Z
Ì Soit pour n ∈ N, an = f (t) cos(nt)dt. à l’aide de la question précé-
π 0
dente montrer que an = 0(n−3/2 ).
Í Montrer sans Fejèr que f est développable en série de Fourier.
Î Montrer avec Fejèr que f est développable en série de Fourier.
Ê Non car en les points 2kπ, (k ∈ Z) f n’est pas dérivable et n’admet pas de dérivée à
droite et à gauche (ailleurs f est continue et C ∞ sauf en les points (2k + 1)π, k ∈ Z ou f
n’est pas dérivable mais admet une dérivée à droite et à gauche).
Ë C’est une bestiale intégration par parties faisant apparaître une intégrale généralisée
(avec la régle habituelle qu’elle marche si deux termes parmi les trois existent : en zéro pas
problème — faire un DL—) il en résulte (le premier terme à droite tend vers zéro car majoré
2)
par 1/2|x| et 1−cos(t
2t2
est intégrable en l’infini puisque majoré en module par 1/|x|2 ) que
R∞ 2)
limx→∞ G(x) existe et vaut 0 1−cos(t2t2
dt = C > 0.
Ì On a :
Z √nπ
2 π√ √
Z
2 u 2u du
an = t cos(nt)dt = √ cos(u2 ) (avec le changement u = nt)
π 0 π 0 n n
Z √nπ
4
= u2 cos(u2 )du
πn3/2 0
√nπ Z √nπ !
u sin2 (u) sin(u2 )
4
= − du
πn3/2 2 0 0 2
2 √ 2C
=− G( nπ) ∼ − (d’après la question précédente).
πn3/2 n→∞ πn3/2
Î Le théorème de Féjer2 assure que les sommes partielles de la série de Fourier d’une
fonction f continue convergent au sens de Césaro vers f , c’est donc ici le cas, mais la série
de Fourier de f est aussi convergente sur R et la convergence usuelle implique la convergence
au sens de Césaro et vers la même limite : le résultat suit. q
Exercice 233 (Une fonction continue non dérivable à l’origine mais dévelop-
pable en série de Fourier (2))
X sin(nx)
Soit f (x) = .
n≥1
n2
Ê Comme supx∈R | sin(nx)/n2 | = 1/n2 la série converge normalement sur R : f est donc
continue sur R, elle bien entendu 2π-périodique.
Ë La série définissant f est une série trigonométrique uniformément convergente sur R :
c’est donc la série de Fourier de f .
Pour justifier ce dernier point, si f (x) = Z αk eikx où la convergence est uniforme sur un
P
intervalle de longueur au moins 2π, il s’agit de montrer que pour tout k ∈ Z : αk = ck (f ).
Nous avons
Z 2π Z 2π Z 2π
1 −ikt 1 X
ilt −ikt 1 X
ck (f ) = f (t)e dt = αl e e dt = αl ei(l−k)t dt = αk
2π 0 2π 0 l∈Z
2π l∈Z 0
π
(1≤k≤N ) =⇒ 0 < kx ≤ E(π/2x)x ≤
2
sin(kx) 2kx 2
≥ = , ∀ k ∈ {1, . . . , N },
kx πkx π
et par suite
N
f (x) X sin(kx) 1 1 X sin(kx)
= +
x k=1
kx k x k≥N +1 k 2
N
2 X 1 1 X sin(kx)
≥ + .
π k=1 k x k≥N +1 k 2
n
!
X 1 − ei(n+1)x
ikx
|ϕn (x)| = Im e ≤
1 − eix
k=0
1 − ei(n+1)x
= −ix/2
e − eix/2
2
≤
|2i sin(x/2)|
1 π
≤ ≤
sin(x/2) x
où l’on a appliqué encore une fois l’inégalité de Jordan dans la dernière inégalité.
• D’où
N
f (x) 2X1
≥ + Rn (x)
x π k=1 k
N
2X1
≥ − |Rn (x)|
π k=1 k
N
2X1 1 X 1 1
≥ − |ϕn (x)| −
π k=1 k x k≥N +1 k 2 (k + 1)2
N
2X1 1 X π 1 1
≥ − −
π k=1 k x k≥N +1 x k 2 (k + 1)2
N
2X1 π
= − 2 2
π k=1 k N x
.
• Si x tends vers zéro, N = E(π/2x) tends vers +∞ et N 2 x2 /π tends vers 4/π de telle
sorte que dans l’inégalité précédente le terme de droite tends vers l’infini avec N qui implique
à son tour limx→0 f (x)
x
= +∞ : f n’est pas dérivable à l’origine. o
Exercice 234 (Une fonction continue dont la série de Fourier diverge à l’origine
)
On considère la suite de fonctions (fn )n≥1 définie sur [0, π] par
1 h 3 xi
fn (x) = 2 sin 2n + 1 .
nP 2
Ê Montrer que la série de fonctions n≥1 fn converge normalement sur [0, π].
On désigne alors par f la fonction paire,
P continue, 2π-périodique sur R
vérifiant pour tout x ∈ [0, π] : f (x) = n≥1 fn (x).
Ë Par parité et prolongement 2π-périodique, limx→−π+ f (x) = limx→π+ f (x) = limx→π− f (x) :
f est donc bien continue en les points kπ, k ∈ Z ce qui était le seul point douteux.
1
avec ck = > 0.
2k + 1
c) Si k ≥ q il est évident que Tq,k ≥ 0.
Maintenant, si k < q il suffit de remarquer que
k+q q−k k+q k+q
X 1 X 1 X 1 1 X 1
= + = + ≥0
j=k−q
2j + 1 j=k−q 2j + 1 j=q−k+1 2j + 1 2(q − k) + 1 j=q−k+1 2j + 1
puisque q − k + 1 ≥ 0.
d) Par décroissance de x 7→ (2x + 1)−1 sur R+ on peut écrire pour tout N ∈ N?
Z N N Z N
dt X 1 dt
≤ ≤ ,
1 2t + 1 k=0
2k + 1 0 2t + 1
soit
N
log(2N + 1) − log(3) X 1 log(2N + 1)
≤ ≤
2 k=0
2k + 1 2
N
X 1 log(N )
qui implique ∼ .
k=0
2k + 1 N →∞ 2
e) Avec (15-b) et (15-d)
2k
1 X 1 log(2k) log(k)
Tk,k = + ∼ ∼ .
2k + 1 j=0 2j + 1 k→∞ 2 k→∞ 2
Í Soit p ∈ N?
π
2 πX
Z Z
1
ap (f ) = f (t) cos(pt)dt = fn (t) cos(pt)dt
π −π π 0 n≥1
2X π
Z
= fn (t) cos(pt)dt (par NCV (question 13) sur [0, π])
π n≥1 0
2X π 1
Z t
n3
= sin 2 + 1 cos(pt)dt
π n≥1 0 n2 2
2X 1
= I n3 −1 .
π n≥1 n2 p,2
Î De l’égalité précédente et sachant que (15-c) les quantités Tp,k sont toujours positives,
on a pour tout p ≥ 1
2
ap (f ) ≥ 2
Ip,2n3 −1 , ∀ n ∈ N? .
πn
Ainsi
2X N 3 −1 2X N 3 −1
a0 (f ) a0 (f )
S2N 3 −1 (f )(0) = + an (f ) = − + an (f )
2 n=1
2 n=0
2X N 3 −1
a0 (f ) 2
≥− + I N 3 −1
2 n=0
πN 2 n,2
a0 (f ) 2
=− + T N 3 −1 ,2N 3 −1
2 πN 2 2
Ï L’équivalent obtenu en (15-e) nous donne
3 −1
2 2 log(2N ) N 3 log(2) N log(2)
T N 3 −1 N 3 −1 ∼ ∼ = .
πN 2 2 ,2 N →∞ πN 2 2 N →∞ πN 2 π
Cette dernière quantité tendant vers +∞ avec N , on a avec la question (17)
lim S2N 3 −1 (f )(0) = +∞.
N →∞
La suite (Sn (f )(0))n est donc divergente : f n’est pas développable en série de Fourier à
l’origine. o
(le terme « entre crochets » est nul car f˜0 est paire). Vu la régularité de f˜, les théorèmes
classiques nous assurent que la série de Fourier de f˜ est normalement convergente sur R avec
pour somme f˜, soit
X 1 X cn (f˜00 ) inπx
f˜(x) = cn (f )einπx = − e , ∀ x ∈ R.
n∈Z
π 2 n∈Z? n2
En particulier
1 X |cn (f˜00 )
∀ x ∈ [0, 1] |f (x)| = |f˜(x)| ≤ 2
:
π n∈Z? n2
4
soit en appliquant l’inégalité de Cauchy-Schwarz ( n≥1 n−4 = π90 )
P
1 X X 1
∀ x ∈ [0, 1] : |f (x)|2 ≤ 4 |cn (f˜00 )|2
π n∈Z? n∈Z?
n4
2 X X 1 1 X
≤ 4 |cn (f˜00 )|2 4
= |cn (f˜00 )|2
π n∈Z? n≥1
n 45 n∈Z?
f˜00 étant 2-périodique et continue par morceaux, on peut lui appliquer le théorème de Par-
seval :
1 1 ˜00 2
Z X
|f (t)| dt = |cn (f˜00 )|2 ,
2 −1 n∈Z?
soit
kf 00 k2
kf k∞ ≤ √
3 5
d’où le résultat. q
Ê Solution 1 : De l’égalité
X
az + b = (z 2 + cz + d) cn z n
n≥0
on tire
b ad − bc
c0 = , c1 = et dcn+2 + ccn+1 + cn = 0, n ∈ N
d d2
si bien que
c 1
cn+1 cn+3 − c2n+2
= cn+1 − cn+2 − cn+1 − c2n+2
d d
c 1
= − cn+1 cn+2 − c2n+2 − c2n+1
d d
c 1
= cn+2 − cn+1 − cn+2 − c2n+1
d d
1 2
= cn+2 cn − cn+1
d
...
1
= n+1 c2 c0 − c21 .
d
ê Ainsi pour c2 c0 − c1 6= 0 i.e. abc − b2 − a2 d 6= 0 :
2
cn cn+1
det
cn+1 cn+2
=d
cn+1 cn+2
det
cn+2 cn+3
x∈R:
a
un (k) (x) = (in)k einx e−n .
a
Notant || ||∞ la norme uniforme sur R, on a donc ||un (k) ||∞ = nk e−n . ParP
croissance compa-
(k) 2 (k)
rée il vient, à k fixé : ||un ||∞ = o(1/n ) quand n → +∞ de sorte que P n>1 un coverge
normalement donc uniformément sur R. Ceci étant vrai pour tout k, n>1 un est de classe
C ∞ sur R, et il en va de même de sa partie imaginaire f .
e(ix−1)
g(x) = .
1 − e(ix−1)
Montrons enfin, si 0 < a < 1, que la série :
X f (k) (0)
hk
k>0
k!
ne converge pour aucune valeur de h dans R∗ . Les arguments donnés dans a) prouvent que :
a
X
∀k ∈ N, ∀x ∈ R, f (4k+1) (x) = n4k+1 e−n cos(nx).
n>0
En particulier :
a
X
f (4k+1) (0) = n4k+1 e−n .
n>0
1/a
Soit, si k ∈ N : nk = E (4k + 1) . Alors :
a 4k+1 −(4k+1)
f (4k+1) (0) > nk 4k+1 e−nk > (4k + 1)1/a − 1 e .
D’où, vu que 1/a > 1 :
(4k)(4k+1)/a e−(4k+1)
uk = ,
(4k + 1)!
uk+1 /uk tend vers +∞ par un calcul immédiat. Il en résulte aussitôt que :
X f (4k+1) (0)
h4k+1
k>0
(4k + 1)!
ne converge que pour h = 0 puis, compte tenu de la convergence absolue d’une série entière
en tout point du disque ouvert de convergence, que :
X f (k) (0)
hk
k>0
k!
ne converge que pour h = 0.
En conclusion, f est développable en série entière au voisinage de tout point si et seulement
si a ∈]0, 1].
i Remarque : Notons E l’ensemble des suites (cn )n∈Z telles qu’il existe r dans ]0, 1[
vérifiant cn = O(r|n| ) quand n → ±∞. On peut démontrer que l’application qui à une
fonction continue sur R et 2π-périodique associe la suite (cn (f ))n∈Z de ses coefficients de
Fourier exponentiels induit une bijection de l’espace des fonctions 2π-périodiques analytiques
de R dans C sur E , la bijection réciproque associant à (cn )n∈Z la fonction :
+∞
X
x 7→ cn einx .
−∞
Avec cette caractérisation, le résultat de l’exercice est immédiat. o
Exercice
P 238 (Séries entières, comportement au bord)
Soit n≥0 an xn une série entière de rayon de convergence égal à 1. On suppose que
X
an xn = +∞.
P
∀ n ≥ 1 : an ≥ 0. Si la série n an diverge, montrer que lim
x→1−
n
!
X X
∀ A > 0, ∃ δA ∈]0, 1[ : ∀ x ∈]1 − δA , 1[, an x n > A =⇒ lim an xn = +∞.
x→1−
n n
2aπ
La fonction f définie par f (x) = eax pour tout x ∈ ]0, 2π[ et f (0) = 1+e2 , prolongée
sur R par 2π-périodicité vérifie les hypothèses du théorème de Jordan-Dirichlet : la série de
Fourier de f converge sur R vers f ; ce qui donne après un petit calcul la première formule.
Pour la seconde il suffit de considèrer la fonction g 2π-périodique, paire et égale à f sur
[0, π]. Pour la dernière, considèrer la fonction h 2π-périodique, impaire et égale à f sur ]0, π[
et vérifiant h(0) = h(π) = h(−π) = 0. Jordan-Dirichlet s’appliquant au trois fonctions, les
trois séries convergent à l’origine respectivement vers f (0), g(0), h(0). o
i Remarque : Moralité, ce n’est pas parce qu’une série trigonométrique converge vers une
fonction qu’elle est la série de Fourier de cette fonction, surtout si le domaine de convergence
est un intervalle de longueur strictement plus petite que 2π. Une condition suffisante pour
pouvoir affirmer une telle chose que l’on recontre souvent est « la convergence uniforme sur
un intervalle de longueur supérieure ou égale à 2π ».
Exercice 240 (SON et SCV ) 0n considère une suite (fn )n ⊂ C 0 ([0, 1]) vérifiant
Z 1 (
0 si n=m
fn (t)fm (t)dt =
0 1 sinon n=6 m
et
sup{|fn (x)| : x ∈ R, n ∈ N } < +∞.
Montrer que la suite (fn )n n’admet pas de sous-suite (fnk )k simplement convergente
sur [0, 1].
ê On commence par remarquer qu’on ne pert pas en généralité en supposant que vect{fn }L2 =
L2 ([0, 1]) : en effet, si ce n’est pas le cas on rajoute à la suite (fn )n une suite (gn )n vérifiant
les mêmes hypothèses mais aussi vect{fn } ∪ {gm }L2 = L2 ([0, 1]) (une telle suite (gn ) existe :
considérer pour cela la base hilbertienne .......).
ê Supposons maintenant qu’il existe une sous-suite (fnk )k et une application f ∈ C 0 ([0, 1])
telles que
Soit m ∈ N, la seconde hypothèse sur la suite (fn )n nous permet d’appliquer le théorème de
la convergence dominée :
Z 1 Z 1
0 = lim fnk (t)fm (t)dt = f (t)fm (t)dt,
k→∞ 0 0
soit
Z 1
∀m ∈ N : f (t)fm (t)dt = 0
0
et par conséquent f = 0L2 est donc nulle presque partout. Mais toujours par convergence
dominée
Z 1 Z 1
1 = lim fn2k (t)dt = f 2 (t)dt,
k→∞ 0 0
Z 2π Z 2π Z 2π
X
|n| 1 1 1
cn (f )r = Pr (t)f (t)dt ≤ |f (t)|Pr (t)dt ≤ kf k∞ Pr (t)dt = kf k∞ .
n∈Z
2π 0 2π 0 2π 0
R1 R1 R1 R1 R1R1
Exercice 242 ( 0 ( 0 f (x, y)dx)2 dy + 0 ( 0 f (x, y)dy)2 dx ≤ ( 0 0 f (x, y)dxdy)2 +
R1R1
0 0
f (x, y)2 dxdy ) Putnam (2005), [34] 2005/8.
Soit f ∈ C 0 ([0, 1] × [0, 1], R), montrer que
Z 1 Z 1 2 Z 1 Z 1 2
f (x, y)dx dy+ f (x, y)dy dx
0 0 0 0
Z 1 Z 1 2 Z 1 Z 1
≤ f (x, y)dxdy + f (x, y)2 dxdy.
0 0 0 0
Considèrons pour m, n ∈ Z
Z 1 Z 1
fb(n, m) = f (x, y)e−2iπ(nx+my) dxdy
0 0
les coefficients de Fourier de f . En particulier
Z 1Z 1
(1) f (0, 0) =
b f (x, y)dxdy.
0 0
R1
La fonction x 7→ 0 f (x, y)dy est une fonction dont la série de Fourier est n∈Z fb(n, 0)e−2iπnx .
P
De carré intégrable sur [0, 1], on peut lui appliquer la formule de Parseval
Z 1 Z 1 2 X 2
(2) f (x, y)dy dx = fb(n, 0) .
0 0 n∈Z
De même
Z 1 Z 1 2 X 2
(3) f (x, y)dx dy = f (0, m) .
b
0 0 m∈Z
Z 1 Z 1 2 Z 1 Z 1 Z 1 Z 1 2
2
f (x, y)dxdy + f (x, y) dxdy − f (x, y)dx dy
0 0 0 0 0 0
Z 1 Z 1 2 X X 2
− f (x, y)dy dx = fb(n, m) ≥ 0
0 0 n∈Z? m∈Z?
Q.E.D. o
Rπ
Exercice 243 (Calcul de 0
cos(cos(x))ch(sin(x)) cos(nx)dx, n ∈ N, via Fourier )
Tout calcul direct semble déraisonnable, on contourne cette difficulté en utilisant comme dans
l’exercice ? ? ? ? ? ? les séries de Fourier. L’application régulière f (x) = cos(cos(x))ch(sin(x))
est 2π-périodique paire, son coefficient de Fourier an vérifie donc
Z π
2 2
an = cos(cos(x))ch(sin(x)) cos(nx)dx = In .
π 0 π
Cette dernière série étant normalement convergente sur [0, 2π], c’est la série de Fourier de
f , par conséquent
k
Z π π (−1) si n = 2k
In = cos(cos(x))ch(sin(x)) cos(nx)dx = 2 (2k)!
0
0 sinon.
Q.E.D. o
Exercice 244 (Preuve du théorème des moments de Hausdorff par les séries
de Fourier )
Soit f ∈ C 0 ([0, π]) telle que
Z 2π
tn f (t)dt = 0, ∀ n ∈ N.
0
Montrer que Z 2π
eint f (t)dt = 0, ∀n ∈ Z
0
et en déduire que f ≡ 0.
Notons fe la fonction 2π-périodique paire continue sur R qui coïncide avec f sur ]0, 2π] ; ses
coefficients de fourier complexes sont donnés par
Z 2π
1
cn (fe) = f (t)eint dt, ∀n ∈ Z.
2π 0
(int)k
Mais par normale convergence sur [0, 2π] de la série entière eint =
P
k≥0 k!
on peut écrire
!
Z 2π X (int)k X (in)k Z 2π
1
cn (fe) = f (t) dt = tk f (t)dt = 0
π 0 k≥0
k! k≥0
2π(2k)! 0
vu les hypothèses sur f : les coefficients de Fourier de l’application fe continue sur [0, 2π]
et 2π-périodique sont tous nuls, comme corollaire de la formule de Parseval3 elle est donc
identiquement nulle et par suite f . Modulo translation et homothétie le résultat en découle
sur tout intervalle [a, b] de R. o
CALCUL DIFFÉRENTIEL
Ω est borné, donc Ω est compact : f continue, atteint donc son suprémum en au moint
un point α. Supposons par l’absurde que
sup f (x) = f (β) < f (α)
x∈∂Ω
(car la frontière ∂Ω est fermée bornée donc aussi compacte) et posons pour
d
X
x = t (x1 , . . . , xd ) ∈ Rd , ϕ(x) = kxk2 = x2i .
k=1
Clairement
∆ϕ(x) = 2d.
considérons maintenant pour ε > 0, la fonction définie sur Ω par
fε (x) = f (x) + εϕ(x).
la quantité
M = sup ϕ(x)
x∈Ω
soit
sup fε (x) < sup fε (x).
x∈∂Ω x∈Ω
∂ 2 fε
(x) ≤ 0
∂x2i
en contradiction avec
d
X ∂ 2 fε
∆fε (x) = (x) = ∆f (x) + 2dε = 2dε > O.
i=1
∂x2i
i Remarque : La preuve est à peine plus simple si Ω est une boule : soit pour ε > 0,
Nous avons
∆(f ) = tr(d2 f ) = 0
nous avons
tr(d2 gε (x)) = ∆(gε )(x) = 2dε > 0
si bien que d2 gε (x) admet toujours au moins une valeur propre strictement positive en chaque
point de la boule ouverte B(a, r) (gε harmonique est de classe C 2 (en fait alors C ∞ ) sa
matrice Hessienne est donc symétrique réelle et ses valeurs propres sont réelles...). Mais f
et donc gε est continue sur le compact B(a, r) : gε y admet donc un maximum x0 qui, vu ce
qui précède se trouvera nécessairement sur la frontière de B. En outre sur la frontière de la
boule εkx − ak2 = εr2 , cette quantité étant constante x0 ne dépend pas de ε i.e.
1
Ê Pour P ∈ R[X], posons uk (P ) = .
1+ k 2 P (k −1 )2
ê Si P (0) 6= 0, alors
1 1 1
uk (P ) = = ∼ (k → +∞)
1+ k 2 (P (0) + 0(k −1 ))2 1+ P 2 (0)k 2 (1 + 0(k −1 ) k→∞ P (0)2 k 2
Avec le critère de comparaison des séries à termes positifs, f (P ) est bien définie.
P
ê Supposons maintenant que P (0) = 0. P ≡ 0 la série k uk (P ) diverge grossièrement ;
sinon soit d := inf{ν : P (ν) (0) 6= 0} la valuation de P . La formule de Taylor assure que
P (d) (0) 1
P (k −1 ) = + o(k −d ) (k → ∞)
d! k d
ainsi, lorsque k tends vers +∞
(
−1
1 (1 + P 0 (0)2 ) , si d = 1,
uk (P ) = 2 −→
P (d) (0) 1, si d > 1.
1 1
1+ +o
d! k 2(d−1) k 2(d−1)
P
Et la série k uk (P ) diverge grossièrement.
ê En résumé, le domaine de définition de f est
Df = {P ∈ R[X] : P (O) 6= 0}.
C’est l’image réciproque de l’ouvert R? par l’application linéaire ϕ : R[X] 3 P 7→ ϕ(P ) =
P (0). Puisque
|ϕ(P )| = |P (0)| ≤ kP k := sup |P (t)|,
[0,1]
2k 2 P (k −1 )H(k −1 ) + k 2 H 2 (k −1 )
=−
(1 + k 2 (P + H)2 (k −1 ))(1 + k 2 P 2 (k −1 ))
2k 2 P (k −1 )H(k −1 ) + k 2 H 2 (k −1 )
=−
(1 + k 2 P 2 (k −1 )2 )
2k 2 P (k −1 )H(k −1 ) 2k 2 P (k −1 )H(k −1 )
+ −
(1 + k 2 P 2 (k −1 )) (1 + k 2 (P + H)2 (k −1 ))(1 + k 2 P 2 (k −1 ))
2k 2 P (k −1 )H(k −1 )
= Lk (H) + R avec Lk (H) = − = −2k 2 P (k −1 )H(k −1 )u2k (P )
(1 + k 2 P 2 (k −1 ))2
notre choix pour Lk (H) purement subjectif peut être motivé par exemple parce qu’il faut
d’une part chercher quelque chose de linéaire en H et, d’autre part si kHk tends vers zéro
2k 2 P (k −1 )H(k −1 ) 2k 2 P (k −1 )H(k −1 )
− ∼ − = Lk (H).
(1 + k 2 (P + H)2 (k −1 ))(1 + k 2 P 2 (k −1 )) kHk→0 (1 + k 2 P 2 (k −1 )2 )
On peut aussi motiver notre choix en utilisant sans aucune rigueur la formule classique mais
toujours d’une importance stratégique :
f (a + th) − f (a)
(f différentiable en a) =⇒ df (a)h = lim .
t→0 t
En effet, pour le cas qui nous interesse
f (P + tH) − f (P ) X uk (P + tH) − uk (P )
lim = lim
t→0 t t→0
k≥1
t
X uk (P + tH) − uk (P )
= lim
k≥1
t→0 t
X X
= −2k 2 P (k −1 )H(k −1 )u2k (P ) = Lk (H) = L(H)
k≥1 k≥1
P P
où bien entendu l’échange lim = lim n’a pas été justifié. Résumons nous : nous avons
un candidat suspect pour df (P )(H) c’est
X
(F) LP (H) = −2k 2 P (k −1 )H(k −1 )u2k (P )
k≥1
pour montrer que c’est bien lui (i.e. LP = df (P )), il va falloir successivement montrer que la
série définissant LP est bien convergente sur Df , qu’alors LP est une application (linéaire)
continue de Df dans R et qu’enfin f (P + H) − f (P ) − LP (H) = o(kHk∞ ) (ou justifier
et pour k ∈ N?
uk (P +H)−uk (P )+2k 2 P (k −1 )u2k (P )H(k −1 ) = −1 + 3k 2 P 2 (k −1 ) + 2k 2 P (k −1 )H(k −1 ) k 2 H(k −1 )u2k (P )uk (P +
donc
|uk (P +H)−uk (P )+2k 2 P (k −1 )u2k (P )H(k −1 )| ≤ k 2 u2k (P )uk (P +H) 1 + k 2 (3kP k2∞ + 2kP k∞ kHk∞ )
et comme 0 < uk (P + H) ≤ 1
|uk (P +H)−uk (P )+2k 2 P (k −1 )u2k (P )H(k −1 )| ≤ k 2 u2k (P ) 1 + k 2 uk (P + H)(3kP k2∞ + 2kP k∞ kHk∞ ) .
Mais
(4)
1 1 + k 2 (P (k −1 ) + H(k −1 ))2
2
= 2
≥ (P (k −1 ) + H(k −1 ))2 ≥ (|P (k −1 )| − |H(k −1 )|)2
k uk (P + H) k
|P (0)|
et par continuité de P à l’origine, il existe N ∈ N tel que k ≥ N implique |P (k −1 )| ≥
2
|P (0)
et comme H peut être choisi de norme arbitrairement petite, imposont kHk∞ ≤ . Alors
4
pour k ≥ N + 1
|P (0) |P (0)|
|P (k −1 ) − H(k −1 ))| ≥ − kHk∞ ≥
2 4
soit, avec (4) :
16
k 2 uk (P + H) ≤
|P (0)|2
k 2 u2k (P ))
P
par conséquent, (C = k≥1
X
|f (P + H) − f (P ) − LP (H)| ≤ |uk (P + H) − uk (P ) + 2k 2 P (k −1 )H(k −1 )u2k (P )|
k≥1
( N
!)
X X
≤ kHk2∞ C + (3kP k2∞ + 2kP k∞ kHk∞ ) k 4 u2k (P )uk (P + H) + k 4 u2k (P )uk (P + H)
k=1 k≥N +1
( N
!)
X 16
≤ kHk2∞ C + (3kP k2∞ + 2kP k∞ kHk∞ ) k 4 u2k (P )uk (P + H) + C .
k=1
|P (0)|2
Il résulte de cette dernière inégalité que
f (P + H) − f (P ) − LP (H) = o(kHk∞ )
lorsque H tends vers 0 dans (R[X], k.k) : f est donc différentiable au point P et la différen-
tielle de f au point P est la forme linéaire continue df (P ) = LP :
X 2k 2 P (k −1 )H(k −1 )
H 7→ − .
k≥1
(1 + k 2 P 2 (k −1 ))2
Exercice 248 (Extrémas en dimension plus grande que 2 : attention aux idées
reçues ! )
Ì Considérons une application f ∈ C ∞ (Rd , R). Si f admet un point critique qui est
un un extrema local (maximum ou minimum local) mais non global, doit-elle pos-
séder au moins un autre point critique ? (d’après J.M.Ash & H.Sexton « A surface
with one local minimum », Math.Mag. (19 ? ?) 58-3, pp. 147-149 and I.Rosenholtz
& L.Smylie « The only critical point in town », Math.Mag. (19 ? ?) 58-3, pp. 149-
150 ).
Ë Dans R[X1 , . . . , Xd ], (d ≥ 1) donner l’exemple d’un polynôme à coefficients
réels minoré sur Rd sans atteindre sa borne inférieure.
le théorème de Rolle assure alors de l’existence d’un réel c ∈]0, b[ tel que f 0 (c) = 0 et nous
avons bien construit un second point critique pour f .
ê Pour un contre-exemple en deux variables, considérons la fonction C ∞
−1 2 4 1
f (x, y) = + (2y − y ) 1 + , (x, y) ∈ R2 .
1 + x2 1 + x2
Pour localiser les points critiques et préciser leur nature sans trop de calculs considérons les
sections du graphe de f à x = c = constante : l’application fc (y) = f (c, y) = −a + (2y 2 −
y 4 ) (ec + a) présente (voir figure 1) trois points critiques y = 0, y = 1 et y = −1. Ainsi, les
éventuels points critiques de f se trouveront sur les sections y = 0, y = 1 et y = −1 (vu
−1
qu’ailleurs, ∂y f 6= 0). Sur y = ±1, f (x, ±1) = ex soit ∂x f = ex > 0. Sur y = 0, f (x, 0) = 1+x 2
2x
soit ∂x f (x, 0) = (1+x 2 )2 qui s’annule si, et seulement si x = 0. f admet donc un unique point
critique : (0, 0).
y
f (0, 0) = −1 > −17 = f (0, 2) : (O, O) n’est donc pas un minimum global. Il nous reste à
vérifier que (0, 0) est néanmoins un minimum local, pour cela il suffit de remarquer que
x2
2 2 1
f (x, y) = −1 + + y (2 − y ) 1 +
1 + x2 1 + x2
x2
2 2 1
= f (0, 0) + + y (2 − y ) 1 +
1 + x2 1 + x2
√
> f (0, 0) pour (x, y) ∈ D((0, 0), 2) \ {(0, 0)}.
D’où le résultat.
si bien que
inf p(x, y) = 0
(x,y)∈R2
alors que le polynôme à deux variables considéré plus haut est constant sur l’axe des ordon-
nées.
Ê Supposons n ≥ 2 et notons
A = {a ∈ Rn : df (a) 6∈ GLn (Rn )}, Ω2 = Rn \ f (A) et Ω1 = f −1 (Ω2 ).
ê Par hypothése A est fini, donc f (A) aussi et par suite Ω2 est ouvert ; f étant continue,
il en est de même pour Ω1
On vérifie facilement que Ω1 ⊂ Rn \ f (A), f (Ω1 ) ⊂ Ω2 si bien que l’on peut encore consi-
dérer la restriction f : Ω1 → Ω2 que l’on notera toujours f .
ê Avec la seconde hypothèse, l’image réciproque f −1 (K) de tout compact K dans Ω2 est
(Ω2 est ouvert, donc K est aussi un compact de Rn ) un compact de Ω1 ; par un résultat
standart1 f est fermée (ie l’image de tout fermé de Ω1 est un fermé de Ω2 ) : en particulier
f (Ω1 ) est fermé dans Ω2 .
Montrons que f (Ω1 ) est ouvert dans Ω2 : soit donc y ∈ f (Ω1 ) et x ∈ Ω1 tel que f (x) = y. Par
construction, df (x) ∈ GLn (R), étant en dimension finie c’est un homéomorphisme linéaire
de Rn dans Rn ; par le théorème d’inversion locale, il existe donc deux ouverts U, V de Rn
avec x ∈ U ⊂ Ω1 , y ∈ V tels que f induise un difféomorphisme de classe C 1 de U sur V .
Donc V = f (U ) ⊂ Ω2 et f (Ω1 ) est ouvert.
1Monier ? ?
ê Ω2 est Rn privé d’un nombre fini de points : il est2 connexe par arcs et donc connexe.
Ainsi, f (Ω1 ) est à la fois ouvert, fermé et non vide dans Ω2 connexe : f (Ω1 ) = Ω2 .
ê Il est maintenant temps de conclure : soit y ∈ Rn il y a alors deux alternatives
B ou bien y ∈ f (A),
B ou bien y ∈ Ω2 = f (Ω1 ),
dans tous les cas, y admet un antécédent : f est bien surjective.
ϕ étant continue sur R2 , un exercice classique3 nous assure que la seconde propriété est
satisfaite par ϕ.
soit (1) q
Ê et Ë sont classiques.
ce n’est rien d’autre que « la restriction » de f à la droite passant par l’origine des complexes
d’argument θ. Bien entendu, ϕθ présente en z = 0 un minimum et ce, quel que soit θ ∈ [0, 2π[
(en particulier ϕ0θ (0) = b1 eiθ = 0 =⇒ b1 = 0). En outre
ϕθ (t) = b0 + bs ts eisθ + · · · + bd td eidθ b0 + bs ts e−isθ + · · · + bd td e−idθ
Par suite
(k) (s)
ϕθ (0) = 0, si k = 1 . . . s − 1 et ϕθ (0) = 2s!re b0 bs eisθ
Ê La matrice jacobienne de f
2x −2y
Jf (a, b) =
2y 2x
de déterminant 4(x2 +y 2 ) 6= 0 sur Ω (ou bien, avec l’identification R2 ' C, f est l’application
z 7→ z 2 holomorphe sur C? et Jf (x, y) est l’écriture matricielle de la multiplication f 0 (z) = 2z)
dans tous les cas, le théorème d’inversion locale s’applique à f (qui est de classe C 1 sur Ω) :
pour tout a 6= 0 il existe deux ouvert U ⊂ Ω, V ⊂ R2 tels que f soit un C 1 difféomorphisme
de U sur V . Vu que f (z) = z 2 il existe donc sur V une determination de classe C 1 (elle est
même holomorphe, voir un cours sur le logarithme complexe) de la racine carrée.
L’inversion locale n’est pas globale (i.e. on ne peut avoir U = C \ {0} ) car f n’est pas
injective (f (z) = f (−z)), l’ouvert maximal ne pourra contenir deux points opposés dans
le plan complexe ; après un petit calcul on vérifie que le demi-plan {z ∈ C : re(z) > 0}
convient ainsi que tout demi-plan deduit de celui-ci par une rotation autour de l’origine,
l’ouvert d’arrivée étant alors C \ R− ou dans les autres cas C privé d’une demi-droite issue
de l’origine (encore une fois le lecteur connaissant le logarithme complexe ne sera pas surpris
par ces solutions).
Ê Md (C) est muni de sa structure d’espace vectoriel normé canonique (dimension finie),
l’application déterminant, polynomiale en les n2 coefficients de A est C ∞ .
Ë Pour A = ((aij ))ij ∈ Md (C), mij désigne le ij-ième cofacteur. En développant par
rapport à la i-ème ligne
d
X
det(A) = ail · mil
l=1
∂ϕ
(A) = mij .
∂aij
Et finalement, (ϕ étant différentiable)
X ∂ϕ X
dϕ(A)(H) = (A)hij = mij hij = tr(tcom(A)H).
1≤i,j≤n
∂a ij ij
Toutes ces applications sont bien sûr différentiables. On a pour A et H dans Md (C)
t
dψ(A)(H) = dB(ω(A)) ◦ dω(A)(H) = dB(A, A)(ω(H)) = dB(A, A)(H, H) = AH +tHA
q
Ê Soit δ > 0 tel que B(a, δ) ⊂ V . La formule de Taylor avec reste intégral ([41], théorème
6.3 page 279) assure que pour tout h ∈ E vérifiant [a, a + h] ⊂ U
Z 1
(1 − t)1 2
f (a + h) − f (a) − Df (a)(h) = f (a + h) − f (a) = D f (a + th)(h, h)dt
0 1!
Alors khk < δ implique [a, a + h] ⊂ V , et l’hypothèse sur D2 f assure la positivité du terme
de droite dans la formule de Taylor. Soit
f (b) ≥ f (a), ∀ b ∈ B(a, δ),
le point a est bien un minimum local de f .
Ë Il est clair que g(x) ∈ l2 (N) pour tout x ∈ l2 (N). En outre pour x, h ∈ l2 (N)
1
g(x + h) − g(x) = (2xn hn )n + (h2n )n := L(h) + Q(h, h).
2
L’inégalité immédiate |xn | ≤ kxk2 , n ∈ N implique que
kL(h)k2 ≤ kxk2 khk2 , kQ(h, h)k2 ≤ khk22 ,
ces inégalités assurent que L (resp. Q) est une application linéaire (resp. bilinéaire) continue
de l2 (N) (resp. l2 (N) × l2 (N)) dans l2 (N) : g est donc C ∞ sur l2 (N) et
Dg(x)(h) = L(h), D2 g(x)(h, h) = Q(h, h) Dk g(x)(hk ) ≡ 0, ∀k ≥ 3.
Ì et ÍIl faut commencer par noter que la série définissant f est certainement convergente
pour tout x ∈ l2 (N). Alors
X 1
f (x) = 2
xn − xn = hg(x), ω − xi, où ω = (n−1 )n ∈ l2 (N).
n≥1
n
Le produit scalaire dans l2 (N) est une application bilinaire continue (Cauchy-Schwarz) donc
C ∞ ; g étant aussi C ∞ sur l2 (N), f le sera par composition.
Î Par composition
X 2xn
Df (x)(h) = hdg(x)(h), x − ωi − hg(x), hi = − 3xn hn , x, h ∈ l2 (N).
2
n≥1
n
En particulier
Df (0l2 (N) ) = 0, ∀ h ∈ l2 (N),
et l’origine 0l2 (N) de l2 (N) est bien un point critique de f .
Ï Pour calculer D2 f (x)(h, h)
X (xn + hn )2 X 2
xn
3 3
f (x + h) − f (x) = − (xn + hn ) − − xn
n≥1
n n≥1
n
X 2xn X1 X
2
= − 3xn hn + − 3xn h2n + h3n
n≥1
n n≥1
n n≥1
X 1
= Df (x)(h) + − 3xn h2n + o(khk2l2 (N) )
n≥1
n
ÉQUATIONS DIFFÉRENTIELLES
La fonction f (x, y) = y 2 sin2 (y) est de classe C 1 sur R2 . On peut donc appliquer le théorème
de Cauchy-Lipschtiz : pour tout (x0 , y0 ) ∈ R2 , il existe une unique solution maximale ϕ de
(F) vérifiant ϕ(x0 ) = y0 et définie sur un intervalle ouvert ]a, b[.
Il y a des fonctions constantes solutions de notre équation, ce sont les fonctions y = kπ, k ∈
Z.
Considérons maintenant une solution maximale ϕ non constante définie sur un intervalle
]a, b[ ; vu ce qui précède, ϕ(]a, b[) ⊂ R \ πZ. Il existe donc k0 ∈ Z tel que ϕ(]a, b[) ⊂
]k0 π, (k0 + 1)π[ et toute solution maximale est bornée.
Supposons b < +∞, la fonction ϕ étant croissante (y 0 = y 2 sin2 (y) ≥ 0...) et majorée, ϕ
admet une limite finie en b prolongons ainsi continuement ϕ à ]a, b]. Le prolongement ψ est
continu sur ]a, b], C sur ]a, b[ et ψ (x) admet une limite égale à ψ (b) sin (ψ(b)) lorsque
1 0 2 2
x tends vers b− . Dans ces conditions, il est classique que ψ est C 1 sur ]a, b] contradisant la
maximalité de ϕ. La seule alternative est donc b = +∞. On montre de même que a = −∞.
Ê Soit (I, ϕ) une solution de (F) et a un point intérieur à I (donc a > 0). Pour
x ∈ I∩]a, +∞[ :
1 ϕ0 (t) ϕ0 (t)
∀ t ∈ [a, x] : ≤ ≤
t t + ϕ2 (t) a + ϕ2 (t)
en intégrant sur [a, x]
x
1 ϕ(x) ϕ(a) π
log ≤√ arctan √ − arctan √ ≤√
a a a a a
si bien que
π
√
∀ x ∈ I∩]a, +∞[ : x ≤ ae a .
y 0 = exp(−xy), y(0) = 0 (E )
est impaire, définie sur R et admet en +∞ une limite l ∈ [1, 1 + e−1 ].
∀ x ∈ I, x ≥ 0 =⇒ 0 ≤ f (x) ≤ x. (F)
f est donc croissante sur I = [−α, , α], majorée par α, elle admet donc une limite en α−
vérifiant
0 ≤ lim f (x) = λ ≤ α
x→α−
Il suffit maintenant de poser f (α) = λ et de vérifier (sans peine) que ce prolongement qui
est C 1 fournit une solution de (E ) sur ] − α, α] qui contredit la maximalité de f (en effet
par Cauchy-Lipschitz, un tel phénomène assure que la solution maximale est définie sur un
intervalle ouvert J contenant strictement ] − α, α] (et donc I) ce qui contredit la définition
de f ...). En conclusion, la solution maximale est bien définie sur R.
ê Soit x ≥ 1, f strictement croissante, positive sur [1, +∞[ implique
0 < f 0 (x) ≤ exp(−xf (1)).
fR 0 est donc intégrable sur [1, +∞[ et par suite f admet une limite l en +∞ (f (x) =
x 0
0
f (t)dt...). Alors
1
l ≤1+ .
e
q
La fonction M 7→ M 2 de Mn (C) dans Mn (C) est de type polynomial, donc C 1 par rapport
à M considérée comme un 2n2 -uplet de réels. Le problème posé est de Cauchy et admet une
et une seule solution maximale. Il reste à voir si cette solution est définie sur R. Calculons
cette solution.
Dans le cas du problème de Cauchy en dimension 1 : dy dt
= y 2 ; y(0) = a, on trouve aisément
y = a(1 − ta)−1 . On peut donc se demander si t 7→ A(1 − tA)−1 est solution. Posons
Φ(t) = A(I − tA)−1 .
Cette fonction Φ est définie sur un intervalle entourant 0 : le plus grand qui ne contient
aucun inverse de valeur propre de A. Les coefficients de Φ sont fonctions rationnelles de
t et Φ est donc mieux que C 1 . De plus Φ(0) = A. Enfin Φ(t)(I − tA) = A ; dérivons :
Φ0 (t)(I − tA) − Φ(t)A = 0 ; donc
Φ0 (t) = Φ(t)A(I − tA)−1 = Φ(t)2 .
La solution cherchée est bien :
t 7→ A(I − tA)−1 .
Elle est définie sur R si et seulement si A ne possède aucune valeur propre réelle non nulle.
ê
Rx
Exercice 260 (Résolution de l’équation f (x) = 1 − 0 (t + x)f (x − t)dt. ) [10],
109-(9/10).
Trouver les applications f ∈ C 0 (R, R) telles que
Z x
(8) ∀ x ∈ R, f (x) = 1 − (t + x)f (x − t)dt.
0
Le changement de variable u = x−t assure que f est de classe C 1 et légitimise une dérivation
de l’équation (8) qui devient
Z x
0
f (x) + xf (x) + 2 f (u)du = 0, & f (0) = 1.
0
Rx
La fonction F (x) = 0 f (u)du est classe C 2 et vérifie F 0 = f . Il est donc équivalent de
résoudre l’équation différentielle
(
F 00 (x) + xF 0 (x) + 2F (x) = 0
(4)
F (0) = 0 et F 0 (0) = 1.
La recherche d’un solution développable en série entière n an xn de cette dernière conduit
P
aux relations
an
an+2 = − , a0 = 1, a1 = 1
n+1
qui elles même conduisent à
(−1)n
∀ n ∈ N, a2n = 0, a2n+1 =
2n n!
soit
∞
(−1)n x2
X
2n+1
F (x) = x = x exp − .
n=0
2n n! 2
2
L’application x 7→ xe−x /2 vérifie (4) et le théorème de Cauchy-Lipschitz permet d’affirmer
que c’est la seule. Par conséquent, la seule solution continue f du problème (8) est définie
sur R par 2
0 2 x
f (x) = F (x) = (1 − x ) exp − .
2
o
ò f étant supposée continue sur R, elle y est locament intégrable ; par conséquent, on a
pour tout t > 0
Z t Z t Z t
2f (x)f (y)dy = f (x + y)dy + f (x − y)dy,
−t −t −t
Remarquons que dans (1), le second membre est une fonction dérivable de la variable x, il
en donc de même pour le terme de gauche et f est donc dérivable sur R. En réitérant ce
processus, f est deux fois dérivable sur R puis de proche en proche C ∞ . Il est donc légitime
de dériver (1) : Z t
0
2f (x) f (y)dy = 2 [f (x − t) − f (x − t)]
−t
qui donne pour x = 0
Z t
0
(2) 2f (0) f (y)dy = 2 [f (−t) − f (−t)] = 0, ∀ t > 0, .
−t
car f est une fonction paire ; mais aussi comme f (0) = 1, la continuité de f nous assure qu’il
existe t > 0 tel que
Z t
(2) f (y)dy > 0.
−t
Si dans (3), on choisit t > 0 assez petit pour que (2) soit valide, on en déduit
f 0 (0) = 0.
Dérivons maintenant deux fois (8) par rapport à la variable y, on obtient
2f (x)f 00 (y) = f 00 (x + y) + f 00 (x − y)
qui pour y = 0 donne
2f (x)f 00 (0) = 2f 00 (x), x ∈ R.
En posant C = f 00 (0), les solutions de l’équation fonctionnelle (8) sont aussi solutions de
l’équation différentielle
00
f (x) = Cf (x), x ∈ R,
(E ) f (0) = 0,
f 0 (0) = 1.
avec les conditions initiales f (0) = 1, f 0 (0) = 0, les solutions de (E ) sont finalement
f ≡ 0, f ≡ 1, f (x) = ch(cx), f(x) = cos(cx).
Réciproquement, on montre qu’elles sont bien des solutions de l’équation fonctionnelle (8).
o
ANALYSE 2
CHAPITRE 11
FONCTIONS HOLOMORPHES
Ê Il est suffisant de montrer que la suite (fk )k converge uniformément sur toute boule
fermée B(b, δ) ⊂ Ω. Comme a ∈ ∂Ω, il existe δ0 > 0 tel que
|z − a| ≥ δ0 , ∀ z ∈ B(b, δ) ;
il existe aussi une constante C > 0 telle que
sup |2n − f (z)| ≤ C.
z∈B(b,δ)
307
308/408 Petit Bestiaire d’Exercices pour l’Oral de l’Agrégation Interne Patrice Lassère
la suite (fk )k est bien uniformément convergente vers f sur tout disque B(b, δ) ⊂ Ω i.e.
fk → f dans O(Ω).
les applications fk ne sont donc pas bornées sur D(a, r) ∩ Ω : fk 6∈ En pour tout k ∈ N. Mais
comme fk → f dans O(Ω), la fonction f n’est pas intérieure à En ; f étant arbitraire, chaque
ensemble En est d’intérieur vide.
Ì Une suite (gk )k ⊂ En convergente dans O(Ω) vers une fonction g converge en particulier
simplement sur Ω et donc sur D(a, r) ∩ Ω. Ainsi, ∀ z ∈ D(a, r) ∩ Ω,
|gk (z)| ≤ n & lim gk (z) = g(z) =⇒ |g(z)| ≤ n
k
Í Vu ce qui précède,
\
O(Ω) ∩ L∞ (D(a, r)) = { f ∈ O(Ω) et bornes sur D(a, r) ∩ Ω } = En
n∈N
Î Soit f ∈ F , il existe un domaine ∆f ⊃ Ω tel que f ∈ O(Ω) ; il existe donc a ∈ ∂Ω, r > 0,
6=
tels que D(a, r) ⊂ ∆f et par suite f est bornée sur D(a, r), donc sur D(a, r) ∩ Ω : il existe
donc n ∈ N tel que f ∈ En,a := { f ∈ O(Ω) : |f (z)| ≤ n, ∀ z ∈ D(a, r) ∩ Ω }. On considère
alors une partie dénombrable dense A ⊂ ∂Ω et la suite (Dn )n des disques centrés en a ∈ A
à rayons rationnels et enfin les ensembles
Enl := { f ∈ O(Ω) : |f (z)| ≤ n, ∀ z ∈ Dl ∩ Ω } .
Vu ce qui précède, les ensembles Enp sont rares et F ⊂ n,l Enl est donc maigre.
S
O(Ω) étant un espace de baire, F maigre implique que G = O(Ω) \ F est non maigre et
partout dense.
Exercice 263 (Calcul de ζ(2) par la méthode des résidus ) Appliquer convena-
blement le théorème des résidus à laPfonction méromorphe f (z) = πz −2 cotan(πz)
pour en déduire la valeur de ζ(2) := n≥1 n12 .
Exercice 264 (Le théorème de Rolle version holomorphe) J.C. Evard & F. Jafari « A
complex Rolle’s theorem », [34], 99, 1992 ou plus simplement [2].
i Remarques : ê Le théorème de Rolle est faux pour une fonction à valeurs vectorielles :
par exemple la fonction f ∈ C ∞ (R, R2 ) (ou bien f (t) = eit ) définie par f (t) = (cos(t), sin(t))
qui vérifie f (0) = f (2π) mais sa différentielle n’est jamais nulle sur [0, 2π].
ê Pour f : C → C holomorphe ce résultat négatif à fortiori subsiste (c.f. f (z) = ez où
|f 0 (z)| = |ez | =
6 0) toutefois comme le précise cet exercice, l’holomorphie (si f est seulement
C c’est sans espoir) permet de conserver le résultat pour les parties réelles et imaginaires
∞
de la fonction. q
Autrement dit
(
ζIn − A ∈ GLn (C), ∀ |ζ| > nkAk
(ζIn − A)−1 = ∞ −(k+1) k
P
k=0 ζ A , ∀ |ζ| > nkAk.
RP PR
La normale convergence sur tout cercle C(0, r), r > nkAk assure l’échange =
ci-dessous
∞
Z Z !
1 1 X
ζ k (ζIn − A)−1 dζ = ζk ζ −(l+1) Al dζ
2iπ C(0,r) 2iπ C(0,r) l=0
∞ Z
X 1
= Al ζ k−l−1 dζ
l=0
2iπ C(0,r)
∞ Z 2π
l 1
X k−l−1 iθ
= A reiθ ire dθ
l=0
2iπ 0
∞ Z 2π
k−l l 1
X
= r A eiθ(k−l) dθ
l=0
2π 0
= Ak , ∀k ∈ N.
Ë Cette formule étant vérifiée pour tout entier k, la linéarité de l’intégrale nous assure que
Z
1
P (A) = P (ζ)(ζIn − A)−1 dζ, ∀ P ∈ C[z], r > nkAk.
2iπ C(0,r)
1 R f (u)
(remarquer l’analogie avec la formule de Cauchy f (z) = C(0,r)
du, |z| < r, ici
2iπ u−z
kAk ≤ nkAk < r....)
En particulier pour le polynôme caractéristique de A : P (z) = PA (z) = det(zIn − A) (ou
det(A − zIn ) selon l’usage)
Z Z
1 −1 1
PA (A) = PA (ζ)(ζIn − A) dζ = det(ζIn − A)(ζIn − A)−1 dζ,
2iπ C(0,r) 2iπ C(0,r)
comme
où Ci,j (ζ) est le cofacteur d’indice j, i de ζIn − A, donc un polynôme en ζ, donc d’intégrale
nulle sur tout cercle C(0, r), nous avons finalement
Z
1
PA (A) = ((Ci,j (ζ)))dζ = 0
2iπ C(0,r)
i.e.
PA (A) = 0, ∀ A ∈ Mn (C).
Le théorème de Cayley-Hamilton est bien démontré. o
Exercice 266 (Une fonction entière prenant des valeurs réelles sur deux droites
sécantes )
On suppose qu’une fonction entière non constante f ne prends que des valeurs
réelles sur deux droites sécantes du plan complexe.
Montrer que l’angle formé par ces deux droites est un multiple rationel de π.
Exercice 267 (Une fonction entière non constante mais bornée sur toute droite
passant par l’origine. ) Bak & Newman, C.Zuily ref. ? ?
On pose pour z ∈ C Z ∞ zt
e
f (z) = dt.
0 tt
Ê Montrer que f est bien définie et continue sur C.
Ë Montrer que f est holomorphe sur C et y vérifie f (z) = f (z).
Ì On désigne par log la fonction logarithme définie dans le demi-plan U = {z =
x+iy : x+y > 0} qui coïncide avec le logarithme usuel sur le demi-axe réel positif
et par w 7→ ww la fonction holomorphe sur U égale à exp(w log(w)). Soient enfin
Cr et Cε les quarts de cercles de rayon respectivement r et ε centrés à l’origine
exp(wz)
dans le premier quadrant. En intégrant la fonction w 7→ sur le contour
ww
ci-contre, montrer que
Z ∞ Z
exp(itz) exp(wz)
f (z) = π − r→∞ lim dw.
0 exp(it log(t) − t 2 ) Cr ww
Í Pour z = x + iy avec y = π2 + C où C > 0. Montrer que la première intégrale
est majorée par C −1 et en déduire que |f (z)| ≤ C −1 . Montrer que |f (z)| ≤ 1 pour
|Im(z)| ≥ π.
Î Soit g(z) = f (z − 2iπ). Montrer que g est bornée sur toute demi-droite issue de
l’origine et holomorphe sur C.
Ï Montrer que lim |g(x + 2iπ)| = +∞.
|z|→∞
bien définie sur C. Pour les mêmes raisons, on peut localement appliquer le théorème de
continuité et dérivation des intégrales à paramètres ([48], théorème 1.16.1) pour affirmer
que f est holomorphe sur C (on peut aussi invoquer ce théorème pour montrer que f est
continue sur C puis conclure avec celui de Moréra ; ou enfin encore développer f en une série
entière de rayon de convergence infini). La formule f (z) = f (z) est elle immédiate.
ε r x
Soit
Z Z Z Z Z
exp(wz)
dw = + + + = 0, ∀ 0 < ε < r.
Cε,r ww [ε,r] Cr Cε [ir,iε]
Bien évidemment
Z Z r Z r
exp(wz) exp(tz) exp(tz)
dw = dt −→ dt = f (z)
[ε,r] ww ε tt r→0
ε→0 ε tt
et
r
eitz eitz
Z Z
exp(wz)
dw = − idt −→ idt.
[ir,iε] ww ε exp it(log(t) + iπ/2) r→0
ε→0
exp it(log(t) + iπ/2)
Z Z π/2
exp(wz)
exp {ε(cos(θ) + i sin(θ))(x + iy)} iθ
dw = iεe dθ
w w exp {ε(cos(θ) + i sin(θ))(log(ε) + iθ)}
Cε 0
Z π/2
exp {ε(x cos(θ) − y sin(θ))}
≤ εdθ
0 exp {ε(log(ε) cos(θ) − θ sin(θ))}
Z π/2
≤ exp {ε cos(θ)(x − log(ε))} exp {ε sin(θ)(θ − y)} εdθ
0
Z π/2 Z 2π
Cπε
= gx,y (ε, θ)dθ = Cεdθ =
0 0 2
par continuité (ε, θ) 7→ gx,y (ε, θ) sur le compact [0, 1] × [0, 2π] (ou, au choix par convergence
dominée) ; en conséquent
Z
exp(wz)
lim dw = 0
ε→0 Cε ww
et finalement
∞
eitz
Z Z Z
exp(wz)
lim lim g(w)dw = 0 = f (z) − idt + lim dw
ε→0 r→∞ Cε,r 0 exp it(log(t) + iπ/2) r→∞ Cr ww
∞
eitz
Z Z
exp(wz)
(4) f (z) = idt − lim dw.
0 exp it(log(t) + iπ/2) r→∞ Cr ww
∞
Z ∞
eitz
Z
idt ≤ |exp(itz)| exp(tπ/2)dt
0 exp it(log(t) + iπ/2) 0
Z ∞
≤ exp {−t(y − π/2)} dt
0
Z ∞
1
≤ exp(−Ct)dt = .
0 C
Î Vu ce qui précède, la fonction entière g(z) = f (z − 2iπ) est bornée en dehors de la bande
horizontale {z ∈ C : π ≤ im(z) ≤ 3π} ; l’intersection de toute droite complexe passant par
l’origine avec cette bande étant compacte, g est bien bornée sur une telle droite et vérifie
donc la propriété désirée.
Ï On a pour x ∈ R+
∞ xt
Z
e
|g(x + 2iπ)| = |f (x)| = dt
0 tt
Z 1 xt Z 1
e
≥ t
dt = et(x−log(t)) dt
0 t 0
Z 1 tx 1
e ex − 1
≥ etx dt = =
0 x 0 x
soit
lim |g(x + 2iπ)| = +∞.
x→∞
La topologie usuelle sur l’espace O(Ω) des fonctions holomorphes sur un ouvert Ω ⊂ C
est la topologie de la convergence compacte Tc sur Ω, i.e. (c.f. [46] où exo ? ?) la topologie
engendrée par la famille de semi-normes (k.kK )K∈K (Ω) . (O(Ω), Tc ) est un espace de Fréchet
(i.e. un espace localement convexe métrisable complet) et une suite (fn )n dans O(Ω) converge
vers fi nO(Ω) si, et seulement si elle converge uniformément sur tout compact de Ω.
Toute application f ∈ O(Ω) étant localement intégrable, O(Ω) se plonge naturellement dans
L1loc (Ω) espace des classes de fonctions localement intégrable sur Ω et induit donc sur O(Ω)
la topologie T1,loc . Sur (L1loc (Ω), T1,loc ) la topologie est engendrée par les semi-normes
Z
kf k1,K := |f (z)|dxdy, K ∈ K (Ω).
K
Montrer que sur f ∈ O(Ω) les deux topologies Tc et T1,loc coïncident équivaut à montrer
que l’identitité
i : (O(Ω), Tc ) −→ (O(Ω), T1,loc )
est un isomorphisme topologique.
De toute évidence, nous avons pour tout K ∈ K (Ω)
Z
kf k1,K == |f (z)|dxdy ≤ λ(K)kf k∞,K , ∀ f ∈ O(Ω).
K
R
(où λ(K) = K dxdy est la mesure de Lebesgue de K) Cette inégalité assure la continuité de
i : (O(Ω), Tc ) −→ (O(Ω), T1,loc ), soit T1,loc ⊂ Tc .
Pour l’autre inclusion, nous aurons besoin de l’égalité de la moyenne planaire locale :
Z
1
∀ D(a, r) ⊂ Ω, ∀ f ∈ O(Ω) : f (a) = 2 f (x + iy)dxdy.
πr D(a,r)
(pour une démontration, passer en polaires dans l’intégrale et penser à la formule de Cauchy)
Montrer inclusion Tc ⊂ T1,loc équivaut à établir la continuite de i−1 : (O(Ω), T1,loc ) −→
(O(Ω), Tc ) i.e.
(8) ∀ K ∈ K (Ω), ∃ L ∈ (Ω), CK > 0 : kf k∞,K ≤ CK kf k1,L , ∀ f ∈ O(Ω).
Soit donc K ∈ K (Ω), il existe ε > 0 tel que
dist(K, ∂Ω) > 2ε.
Avec ce choix, L := K + D(0, ε) = {z ∈ Ω : dist(z, K) ≤ ε} est un compact de Ω vérifiant
∀ z ∈ K, D(z, ε) ⊂ L ⊂ Ω,
si bien qu’avec la formule de la moyenne nous avons pour tout z ∈ K et f ∈ O(Ω)
Z
1 1 1
|f (z)| = 2
f (x + iy)dxdy ≤ 2 kf k1,D(z,ε) ≤ 2 kf k1,L
πε D(z,ε) πε πε
Exercice 269 (Une fonction entière universelle ) ( C.Blair & L.A.Rubel, [34],
5-1983).
Montrer qu’il existe une fonction entière f telle que l’ensemble {f (n) , n ∈ N} de
toute les dérivées de f soit dense dans l’ensemble des fonctions entières O(C). Une
telle fonction est dire « universelle ».
d’où le résultat.
Pour achever la démonstration, il ne reste plus qu’à montrer que la suite (Kn )n peut être
choisie vérifiant (8). Pour cela, si on remarque que I(z r ) = z r+1 /(r + 1) on a
z r+k k k
≤ |z|r |z| ≤ Rr R ,
k r
I (z ) =
(r + 1) . . . (r + k) ∀ z ∈ D(0, R).
k! k!
Ainsi, pour tout r ∈ N, la suite (I k (z r ))k converge uniformément vers 0 sur tout disque
{z : |z| ≤ R} (i.e. converge vers 0 dans O(C)). Il en est de même pour I k (Pn ) (n ∈ N) comme
(d)
combinaison linéaire finie de I k (z r ) ainsi que de leur dérivées I k (z r ) par continuité de
la dérivation dans O(C). Il est donc possible de choisir Kn assez grand pour que (8) soit
réalisée. o
f + ig est donc une fonction entière sans zéros sur C, il existe donc ([48], corollaire 1-14-4)
une fonction entière h telle que f + ig = eh . Mais alors
1
f − ig = = e−h ,
f + ig
soit
eh + e−h
f= = ch(h) = cos(ih) = cos(−ih)
2
eh − e−h 1 1
g= = sh(h) = [−i sin(ih)] = − sin(ih) = sin(−ih),
2i i i
an
P
Cette inégalité assure la normale convergence et donc l’holomorphie de la série n>n0 z−n
sur D(0, R). R > 0 étant arbitraire, f est une fonction méromorphe sur C admettant pour
pôles les entiers n ∈ N? .
Soit n, k ∈ N? , pour z ∈ C(0, k + 12 ) on a
1 1
|z − n| ≥ |z| − n = |k + − n| ≥ ,
2 2
i.e.
1 1
≤ 2, ∀ z ∈ C(0, k + ), et n ∈ N?
|z − n| 2
et on a finalement X
max 1 |f (z)| ≤ 2 |an |, ∀ k ∈ N? .
|z|=k+ 2
n≥1
1G.Julia, « Leçons sur les fonctions uniformes à point singulier essentiel isolé », Gauthier-Villars (1924).
f est donc bornéee sur les cercles C(0, k + 12 ) et, en remplacant z par 1/z on obtient une
fonction méromorphe X
1 an z
g(z) = f =
z n≥1
1 − nz
admettant pour pôles les points 1/n, (n ∈ N? ) et l’origine comme point singulier essentiel.
−1
Toutefois, vu ce qui précède, son module reste borné sur les cercles de rayon rk = k + 21
ce qui nous fourni l’exemple désiré. o
i Remarque : Toutefois, malgré l’exemple ci-dessus, le comportement au voisinage
d’un point singulier essentiel et des plus chaotique : f prends chaque valeur sauf peut être
une, une infinité de fois, c’est le théorème2 de Casorati-Weierstrass.
ANALYSE FONCTIONNELLE
Exercice 272 (L2 ([0, 1]) est maigre dans L1 ([0, 1]) )
Démontrer que L2 ([0, 1]) est maigre dans L1 ([0, 1])
n R1 o
Ê En considérant les ensembles In = f ∈ L2 ([0, 1]) : 0 |f (t)|2 dt ≤ n .
Ë En considérant l’injection canonique i : L2 ([0, 1]) ,→ L1 ([0, 1]).
R1
Ì En considérant les fonctionnelles Λn : L1 ([0, 1]) 3 f 7→ Λn (t) = 0 f (t)gn (t)dt
où gn (t) = n1[0,n−3 ] (t).
i Par Cauchy-Schwarz, kf k2 ≤ kf k1 , ∀ f ∈ L2 ([0, 1]) : L2 ([0, 1]) est donc inclu dans
L1 ([0, 1]) et l’injection canonique i : L2 ([0, 1]) ,→ L1 ([0, 1]) est continue.
321
322/408 Petit Bestiaire d’Exercices pour l’Oral de l’Agrégation Interne Patrice Lassère
Ë Comme nous l’avons remarqué en préambule, l’injection canonique entre les deux espaces
de Frechets
i : (L2 ([0, 1]), k · k2 ) ,→ (L1 ([0, 1]), k · k1 )
est continue : par le théorème de l’application ouverte i(L2 ([0, 1])) = L2 ([0, 1]) est soit égal à
L1 ([0, 1]) soit maigre dans L1 ([0, 1]). Comme L2 ([0, 1]) ⊂ L1 ([0, 1]) (considérer t 7→ t−1/2 ...)
6=
L2 ([0, 1]) est bien maigre dans L1 ([0, 1]).
alors, ou bien C est maigre dans L1 ([0, 1]) ou bien C = L1 ([0, 1]). Cette dernière alternative
est exclue, pour s’en convaincre, considérons les applications fα (t) = t−α ∈ L1 ([0, 1]), (α <
1), par un calcul élémentaire
n3α−2
Λ(fα ) = −→ +∞
1 − α n→∞
pour 2/3 < α < 1. C est donc maigre dans L1 ([0, 1]). Maintenant, pour f ∈ L2 ([0, 1])
Z −3 Z n−3 !1/2 Z −3 !1/2
n n
kf k2
|Λn (f )| = nf (t)dt ≤ f 2 (t)dt n2 (t)dt ≤ √
0 CS 0 0 n
soit
∀ f ∈ L2 ([0, 1]) : lim Λn (f ) = 0.
n
Autrement dit L2 ([0, 1]) est inclu dans C maigre dans L1 ([0, 1]), il est aussi maigre dans
L1 ([0, 1]). o
Exercice 273 (Une bijection linéaire continue dont l’application réciproque est
discontinue ) [34]
On munit l’espace O(C) des fonctions entières de la norme
kf k = sup |f (z)|, f ∈ O(C).
|z|=1
À (O(C), k.k) est bien un espace vectoriel normé, la seule assertion non trivialle à savoir
kf k = 0 ⇒ f ≡ 0 résulte du théorème des zéros isolés.
Toute fonction entière f ∈ O(C) se développe en série entière sur tout le plan complexe
∞ ∞
X
n
X an
f (z) = an z donc L(f )(z) = zn, zi nC,
n=0 n=0
2n
et l’unicité d’un tel développement assure l’injectivité de l’opérateur L. En outre, pour
f ∈ O(C) P on a L(g) = fPoù g(z) = f (2z) : L est donc un isomorphisme algèbrique de O(C).
Pour f = n an z , g = n bn z n ∈ O(C)
n
∞ ∞
X an n X bn n
kL(f ) − L(g)k = sup z − z
n n
|z|=1 n=0 2 n=0
2
X ∞ z n
= sup (an − bn )
2
|z|=1 n=0
X ∞
≤ sup (an − bn )z n
|z|=1 n=0
= kf − gk.
où la première inégalité est justifiée par le principe du maximum. L est bien continue sur
O(C).
Á Soit donc f ∈ O(C), la suite (fn (z) = f (z) + (z/2)n )n converge vers f dans (O(C), k.k)
car z n
kf − fn k = sup −→ 0
|z|=1 2 n→∞
Mais n
−1 −1
2z
kL (f ) − L (fn )k = sup = 1
|z|=1 2
et par suite L−1 est discontinue au point f donc en tout point de O(C).
 (O(C), k.k) ne peut être un espace de Banach car dans ce cas nous aurions pu appliquer
le théorème de l’application ouverte à L et en déduire la continuité de L−1 .
On peut d’ailleurs vérifier directement que (O(C), k.k) n’est pas complet en considérant la
k
suite de fonctions entières de terme général fn (z) = nk=0 z2 : pour tout n, p ∈ N
P
n+p
X X
kfn+p − fn k ≤ 2−k ≤ 2−k −→ 0,
n→∞
k=n+1 k≥n+1
c’est donc une suite de Cauchy dans (O(C), k.k). Si elle converge vers f dans (O(C), k.k),
alors, vu le choix de la norme, elle sera simplement convergente sur le cercle unité vers f ,
i.e.
∞
X z k 2
f (z) = lim fn (z) = = , ∀ |z| = 1.
n→∞
k=0
2 2 − z
f serait alors une fonction entière égale (encore les zéros isolés) à la fonction holomorphe
2
f (z) = 2−z sur le disque D(0, 2) : tout ceci est absurde et (O(C), k.k) n’est pas un espace de
Banach. o
À La fonction t 7→Re−iπt f (t) est intégrable sur [0, 1] (car L2 ([0, 1]) ⊂ L1 ([0, 1]) par Cauchy-
x 1
Schwarz) donc x 7→ 0 e−iπt f (t)dt et x 7→ x e−iπt f (t)dt sont continues sur [0, 1] et par suite
R
T (f ).
Á La linéarité est évidente. Nous avons pour tout f ∈ L2 ([0, 1]) et x ∈ [0, 1]
Z x Z 1 Z x Z 1
−iπt −iπt
|T (f )(x)| ≤
e f (t)dt+
e f (t)dt ≤
|f (t)|dt+ |f (t)|dt = kf kL1 ≤ kf kL2 .
0 x 0 x
(la dernière inégalité est Cauchy-Schwarz) Ainsi
Z 1 1/2 Z 1 1/2
2
kT (f )kL2 = |T (f )(t)| dt ≤ kf k2L2 dt = kf kL2
0 0
 Soit (fn )n une suite dans la boule unité de L2 ([0, 1]) faiblement convergente vers f dans
L2 ([0, 1]). L’inégalité |T (f )(x)| ≤ kf kL2 établie dans la question précédente assure que les
formes linéaires sur L2 ([0, 1]) définies par f 7→ T (f )(x) sont continues ; la suite (fn )n étant
faiblement convergente vers f , nous avons donc limn T (fn )(x) = T (f )(x) pour tout x ∈ [0, 1]
d’où la simple convergence de (T (fn ))n vers f sur [0, 1].
Il en résulte que limn |T (fn )(x) − T (f )(x)|2 = 0 sur [0, 1] et |T (f )(x)| = limn |T (fn )(x)| ≤
limn kfn kL2 ≤ 1. Par conséquent nous avons la domination
|T (fn )(x) − T (f )(x)|2 ≤ 4 ∈ L1 ([0, 1])
et
lim |T (fn )(x) − T (f )(x)|2 = 0 sur [0, 1].
n
On peut donc appliquer le théorème de la convergence dominée
Z 1 1/2 Z 1 1/2
2 2
lim kT (fn )−T (f )kL2 = lim |T (fn )(x) − T (f )(x)| dx = lim |T (fn )(x) − T (f )(x)| dx =0
n n 0 0 n
à Il faut montrer que toute suite (gn )n dans l’image par T de la boule unité de L2 ([0, 1])
admet une sous-suite convergente dans L2 ([0, 1]). Soit donc une telle suite, il existe une suite
(fn )n dans la boule unité de L2 ([0, 1]) telle que gn = T (fn ), ∀ n ∈ N. La boule unité de
L2 ([0, 1]) étant ([46], Théorème ? ? page ? ?) faiblement compacte, elle admet une sous-suite
(fnk )n faiblement convergente vers f dans L2 ([0, 1]). Avec la question précédente, gnk =
T (fnk ) converge vers T (f ) dans L2 ([0, 1]).
Ä D’aprés la première question T (L2 ([0, 1])) ⊂ C 0 ([0, 1]) ⊂ L2 ([0, 1]), T n’est pas surjectif :
6=
0 est bien une valeur spectrale.
Les deux applications de Fubini sont justifiées car F (x, t) = ieiπx−iπt k(x, t)f (t)g(x) ∈ L2 ([0, 1]×
[0, 1]). T est donc bien hermitien et ses valeurs propres sont alors nécessairement réelles
(l’écrire).
Æ ê Si f est continue sur [0, 1] il est clair que T (f ) ∈ C 1 ([0, 1]). On peut donc écrire
T (f )0 (x) = iπT (f )(x) + ieiπx e−iπx f (x) + e−iπx f (x) = iπT (f )(x) + 2if (x).
ê Soit λ une valeur propre non nulle de T et fλ un vecteur propre associé. T (fλ ) = λ · fλ ,
donc fλ = λ−1 T (fλ ) ∈ C 0 ([0, 1]) : fλ est donc continue et par suite T (fλ )C 1 ([0, 1]). Les
sous-espaces propres de T (si T admet des valeurs propres seront donc toujours inclus dans
C 1 ([0, 1]). On a donc avec l’égalité ci-dessus
soit
fλ0 = i 2λ−1 + π fλ ,
Z 1 Z 1
−iπt
T (f )(0) = −ie iπ0
e f (t)dt = ie iπ
e−iπt f (t)dt = T (f )(1)
0 0
soit fλ (0) = fλ (1) qui implique avec (8) que eiω = 1. Sachant que les valeurs propres de T
sont réelles, on a finalement
1
ω = 2kπ, k ∈ Z, ⇐⇒ λ= := λk , k ∈ Z.
π(k − 1/2)
Réciproquement un calcul élémentaire montre que les applications fλk sont bien des vecteurs
propres de T associées aux valeurs propres λk .
ê Le spectre de T est donc l’adhérence de la famille {λk , k ∈ Z} soit σ(T ) = {0}∪{λk , k ∈
Z}. o
Rx
Ì Soit f ∈ Lp ([0, 1]) et n ≥ 1, la continuité de x 7→ 0 |f (t)|p dt et le théorème des valeurs
intermédiaires assurent l’existence d’une partition x0 = 0 < x1 < · · · < xn = 1 de [0, 1]
vérifiant
Z xi
δ(f )
|f (t)|p dt = , ∀1≤i≤n
xi−1 n
R1
où δ(f ) = 0 |f (t)|p dt. Ainsi fi := nf χ[xi−1 ,xi ] ∈ Lp ([0, 1]) et f = n1 (f1 + · · · + fn ). De plus par
construction δ(fi ) = np−1 δ(f ) ; il suffit alors de choisir n assez grand pour que np−1 δ(f ) ≤ ε.
Í Soit V un voisinage convexe de l’origine dans Lp ([0, 1]). Il existe ε > 0 tel que B(0, ε) ⊂
V . Soit f ∈ Lp ([0, 1]), vu la question précédente, il existe f1 , . . . , fn ∈ Lp ([0, 1]) vérifiant
δ(fi ) < ε i.e. fi ∈ B(0, ε) ⊂ V et finalement f ∈ V puisque V est convexe. Lp ([0, 1]) est
donc le seul voisinage convexe de l’origine. Lp ([0, 1]) est bien le seul voisinage convexe de
l’origine, ce n’est n’est donc pas un espace de Fréchet. Soit ϕ ∈ Lp ([0, 1])0 et B une base de
voisinages convexes de 0K , ∀B ∈ B : ϕ−1 (B) est un voisinage ouvert convexe de l’origine
dans Lp ([0, 1]) i.e. ϕ−1 (B) = Lp ([0, 1]) soit ∀B ∈ B : ϕ(Lp ([0, 1])) ⊂ B =⇒ ϕ ≡ 0, soit
Lp ([0, 1])0 = {0}. (remarquer que le même raisonnement vaut pour Lc (Lp ([0, 1]), E) où E
est un evtlc...). q
C 0 ([0, 1]) est muni de la topologie de la convergence uniforme sur [0, 1]. Avec un corollaire
du théorème de Hahn-Banach [46], il suffit de montrer que toute forme linéaire continue
T ∈ C 0 ([0, 1])0 nulle sur V est identiquement nulle. Considérons donc une telle forme T .
Posons pour k ∈ N, gk (x) = xk alors pour tout entier n ∈ N la série de fonction ∞ gk (x)
P
k=0 akn
est normalement convergente sur [0, 1] vers −an fan . Par continuité de T sur C 0 ([0, 1]) nous
avons
∞
! ∞
X gk (x) X T (gk )
0 = T (−an fn ) = T k
= k
,
k=0
a n k=0
a n
soit
gk (x)
(1) ∀n ∈ N : = 0.
akn
possède un rayon de convergence supérieur ou égal à 1 : f donc est holomorphe sur le disque
D(0, 1). Mais vu (1)
∞
!
X T (gk )
f a−1
k
= 0, ∀ n ∈ N ⇐⇒ n = 0, ∀ n ∈ N .
k=0
an
Or, vu les hypothèses, la suite (a−1n )n ⊂ D(0, 1) converge vers 0 : par le théorème des zéros
isolés, f ≡ 0 ce qui entraine T (gk ) = 0, ∀ k ∈ N et donc T ≡ 0 sur R[X]. Ainsi, toute forme
linéaire continue nulle sur V est nulle sur R[X] ; par transitivité de la densité et le théorème
de Weierstrass une telle forme est donc identiquement nulle. C.Q.F.D. q
On vérifie facilement que N est une norme sur E qui est donc équivalente à k.k : il existe en
particulier une constante c > 0 telle que
donc
(|ϕ(x)| ≤ (1 − c)kxk, ∀ x ∈ E) =⇒ (ϕ ∈ E 0 ) .
(toutes les normes sur E étant équivalentes le dual topologique ne dépend pas du choix de
la norme) d’où le résultat.
Ë Vu la question précédente, il suffit de montrer que sur tout espace vectoriel de dimension
infinie, il existe toujours des formes linéaires non continues. Soit E un tel espace et {ei }i∈I une
base de E (il en existe toujours une avec l’axiome du choix). I est infini, et soit (in )n∈N ⊂ I
une partie stricte dénombrable. Pour une norme k.k sur E la forme linéaire définie sur {ei }i∈I
(et donc sur E) par
(
nkein k si i = in ,
∀ i ∈ I : f (ei ) =
0 sinon.
est visiblement discontinue sur E. Ainsi, sur tout espace vectoriel normé de dimension infinie
on peut construire une forme linéaire non continue, et par suite un espace vectoriel sur lequel
toutes les normes sont équivalentes est forcément de dimension finie. q
Soit f ∈ L1 (R), la clef est de remarquer que si F (f ) est impaire il existe une constante
C > 0 telle que
F
Z b
(f )(t)
≤ C, ∀ 1 < b < ∞.
dt
t
1
ceci résulte des deux faits élémentaires suivants :
Z β
sin(t)
∃C > 0 : dt ≤ C, ∀ α < β
α t
Z
F (f )(x) = −i f (t) sin(xt)dt.
R
et du théorème de Fubini.
Ainsi, pour exhiber un élément g ∈ C0 (R) \ L1 (R) il suffit de construire une fonction impaire
Rb
g ∈ C0 (R) telle que 1 g(t)
t
dt soit non borné lorsque b tends vers l’infini. Par exemple une
fonction continue impaire égale à 1/ log(x) pour x > 2 convient. q
La formule d’inversion de Fourier dans L2 (R) implique que F (f ) = g qui est dans L1 (R)
comme désiré ; on vérifie facilement que f 6∈ L1 (R) ou plus simplement que F (f ) = g n’est
pas continue. q
Z 1
Rn (x) = (λn − m)x λn
Rn−1 (t)t−1−λn dt
x
n−1
!
Z 1 X
= (λn − m)xλn tm − an−1,k tλk t−1−λn dt
x k=0
n−1
!
Z 1 X
= (λn − m)xλn tm−1−λn − an−1,k tλk −1−λn dt
x k=0
n
( Pn−1 m−λn
X an,n = 1 − k=0 a
λk −λn n−1,k
,
= xm − an,k xλk avec m−λn
k=0
an,k = λk −λn an−1,k , 0 ≤ k ≤ n − 1.
D’ou le résultat.
Ë Pour x ≥ 1 et x ∈ [0, 1]
Z 1
|Rn (x)| ≤ |λn − m| x λn
kRn−1 k∞ t−1−λm dt
x
|λn − m|
≤ kRn−1 k∞ xλn (−1 − x−λn )
λn
m
≤ 1 − kRn−1 k∞
λn
soit
n n
m Y m Y m
kRn k∞ ≤ 1 − kRn−1 k∞ ≤
1 − λk kR0 k∞ =
1 − λk .
λn k=1 k=1
Í Vu la question précédente, R[X] est inclus dans l’adhérence de E pour la norme « sup » ;
le résultat suit avec Weierstrass et la transitivité de la densité. q
une autre consiste à commencer par établir le résultat dans L2 ([0, 1]) qui a le bon gout de
posséder une structure hilbertienne (Chamber-Loir et Fermigier) le passage aux fonctions
continues ne pose alors pas de problèmes.
ê Il est intéressant de remarquer qu’à ce jour, on ne connait pas de démonstration de ce
résultat qui ne s’appuie pas sur le théorème de Weierstrass.
ê Il existe de nombreuses généralisations de ce théorème, on pourra consulter à ce sujet le
remarquable ouvrage de J.Borwein et T.Eyderly Polynomials inequalities, Springer 19 ? ?. ref. !
Exercice 281 (Sous-espaces de C ([0, 1]) fermés dans L2 ([0, 1]) ) ([10], 2003/04 et
[34]- ? ? ? ?
Ê Si F = E, alors
(F) ∃C > 0 : ∀f ∈ E : kf k∞ ≤ Ckf k2 .
Il suffit alors de considérer la suite de fonctions continues (fn )n≥1 définies par
(
0 si x ∈ [0, 1 − n1 ],
fn (x) = √ 2
n x + n − n2 pour x ∈]1 − n1 , 1]
en effet, vu ce choix √
kfn k∞ = n et kfn k2 = 1, n ≥ 1
rendant (F) absurde.
N N N N
!1/2
X X X X
ai fi (x) ≤ k ai fi k∞ ≤ Ck ai fi k2 = C a2i .
i=1 i=1 i=1 i=1
En particulier, x ∈ [0, 1] étant fixé, le choix ai = fi (x), 1 ≤ i ≤ N nous donne
N N
!1/2
X X
fi2 (x) ≤ C fi2 (x)
i=1 i=1
i.e.
N
X
fi2 (x) ≤ C 2 pour tout x ∈ [0, 1]
i=1
et en intégrant cette inégalité sur [0, 1], la famille (fi )N
1 étant orthonormale
Z N
1X N
X
fi2 (t)dt = kfi k22 = N ≤ C 2
0 i=1 i=1
√
ainsi N ≤ √ C et ceci pour tout N ≥ 1 : contradiction ! F est donc de dimension finie et
dim(F ) ≤ N .
N
! 21 N
! 21
X X
≤ sup a2i fi2 (t) (par Cauchy-Schwarz)
t∈[0,1] i=1 i=1
N
! 21 N
! 21
X X
≤ a2i sup fi2 (t)
i=1 t∈[0,1] i=1
N
! 21
X
= kf kL2 sup fi2 (t) (voir la seconde question)
t∈[0,1] i=1
√
≤ N kf kL2
la dernière inégalité résultant du fait que par construction, les support des fi sont deux à
deux disjoints et donc pour tout t ∈ [0, 1], il existe un 1 ≤ i0 ≤ N vérifiant t ∈ (xi0 −1 , xi0 ) la
somme est réduite à un terme, si bien que :
N
!1/2
X √
fi2 (t) = |fi0 (t)| ≤ kfi0 k = N .
i=1
On a donc
√
∀f ∈ F : kf k∞ ≤
N kf kL2
√
et le résultat de la seconde question montre bien que N est la meilleure constante possible
(il est d’ailleurs encore plus facile de le vérifier en prenant f = fi ...) : F est donc bien
l’exemple désiré. √
Remarquons aussi que N est la norme de l’application linéaire continue canonique (l’iden-
tité)
id : (F, k.k∞ ) → (F, k.kL2 )
q
i Quelques remarques : ê La première est, en utilisant des outils un peu plus sophis-
tiqués (niveau maitrise), que cette propriété est caractéristique des sous espaces de fonctions
continues fermés dans (L2 ([0, 1]), h, i2 ), plus précisément :
∀ n, p ∈ N : kfn − fp k∞ ≤ Ckfn − fp k2
i.e. (fn )n est aussi de Cauchy pour la norme infinie : elle converge donc uniformément sur
[0, 1] vers une fonction g ∈ C 0 ([0, 1]). Par Cauchy-Schwarz
kfn − gk2 ≤ kfn − gk∞ → 0 (n → +∞)
i.e. (fn )n vers g pour la norme L2 et par unicité de la limite, f = g dans L2 ([0, 1]) puis
sur [0, 1] par continuité des deux fonctions. Il reste alors à faire tendre n vers +∞ dans
L2
kfn k∞ ≤ Ckfn k2 pour obtenir kf k∞ ≤ Ckf k2 et (8) est donc encore vraie sur F .
ê Donnons une autre preuve (toujours avec les outils d’analyse fonctionnelle) que F est
nécessairement de dimension finie.
Pour cela, on considère B := {f ∈ F : kf k∞ ≤ 1} la boule unité de F pour la norme
infinie. On va montrer que B est compacte dans (F, k.k2 ) (cet espace étant normé, il suffit
de montrer que de toute suite de B, on peut extraire une sous-suite qui converge pour la
norme L2 vers une limite appartenant à B).
Les deux topologies coïncidant sur F , B est bornée dans l’espace de Hilbert (F, k.k2 ) donc
faiblement compacte : on peut donc, de toute suite (gn ) ⊂ B extraire une sous-suite (gnk )k
L2 -faiblement convergente. Mais dans l’Hilbert (F, k.k2 ) les masses de Dirac δx : f ∈ F 7→
δx (f ) := f (x) sont des formes linéaires continues (pas sur (L2 ([0, 1]), k.k2 ) bien entendu !) ; et
par suite pour tout x ∈ [0, 1] la suite (δx (gnk ))k = (gnk (x))k converge, i.e. la suite (gnk )k est
simplement convergente sur [0, 1]. Notons g sa limite, on montre facilement que kgnk −gk2∞ ≤
4. On peut ainsi appliquer le théorème de la convergence dominée : |gnk − g|2 → 0 dans L1 ,
i.e. limk gnk = g dans L2 ([0, 1]) et B est séquentiellement compact dans (F, kk2 ) donc dans
(F, k.k∞ ) : la boule unité de (F, k.k∞ ) est compacte : par le théorème de Riesz dim(F ) < +∞.
CQFD
ê Avec un petit effort supplémentaire on peut montrer que le résultat ne subsiste plus si
on remplace
« sous-espace de fonctions continues sur [0, 1] dans L2 ([0, 1]) »
par
« sous-espace de fonctions continues sur ]0, 1] dans L2 ([0, 1]) »
En
1
considérons pour n ≥ 1 des applications fn : [0, 1] → R continues, à support
1 effet, 2
dans
,
n+1 n
vérifiant kfn k2 = 1. C’est visiblement une famille orthonormale dans L ( [0, 1]).
Avec la formule de Parseval, on montre facilement que l’application
X
L : α = (αi )i≥1 ∈ l2 (N) 7→ L(α) := αi fi ∈ L2 ( [0, 1])
i≥1
est une isométrie. En outre, la somme étant localement finie pour tout x ∈]0, 1]
L(α) ∈ C 0 (]0, 1]) pour tout α ∈ l2 (N).
En conclusion, L (l2 (N)) est un sous-espace de fonctions continues (en prenant des fonctions
affines par morceaux, on se persuade rapidement qu’il est facile de fabriquer dans L(l2 (N))
des fonctions discontinues à l’origine), de dimension infinie, fermé (car isométrique à l2 (N))
dans L2 ( [0, 1]). Bref, le contre-exemple désiré.
ker(T 2 ) = ker(D),
mais ker(D) est l’ensemble des fonctions constantes, il est donc de dimension 1 et par suite
dim ker(T 2 ) = 1 et T n’est pas injectif, donc
ker(T p ) = ker(T ), ∀p ∈ N?
et en particulier
Ë Supposons que (l∞ (N))0 = l1 (N) avec la question précédente, il existerai α = (αn )n ∈
l1 (N) tel que X
L(u) = αn un , ∀ n ∈ N.
n≥0
Mais en appliquant cette formule aux suite δk = (δnk )n ∈ l∞ (N) on obtient αk = 0, ∀ k ∈ N,
soit L ≡ 0 ce qui est absurde. o
ò Soient ε > 0, g ∈ L2 (R). Par densité4 de C0∞ (R) dans L2 (R), il existe fε , gε ∈ C0∞ (R)
vérifiant
kf − fε k2 ≤ ε, kg − gε k2 ≤ ε.
On peut alors écrire
Z Z
gn (t)g(t)dt = f (t − n)g(t)dt
R
ZR Z
≤ g(t) (f (t − n) − fε (t − n)) dt + g(t) − gε (t) fε (t − n)dt
R R
Z
+ gε (t)fε (t − n)dt
R
Z
≤ kgk2 · kf − fε k2 + kg − gε k2 · kfε k2 + gε (t)fε (t − n)dt .
R
Pour obtenir la dernière inégalité, nous avons appliqué deux fois l’inégalité de RCauchy-
Schwarz ainsi que l’invariance par translation de la mesure de Lebesgue pour l’égalité R |fε (t−
n)|2 dt = kfε k22 .
Maintenant on peut choisir n ∈ N assez grand (disons n ≥ nε ) pour que l’intersection des
supports de gε et t 7→ fε (t − n) soit vide (c’est classique : le support de gε est compact,
donc inclu dans un intervalle [a, b] et celui de fε dans un intervalle [c, d] ; alors, celui de
4Donner une référence, Wagschall...
ò Pour la seconde suite la procèdure est identique et inutile à détailler, le dernier terme
dans l’inégalité tendant vers 0 avec n aprés un calcul direct :
Z Z
gε (t) √1 fε t dt ≤ √1 kfε k∞
|gε (t)|dt −→ 0.
R n n n R n→∞
ò Pour la troisième, on se ramène à la première via la tranformée de Fourier qui est une
isométrie de L2 (R) :
hg, kn i = hb
g , kbn i,
et la formule élémentaire
kn (t) = fb(t − n).
b
o
Ê Il s’agit donc de montrer que pour tout P ∈ R[x], l’application coordonnée BP : R[x] →
R définie par BP (Q) = B(P, Q) est continue (la continuité de l’autre application coordonnée
P 7→ B(P, Q) se déduit par symétrie). BP est clairement linéaire et on a pour tout Q ∈ R[x]
Z 1 Z 1
|BP (Q)| = P (t)Q(t)dt ≤ kP k∞ |Q(t)|dt = kP k∞ · kQk1 .
0 0
Cette inégalité assure la continuité de BP (et montre aussi que la norme de BP est inférieure
ou égale à kP k∞ ).
B est donc bien discontinue à l’origine de R[x] × R[x] et donc partout par bilinéarité. o
Ê Munissons l’espace produit RN des suites réelles de cette topologie et considérons l’en-
semble des suites bornées L∞ (N) ⊂ RN . Une suite (xk )k ∈ L∞ (N) si, et seulement si, il existe
un entier n tel que (xk )k ∈] − n, n[N ; soit
[
L∞ (N) = ] − n, n[N .
n≥1
L∞ (N) est donc ouvert dans RN comme produit d’ouverts de R. Mais on a aussi
(xk )k ∈ L∞ (N) ⇐⇒ ∀ n ∈ N, ∃ k ∈ N : |xk | ≥ n,
soit encore \ Y \
L∞ (N) = Xkn := Fn
n≥1 k≥1 n≥1
où (
Y [n, +∞[, si k = n,
Fn = Xkn avec Xkn =
k≥1
R, si k 6= n.
Comme produit de fermés les ensembles Fk sont fermés et par suite L∞ (N) = n≥1 Fk est
T
fermé comme intersection de fermés : c’est une partie non vide distincte de RN qui est à la
fois ouverte et fermée, RN n’est pas connexe bien qu’étant le produit des connexes R.
Ë Considérons maintenant le produit d’espaces compacts [0, 1]N toujours bien entendu
équipé de la topologie définie dans l’énonçé et montrons que [0, 1]N n’est pas compact.
Pour cela, nous allons construire un recouvrement ouvert de [0, 1]N qui n’admet pas de
sous-recouvrement fini : soient A0 = [0, 1[, A1 =]0, 1]. Ce sont deux ouverts de [0, 1] et par
conséquent les ensembles de la forme
Y
Aεn , où (εn )n ∈ {0, 1}N
n≥1
Y 1 ! \
−1 1 1 1
f − + a, a + = − + a, a + = {a}
n≥1
n n n≥1
n n
qui n’est pas ouvert dans R : f est donc discontinue sur R.
On peut tout de même remarquer que les applications coordonnées sont elles bien continues
(tout le monde aura noté que la topologie produit TP est inclue dans T).
Q
Í Supposons au contraire, qu’il existe une application continue f : X → a∈A Xa et un
point x ∈ X tels que pour une infinité d’indices a ∈ A l’application coordonnée fa : X → Xa
soit non constante au point x. On peut alors construire dans A, une suite infinie d’indices
a1 , a2 , . . . et une suite de réels strictement positifs ε1 , ε2 , . . . telles que
∀ i ∈ N? , ∃ xi ∈ BX (x, 1/i) tel que dXai (fai (xi ), fai (x)) > εi .
Il en résulte immédiatement que l’image réciproque par f du voisinage ouvert de f (x)
!
Y Y
BXai (fai (x), εi ) × Xa
i≥1 a∈A\{aj , j≥1}
ne contiendra aucune boule BX (x, 1/j), (j ≥ 1), et ne peut par conséquent être un voisinage
de x : f est donc discontinue au point x, d’où la contradiction.
Réciproquement, considérons un point x ∈ X admettant un voisinage Ux sur lequel les
applications coordonnées Q fa , (a ∈ A) sont constantes sauf pour un nombre fini d’indices
a1 , . . . , an . Soit V = a∈A Va ∈ T un voisinage élémentaire de f (x) alors
U := fa−1
1
(Va1 ) ∩ fa−1
2
(Va2 ) ∩ . . . fa−1
n
(Van ) ∩ Ux
est un voisinage de x vérifiant f (U ) ⊂ V : f est bien continue au point x (l’image réciproque
par f de tout voisinage de f (x) est un voisinage de x....).
Î C’est un corollaire immédiat de la question précédente. o
chez les étudiants qui ont tendance à la confondre avec T (en dimension finie, ces deux
topologies bien entendu coïncident) ; l’exercice ci-dessus à pour objectif de leur montrer que
TP est la bonne topologie sur l’espace produit car il est bien connu ([46], page ? ?) que
pour la topologie produit, un produit de connexe est connexe, unQproduit de compact est
compact (théorème de Tychonov) et une application f : X → a∈A Xa est continue si,
et seulement si chaque application coordonnée fa = πa (f ) : X → Xa est continue ce qui
n’est pas le cas pour T . Moralité : plus vous avez d’ouverts moins vous avez de compacts,
de connexes et de fonctions continues. Vu sa définition, la topologie produit est la topologie
la plus économique (la moins fine ou initiale)Q i.e. possédant les moins d’ouverts, rendant
continues les applications coordonnées πb : a∈A Xa → Xb , (b ∈ A).
ò Désignons par (Qi )i∈N la famille (dénombrable) de tous les pavés ouverts de Ω de la
forme Q =]a1 , b1 [× . . . ]an , bn [⊂ Ω avec ai , bi ∈ Q et soit E le Q-espace vectoriel engendré par
la famille des fonctions indicatrices (χQi )i . E est une partie dénombrable de Lp (Ω) et on va
montrer que E est dense Lp (Ω).
Soit f ∈ Lp (Ω). L’espace Cc (Ω) des fonctions continues à support compact dans Ω est
notoirement dense dans Lp (Ω) ([47], ? ? ?) ; étant donné ε > 0, il existe donc gε ∈ Cc (Ω)
vérifiant kf −gε kLp ≤ ε. Considérons maintenant un ouvert borné Ω0 vérifiant supp(g) ⊂ Ω0 ⊂
Ω. Il n’est alors pas difficile de construire une application h ∈ Cc (Ω) vérifiant supp(h) ⊂ Ω0
et |h(x) − g(x)| ≤ ε/λ(Ω)1/p , ∀ x ∈ Ω0 (utiliser la continuité uniforme de g pour recouvrir le
support de g par un nombre fini de pavés sur lesquels l’oscillation de g est | ≤ ε/λ(Ω)1/p ...).
Avec ce choix kg − hkLp ≤ ε et finalement kg − hkLp ≤ ε.
densité de (fn )n l’application Ω 3 a 7→ n(a) ∈ N est bien définie et les ouverts Oa étant deux
à deux disjoints, cette correspondance est bijective d’où la contradiction). o
ò Posons pour tout j ∈ N : Ej := (δij )j ∈ lp (N) (où δij est le symbôle de Kronecker). Soit
X = (xi )i ∈ lp (N), alors puisque p < ∞ on a
n ∞
!1/p
X X
kX − xi Ei kp = |xi |p −→ 0
n→∞
i=0 i=n+1
comme reste d’une série convergente. La famille (Ej )j est donc totale dans lp (N) qui est donc
bien (considérer comme toujours vectQ {Ej , ji nN}) séparable.
ò Montrons par l’absurde5 que l∞ (N) n’est pas séparable, supposons donc qu’il existe dans
l∞ (N) une suite dense (Xn )n . Soient A ∈ P(N), χA : N → {0, 1} la fonction indicatrice
de A et EA := (χA (i))i ∈ l∞ (N). Par densité de (Xn )n dans l∞ (N) il existe n = n(A) ∈ N
tel que kXn − EA k∞ < 1/2 et désignons par nA le plus petit entier vérifiant cette propriété.
Nous venons de construire une application φ : P(N) 3 A 7→ φ(A) = nA et cette application
est injective car si n = nA = nB
( kEn − EA k∞ < 1/2 & kEn − EA k∞ < 1/2) =⇒ (kEA − EB k∞ < 1)
qui implique immédiatement EA = EB soit φ(A) = nA = nB = φ(B) : φ est bien injective.
En résumé, la séparabilité de l∞ (N) permet de construire une injection de P(N) dans N ce
qui est absurde (Bernstein), d’où le résultat.
On équipe C 0 ([0, 1]) de la norme « sup », c’est un espace de Banach. Soit X l’adhérence
de vect{ fn , n ∈ N} dans C 0 ([0, 1]). On suppose par l’absurde que X ⊂ C 1 ([0, 1]) ; par un
théorème de Weierstrass de L2, le graphe de l’opérateur de dérivation entre les espaces de
5Avez vous remarqué pourquoi la preuve précédente ne marche plus si p = ∞ ?
Banach X et C 0 ([0, 1]) est fermé : il est donc continu par le théorème du graphe fermé.
Mais dans ce cas (fn )n converge uniformément vers 0 sur [0, 1] implique que (fn0 )n converge
uniformément vers 0 sur [0, 1] ce qui est contraire à l’hypothèse. o
Exercice 293 (Une permutation qui conserve les séries convergentes ) [34]
1-2006.
P l’espace vectoriel S des suites a = (an )n de nombres réels telles que
On considère
la série ∞ 0 an converge.
P∞ π : N → N une permutation telle que pour tout a = (ak )0 ∈ S la série
∞
Soit
k=0 aπ(k) converge. On va montrer que
∞
X ∞
X
aπ(k) = ak , ∀a ∈ S.
k=0 k=0
( n )
X
Ê Montrer que S , muni de la norme kak = sup ak est un espace de
n∈N
k=0
Banach.
Ë Montrer que les formes linéaires sur S ci-dessous sont continues
Xn
Un : S 3 a 7→ Un (a) = ak ,
k=0
Tn : S 3 a 7→ Tn (a) = an ,
X∞
T : S 3 a 7→ T (a) = ak .
k=0
X∞
N (a) = sup ak .
n≥0
k=0
n
X
Un : S 3 a 7→ Un (a) = ak ,
k=0
Tn : S 3 a 7→ Tn (a) = an ,
X∞
T : S 3 a 7→ T (a) = ak .
k=0
sont continues. Les inégalités |Un (a)| ≤ N (a) assurent la continuité des Un , comme T0 =
U0 , Tn = Un − Un−1 , (n ≥ 1) les formes Tn sont aussi continues ; enfin T est continue puisque
par exemple |T (a)| ≤ 2N (a) (on peut aussi invoquer le théorème de Banach-Steinhaus
puisque T (a) = limn Un (a), ∀ a ∈ S ).
mais
Uk (Tπ (an )) = anπ(k) , on a donc aussi lim Uk (Tπ (an )) = lim anπ(k) = aπ(k)
n k
Í P précède T ◦Tπ −T est une forme linéaire continue sur S avec (T ◦Tπ −T )(a) =
P Vu ce qui
k aπ(k) − k ak . Il ne reste plus qu’à remarquer que pour toute suite a ∈ S nulle à partir
d’un certain rang (T ◦ Tπ − T )(a) = 0 et comme l’ensemble de ces suites est dense dans S
par continuité T ◦ Tπ − T ≡ 0 i.e.
∞
X ∞
X
aπ(k) = ak , ∀a ∈ S.
k=0 k=0
C.Q.F.D. o
Il est clair que limn fn = f dans (C ([0, 1]), T ) si et seulement si, (fn )n converge simple-
ment vers f sur [0, 1].
Pour montrer que la fonction nulle f = 0 ∈ A il faut montrer que tout voisinage de f
VF,ε = { g ∈ C ([0, 1]) : |g(x)| < ε, ∀ x ∈ F } ,
rencontre A ce qui est évident car étant donnés F ⊂ [0, 1] fini et ε > 0, il est facile de
construire une fonction g ∈ C ([0, 1]) affine par morceaux qui soit nulle sur F et égale à 1
sur une reunion disjointe d’intervalles I1 , . . . , Ip de longueur 1/2.
Toutefois, s’il existais une suite (fn )n dans A qui converge vers f alors elle convergerai
simplement vers f sur [0, 1] et par convergence dominée (car 0 ≤ fn ≤ 1) on aurai
Z 1 Z 1 Z 1
0= f (t)dt = lim fn (t)dt = lim fn (t)dt ≥ 1/2
0 0 n n 0
ce qui est absurde. CQFD o
EXERCICES EN COURS.....
Exercice 295 (Histoire dans un corps )
Soient a1 , a2 , . . . , a51 des éléments non nuls d’un corps K. On remplace simultané-
ment chacun de ces éléments par la somme des 50 autres. Soit b1 , b2 , . . . , b51 la suite
obtenue, si cette nouvelle suite est une permutation de l’originale que peut être la
caractéristique de K ?
Nous avons
S := a1 + a2 + · · · + a51 , b1 + b2 + · · · + b51 = 50S
soit, pour toute permutation b1 , b2 , . . . , b51 de a1 , a2 , . . . , a51
50S = S qui implique 49S = 0.
Si car(K) 6= 7 alors 49S = 0 =⇒ S = 0 puis bi = −ai pour tout 1 ≤ i ≤ 51. D’un autre
coté, il existe une permutation σ ∈ S51 telle que bi = aσ(i) = −ai . Si la caractéristique de K
est différente de 2, on peut alors construire une partition {ai , aσ(i) }51
1 de la suite a1 , a2 , . . . , a51 ,
fait absurde puisque 51 est impair. La caractéristique de K vaut donc 2 ou 7.
Les valeurs 2 et 7 sont toutes les deux possibles : pour car(K) = 7, x1 = x2 = · · · = x51 = 1
est un choix possible et pour le cas de 2, tout élément peut être choisi pour que S = 0
puisque bi = ai = −ai . o
Autrement dit, l’opérateur F injecte l’espace L1 ([0, 1]) dans l’espace vectoriel c0
des suites de complexes (an )n∈Z qui convergent vers zéro, i.e. F (L1 ([0, 1])) ⊂ c0 .
Ë Montrer que l’inclusion F (L1 ([0, 1])) ⊂ c0 est stricte.
Ê Par densité des polynômes trigonométriques VectC {einx , n ∈ Z} dans C (R/Z) (muni de
la norme « sup ») et de C 0 ([0, 1]) dans L1 ([0, 1]) (muni de la norme L1 ), il existe pour tout
f ∈ L1 ([0, 1]) deux applications g ∈ C (R/Z) et h ∈ VectC {einx , n ∈ Z} vérifiant
kf − gk1 ≤ ε et kg − hk∞ ≤ ε.
Par l’inégalité triangulaire et domination de la norme L1 par la norme « sup » on a alors
kf − hk1 < 2ε.
h étant un polynôme trigonométrique, ses coefficients de Fourier sont nuls à partir d’un
certain rang. Par orthogonalité, on a donc
Z 1
|fˆ(n)| = (f (x) − h(x))e−2iπnx
dx ≤ kf − hk1 < 2ε.
0
Ë On munit c0 de la norme k(an )kc0 = supn∈Z |an |, c’est (exercice facile) un espace de Banach
et l’inégalité
Z 1
|fˆ(n)| = −2iπnx
f (x)e dx ≤ kf k1 , ∀ f ∈ L1 ([0, 1]), n ∈ Z
0
assure que l’opérateur
F : L1 ([0, 1]) −→ c0
est une forme linéaire continue de norme inférieure ou égale à 1 (et en fait égale à 1 en
considérant f ≡ 1).
ê Nous allons maintenant vérifier que F est injective : soit f ∈ L1 ([0, 1]) vérifiant fˆ(n) =
0 ∀ n ∈ Z et montrons que f ≡ 0.
Il est déja clair que pour tout polynôme trigonométrique h ∈ VectC {einx , n ∈ Z}
Z 1
f (x)h(x)dx = 0,
0
et par convergence dominée via la densité des polynômes trigonométriques ceci vaut pour
toute fonction continue puis (par convergence dominée via Lusin..à détailler)) sur pour toute
fonction indicatrice h d’ensemble mesurable ; le choix h := 1{f >0} donne kf k1 = 0, soit f = 0.
F est bien injective.
ê Si l’inclusion n’est pas stricte, F est alors une application linéaire bijective continue
entre les deux Banach L1 ([0, 1]) et c0 . Par le théorème de l’application ouverte F est un
isomorphisme topologique : il existe donc une constante C > 0 telle que
(8) kF (f )kc0 ≥ Ckf k1 , ∀ f ∈ L1 ([0, 1]).
On considére alors pour tout N ∈ N
X
fN (x) = e−2iπnx ,
|n|≤N
il est immédiat que kF (fN )kc0 = 1 et kfN k1 = 2N + 1 contredisant (8) lorsque N tends
vers l’infini : l’inclusion F (L1 ([0, 1])) ⊂ c0 est bien stricte. o
Exercice 297 (Trois problèmes d’optimisation autour d’une droite et une pa-
rabole )
On considère un point P , distinct de l’origine et situé sur la parabole P d’équation
y = x2 . La normale à (P) passant par P recoupe la parabole en un point Q.
Ê Déterminer P pour que l’arc de parabole soit minimum.
Ë Déterminer P pour que le périmètre de la région bornée délimitée par (P) et
P Q soit minimum.
Ì Déterminer P pour que l’aire de la région bornée délimitée par (P) et P Q soit
minimum.
Ë Désignons par R(x) l’aire de la région bornée lorsque les coordonnées du point P sont
(x, x2 ). Nous avons vu dans la question précédente que Q est associé au paramètre z(x) =
2
− 2x2x+1 . Avec ceci, le périmètre est
Z x √ p
R(x) = 1 + 4t2 dt + (x − z(x)) 1 + (x + z(x))2
z(x)
Quelques manipulations algébriques sur R0 (x) montrent que la (les) solution est racine du
polynôme
8x3 + 4x2 − 4x − 1 = 0.
Un logiciel de calcul nous donne
√
7 √ 1
x= cos(Arcos( 7/14)/3) − ' 0, 62349..
3 6
et −x par symétrie o
Ainsi, toute suite (fn )n faiblement convergente dans C 0 (X) vers f vérifie
Z Z
∀ x ∈ X, fn δx = fn (x) −→ f δx = f (x)
X n→∞ X
on tire |||fn ||| ≤ ||fn ||∞ et comme cette inégalité est une égalité pour µ = δxn où xn ∈ X
vérifie |fn (xn )| = ||fn ||∞ on a
(8) |||fn ||| ≤ ||fn ||∞ .
En outre avec l’hypothèse de convergence faible la suite (hfn , µi)n est bornée (puisque conver-
gente) : on peut donc appliquer le théorème de Banach-Steinhaus qui assure que la suite
(|||fn |||)n est elle même bornée ce qui, avec (8) achève la première implication.
Réciproquement, considèrons une suite (fn )n C 0 (X) simplement convergente sur X vers
f et uniformément bornée (disons par C > 0). Comme 1A ∈ L1 (X, |µ|) pour toute mesure
de Radon sur X, on peut appliquer le théorème de la convergence dominée :
Z Z
lim fn µ = f µ, ∀ µ ∈ C 0 (X)0
n→∞ X X
Ë L’existence de la suite (f˜n )n équivaut à montrer que f est dans l’adhérence de l’enveloppe
convexe C des fn lorsque fn + f . La forme géométrique du théorème de Hahn-Banach nous
dit qu’il est équivalent de montrer qu’aucune forme linéaire ne sépare f et C :
∀ ϕ ∈ C 0 (X)0 , ϕ(C) ≤ α =⇒ ϕ(f ) ≥ α.
Soit ϕ une telle forme et µ la mesure associée :
Z
ϕ(f ) = f µ, f ∈ C 0 (X).
X
Vu les hypothèses et Ê (ou par convergence dominée) (fn )n converge faiblement vers f , en
particulier
ϕ(fn ) −→ ϕ(f )
n→∞
C (X) 0
et comme ϕ(fn ) ≥ α on aura ϕ(f ) ≥ α i.e. f ∈ C i.e. il existe une suite (f˜n )n ⊂ C qui
converge vers f dans C (X) soit uniformément sur X.
0
o
i Un polynôme trigonométrique de degré au plus n peut s’écrire sous l’une ou l’autre des
formes suivantes :
n
X n
X
∀x ∈ R f (x) = (ak cos(kx) + bk sin(kx)) = ck eikx = e−inx P (x)
k=0 k=−n
Pn k+n
où P (X) = k=−n ck X est un polynôme de degré au plus 2n.
Ê Soit f un tel polynôme. Si f admet 2n + 1 racines distinctes θ1 , θ2 , . . . θ2n+1 dans [0, 2π[,
les 2n + 1 nombres complexes distincts eiθ1 , . . . , eiθ2n+1 sont racines du polynôme P quiest
donc le polynôme nul et par suite f est la fonction nulle.
π kπ
xk = − , k = O, 1, . . . , 2n.
2n n
Nous avons
(−1)k 0
g(xk ) = kf k − f (xk )
n
et l’hypothèse kf 0 k∞ > nkf k assure que g(xk ) est du signe de (−1)k . Par conséquent g
s’annule sur chaque intervalle ]xk , xk+1 [ ce qui donne 2n zéros sur l’intervalle [x0 , x0 + 2p i].
La 2π-périodicité permet d’affirmer que g s’annule également 2n fois sur [0, π[.
Ì o
1) Supposons qu’il existe un intervalle [a, b] ⊂ R sur lequel f n’atteint pas son maximum
en les extrémités a et b. Par continuité de f nous avons tout de même
à suivre.........
24A (ORM )
p= √ .
3
Fixons l’origine en O, l’axe des abcisses positives suivant OB et celui des ordonnées positives
suivant la direction de AM . Les coordonnées (x, y) de R vérifient
√
x√ 1 2 3 2 3
y= 3, (x − ) + (y − ) =
3 2 2 4
soit
1 √ 1 √ √
x = (3 − 6), y = ( 3 − 2).
4 4
à suivre................... q
Rx a0
Ê Par périodicité f est intégrable sur [0, 2π] et par suite la fonction F (x) = 0
(f (t)− )dt
2
est à variation bornée sur [0, 2π] ; en outre, elle est 2π-périodique et
Ë
Ì
Í
Î
est une forme linéaire continue sur Ks [[x]] et que l’on obtient ainsi toutes les formes
linéaires continues.
c) Montrer que le sous-espace vectoriel K[x] des polynômes est dense dans
Ks [[x]] et que les formes linéaires sur ce sous-espace s’écrivent P 7→ hP, Qi où
Q ∈ K[x]. Si P est une série formelle qui n’est pas un polynôme, vérifier que la
forme linéaire P 7→ hP, Qi sur l’espace K[x] n’est pas continue.
où J est une partie finie de I. Il est clair qu’un tel voisinage n’est pas borné (comme X est
infini, il contient les droites vectorielles Kf où f ∈ Fs (X, E), f 6= 0 et f/J ≡ 0). Ts est une
topologie non localement bornée, donc non normable. Réciproquement, X fini et E normé
impliquent Fs (X, E) normable : si X = {x1 , . . . , xn } on définit une norme sur F (X, E) en
posant N (f ) = max1≤i≤n kf (xi )k et la topologie associée à cette est norme coïncide avec Ts ;
pour s’en persuader on peut remarquer par exemple que
∀1 ≤ i ≤ n : kfi (x)k ≤ N (f ) = max kf (xi )k
1≤i≤n
assure que l’identité entre (F (X, E), TN ) et (F (X, E), Ts ) est un isomorphisme topologique.
Ï a) Soit a ∈ X l’égalité |δa (f )| = |f (a)| = pa (f ) garanti la continuité de δa .
b) et c) Un résultat d’algèbre (exercice ? ? ? i) linéaire assure que pour tout espace
vectoriel E, si des formes linéaires f, f1 , . . . , fn ∈ E ? vérifient (fi (x) = 0, ∀ i ∈ {1, . . . , n}) ⇒
(f (x) = 0) alors il existe des constantes c1 , . . . , cn ∈ K telles que f = c1 f1 + · · · + cn fn .
Soit T une forme linéaire continue sur F (X, E), il existe une partie finie A ⊂ X et une
constante C > 0 telles que
∀ F (X, E) : |T (f )| ≤ C sup pa (f ) = C sup |f (a)| = C sup |δa (f )|.
a∈A a∈A a∈A
Autrement dit,
Y
B ⊂ K := {|z| ≤ Cl } ⊂ KN ' Fs (N, K)
l∈N
La topologie induite sur K par Fs (N, K) est bien entedu la « topologie produit » : K est
donc une partie compacte de Fs (N, K) d’aprés le théorème de Tychonoff ([46] théorème
2.32.5) et B est bien relativement compacte.
b)
c)
o
Ainsi, pour t = g(x) ∈ im(g), la formule h1 (t) = f (x) définit bien une application h1 ∈
L (im(g), F ) vérifiant f = h1 ◦ g sur im(g) et le problème est déja résolu sur im(g). Pour
construire une solution sur G, on considére (puisque G est de dimension finie) un supplé-
mentaire H de im(g) dans G et p la projection sur im(g) paralellement à H ; alors, h = h1 ◦ p
répond visiblement au problème.
Ë Pour l’application en considèrant g = (f1 , . . . , fn ) ∈ L (E, Kn ), on se retrouve dans la
situation précédente avec F = K et G = Kn : il existe donc h ∈ L (Cn , C) telle que f = h ◦ g.
Mais la forme générale des éléments de L (Kn , K) est bien connue : il existe c1 , . . . , cn ∈ K
tels que h(z) = (z1 , . . . , zn )) = c1 z1 + · · · + cn zn d’où le résultat.
i Remarque : Ce dernier résultat est essentiel en analyse fonctionnelle (voir l’exercice pré-
cédent où la question ci-dessous) ; on trouvera aussi dans H.Brézis « Analyse fonctionnelle »
Masson 19 ? ? une démonstration trés amusante de ce résultat s’appuyant sur le théorème
d’Hahn-Banach.
Ì T ∈ D 0 (Rd ) est à support compact, donc d’ordre fini : il existe une constante C > 0, un
entier N ∈ N tels que6
|hT, ϕi| ≤ C max |ϕ(k) (0)|, ∀ ϕ ∈ D(Rd ).
n∈Nd , |k|≤N
Ainsi, la forme linéaire T s’annule au point ϕ dés que les formes linéaires δ (k) : ϕ 7→
hδ (k) , ϕi = (−1)|k| ϕ(k) (0), |k| ≤ N s’annulent. Avec la question précédente, il existe des
scalaires ci tels que
X
T = ci δ (i) ,
|i|≤N
Q.E.D. o
Exercice 306 (Exemple d’une série trigonométrique qui n’est pas une série de
Fourier)
P [37]
Soit n≥1 an sin(nt) une série trigonométrique où an ≥ 0 ∀ n ≥ 1.
Ê On suppose que cette sérieR est une série de Fourier, c’est à dire qu’il existe
2π
f ∈ L1loc (R) telle que cn (f ) = 0 f (t)e−int dt ∀ n ∈ Z i.e. c0 (f ) = 0 et, si n ≥ 1 :
Rt
cn (f ) = a2in , c−n (f ) = − a2in . Soit F (t) = 0 f (u)du. Montrer que F est continue et
2π-périodique sur R et, pour
a|n|
|n| ≥ 1 : c|n| (F ) = −
2|n|
an
P
Ë En déduire que n≥1 n converge.
Ì Montrer que la série trigonométrique partout convergente n≥2 sin(nt)
P
log(n)
n’est pas
une série de Fourier.
La solution..........
6rappel : si k = (k , . . . , k ) ∈ Nd on note |k| := k + · · · + k .
1 d 1 d
Ê À tout point p = (x, y) ∈ Z2 associons C(p) le carré plein centré en p dont les cotés de
longueur 1 sont paralléles aux axes. On vérifie facilement que si p 6= p0 ∈ Z2 alors l’aire de
C(p) ∩ C(p0 ) est nulle. Posons alors :
[ [ [
A(n) = C(p), B(n) = C(p), C(n) = C(p)
p∈Z2 p∈Z2 ∩D(n) p∈Z2
C(p)⊂D(n) C(p)∩D(n)6=∅
Visiblement A(n) ⊂ B(n) ⊂ C(n), et par construction même, l’aire de B(n) est précisément
pn . En outre comme
soit pn ∼ πn2 .
n→∞
Maintenant, remarquons que dans D(n), les points à coordonnées entières sur les axes sont
au plus 4n = o(n2 ) donc négligeables par rapport à pn : on peut donc ignorer ces points, des
πn2
arguments évidents de symétrie impliquent alors que p+ n ∼ p n /4 soit p +
n ∼ .
n→∞ n→∞ 4
d
X √
Maintenant, si l’on observe que q(k) = p+ ( d), on peut écrire
k=0
∞
X √
G(t) = p+ ( k)tk .
k=0
√ πd
D’aprés la première question p+ ( d) ∼ . Considérons alors pour |t| < 1
n→∞ 4
∞
πX π 1
H(t) := (n + 1)tn = .
4 n=0 4 (1 − t)2
Comme
πn/4
lim = 1,
n→∞ π(n + 1)/4
soit encore
∞ √
X
n2 π
t ∼ √ .
t→1− 2 1 − t
n=0
Q.E.D.
Ì Commencer par calculer la limite lorsque x tends vers 1− et pour en déduire la limite en
−1 découper la somme en parties paire et impaires...
Solution 1 :
Solution 2 : q
i Remarque liminaire : dans une configuration optimale les termes σ(i) doivent alternati-
vement croitre et décroitre : en effet si σ(i) < σ(i+1) < σ(i+2) (ou σ(i) > σ(i+1) > σ(i+2))
alors |σ(i) − σ(i + 1)| + |σ(i + 1) − σ(i + 2)| = |σ(i) − σ(i + 2)|, si bien que la permutation
associée à l’une des deux deux suites d’entiers (σ(i + 1), σ(1), . . . , σ(i), σ(i + 2), . . . , σ(n)) et
(σ(1), . . . , σ(i), σ(i + 2), . . . , σ(n), σ(i + 1)) fournira une somme strictement plus grande que
celle associée à σ.
Passons à la solution. Mieux vaut distinguer les cas n pair et impair. Si n = 2m............ o
Ê Il n’est pas difficile de démontrer que Df est de dimension finie si, et seulement si, il
existe une suite λ1 , . . . , λd de nombres complexes tels que
f (d) + λd f (d−1) + · · · + λ1 f = 0.
f est donc solution d’une équation différentielle linéaire à coefficients constants dont la
structure des solutions est parfaitement connue ([12], [33])
d
X
f (t) = Pj (t)eλj t , t ∈ R où λj ∈ C, Pj ∈ C[X].
j=1
Réciproquement, il est bien évident que pour de telles fonctions Df est de dimension finie.
Ë Tf étant engendré par les translations x 7→ fa (x) = f (x−a), admet, s’il est de dimension
finie, une base de la forme fa1 , . . . , fad . Autrement dit, il existe des applications ϕ1 , . . . , ϕd
de R dans C telles que
Xd
∀ a ∈ R, fa = ϕj (a)faj
j=1
et il reste à montrer que ces applications ont la même régularité que f . Pour cela, le lemme
suivant est crutial
« Si E est un sous-espace vectoriel de dimension finie de C 0 (R) il existe une base de E ?
de la forme (δx1 , . . . , δxd ) (où δx (f ) := f (x) est la masse Dirac au point x). En outre, si
(f1 , . . . , fd ) est une base de E : det((δxj (fi )))ij 6= 0. »
Preuve du lemme : Comme dim(E ? ) = dim(E) < ∞ et que toute famille (δx1 , . . . , δxd )
est libre dans E ? dès que les réels xi sont deux à deux distincts l’existence de telles bases
est élémentaire. On vérifie alors sans peine que la matrice de passage P de la base de E,
(g1 , . . . , gd ) duale de (δx1 , . . . , δxd ) à la base (f1 , . . . , fd ) est précisément la matrice ((δxj (fi )))ij
qui est donc de déterminant non nul.
Ceci étant acquis, étant donné une base (fa1 , . . . , fad ) de Tf et (c.f. le lemme) (x1 , . . . , xd )
tels que la matrice A = ((fai (xj ))) soit inversible, nous avons
d
X
(8) ∀ a ∈ R, fa (x) = ϕi (a)fai (x)
i=1
et en particulier
d
X d
X
fa (xj ) = ϕi (a)fa (xj ) = ϕi (a)δxj (fa ), ∀ j ∈ {1, . . . , d}.
i=1 i=1
A .
. = .. .. .. = ..
. . . . .
ϕd (a) fad (a1 ) . . . fad (ad ) ϕd (a) f (ad − a)
mais par le lemme précédent, A est inversible, si bien qu’en posant B = A−1 = ((bij )) le
système linéaire précédent s’inverse pour donner
d
X
ϕi (a) = bij f (aj − a), 1 ≤ i ≤ d.
j=1
Ces égalités assurent que les applications ϕi sont au moins aussi régulières que f .
Ì Cette question maximise la précédente puisqu’il s’agit de montrer que les applications
ϕj sont C ∞ dès que f est continue (rien d’étonnant, tout va s’expliquer dans la dernière
question). Pour cela, considérons7 une « approximation de l’identité » (θk )k . Les applications
θk étant à support compact, et f continue donc localement intégrable, l’application θk ? f
est bien définie, de classe C ∞ et on vérifie facilement que (θk ? f )a = θk ? fa . En outre, la
convolution étant linéaire
d
! d d
X X X
(8) (θk ? f )a = θk ? fa = θk ? ϕj (a)faj = ϕj (a)(θk ? faj ) = ϕj (a)(θk ? f )aj .
j=1 j=1 j=1
L’espace vectoriel Tθk ?f admet donc ((θk ? f )a1 , . . . , (θk ? f )ad ) comme famille génératrice : il
est donc de dimension finie. En outre, (θk )k étant une approximation de l’identité, la matrice
Ak = ((θk ?fai (xj )))ij = ((δxj (θk ? fai )))ij converge vers la matrice inversible A = ((faj (xi ))).
Par continuité du déterminant, il existe un entier k0 tel que Ak ∈ GLd (R), ∀ k ≥ k0 . La
linéarité des masses de Dirac δxj implique alors que la famille ((θk0 ? f )a1 , . . . , (θk0 ? f )ad ) est
aussi libre, c’est donc une base de Tθk0 ∗f et la formule (8) implique alors que les coordonnées
7[47] : θ ∈ C ∞ (R), R θ (x)dx = 1, θ (x) = 0 ∀ |x| ≥ 1/k et lim θ ? f (x) = f (x) pour tout x ∈ R et f ∈ L1 (R)...
k R k k k k loc
de (θk0 ? f )a sont ϕ1 (a), . . . , ϕd (a) : il ne reste plus qu’à appliquer la question Ë à la fonction
(θk0 ? f )a ∈ C ∞ (R) pour pouvoir affirmer que ϕ1 , . . . , ϕd sont C ∞ et conclure.
Í Soit f ∈ C 0 (R) telle que dim(Tf ) < ∞. Avec (4), nous avons
d
X d
X
∀ x ∈ R, f (x) = f−x (0) = ϕj (x)faj (0) = ϕj (−x)f (−aj )
j=1 j=1
Î Il reste à établir
d
!
X
(dim Tf = d < ∞) ⇐⇒ (dim Df = d < ∞) ⇐⇒ f (t) = Pj (t)eλj t .
j=1
La seconde équivalence a fait l’objet de la première question. Supposons que dim(Tf ) < ∞,
f ∈ C ∞ (R) et pour tout k ∈ N
d
X d
X
(k) (k)
(f )−a = (f−a ) = ϕi (−a)fx(k)
i
= ϕi (−a)(f (k) )xi , ∀ a ∈ R,
i=1 i=1
ce qui montre que toutes les dérivées de f sont dans l’espace vectoriel engendré par les
fonctions a 7→ ϕi (−a), 1 ≤ i ≤ d : Df est donc de dimension finie.
Réciproquement, si dim(Df ) < ∞, on peut écrire
d
X
f (x) = Pi (x)eλi x .
i=1
nR o
1
Exercice 311 (inf 0
|f 0 (x) − f (x)|dx, f ∈ C 1 ([0, 1], R), f (0) = 0, f (1) = 1 =
e−1 . ) [10]
Existence et calcul de
Z 1
m := inf |f (x) − f (x)|dx, f ∈ C ([0, 1], R), f (0) = 0, f (1) = 1
0 1
.
0
On vérifie sans peine que fa ∈ C 1 ([0, 1], R) pour tout 0 < a < 1 et comme
Z 1 a
|fa0 (x) − fa (x)|dx = ea−1 1 −
0 2
nous avons
1 a 1
≤ m ≤ ea−1 1 − −→ ,
e 2 a→0 e
et finalement m = e−1 . o
Ë Pour y = x l’équation fonctionnelle donne f (xf (x)) = xf (x) et donc xf (x) est un point
fixe (non nul) de f pour tout x ∈ R?+ . Notons a = xf (x) un tel point fixe. Nous avons o
Pour l’application, supposons que la série harmonique converge, alors avec (8)
∞
X 1 1 1 1 1 1 1 3 3 3
H := =1+ + + + + + + ··· < 1 + + + + ··· = 1 + H
k=1
k 2 3 4 5 6 7 3 6 9
Ë Comme on le voit sur la figure tout tel polygône peut être considère comme un agglomé-
rat de triangles isocèles admettant tous l’origine comme un des sommets et dont la réunion
des aires est celle du polygône. L’aire achurée du triangle sur la figure vaut 21 sin(θ1 ) et si on
désigne par θ1 , . . . , θN les angles correspondants, l’aire du polygône sera
N N
X 1 X
A= sin(θj ), 0 < θj < π, θj = 2π.
j=1
2 j=1
Par concavité de la fonction sinus sur [0, π] nous avons avec Jensen
N N
!
1X n 1X n 2π
A= sin(θj ) ≤ sin sin(θj ) = sin .
2 j=1 2 n j=1 2 n
Par stricte concavité de la fonction sinus sur [0, π], le cas d’égalité dans la formule de Jensen
assure qu’il y aura égalité dans la formule précédente si, et seulement si θj = 2πn
, configuration
qui correspond bien au cas d’un polygône régulier.
i Profitons-en pour signaler que cette inégalité et son application pour la divergence de la série harmonique sont attribuées
au mathématicien Italien Pietro Mengoli (1625-1686).
Bn
≤ 1 et le rayon de convergence R de la série entière f (z) = ∞ Bn n
P
On a donc k=0 n! z est
n!
supérieur ou égal à 1.
On va utiliser la formule démontrée dans la première question pour calculer f (z). Pour
z ∈] − R, R[
∞ ∞
X Bn n X Bn+1 n+1
f (z) = z =1+ z ,
k=0
n! k=0
(n + 1)!
donc
∞
0
X Bn+1
f (z) = zn
k=0
n!
∞ n
!
X 1 X
= Cnk zn
k=0
n! k=0
∞ n
!
X X Bk 1
= zn
k=0 k=0
k! (n − k)!
P∞ zn
Ì Le rayon de convergence de la série entière k=0 n! = ez étant infini, on a
∞ ∞ ∞
!
ez
X enz X 1 X (nz)k
e = = , ∀ z ∈ C.
n=0
n! n=0
n! k=0
k!
k
La série double (n,k)∈N2 un,k (où un,k = (nz) ) est sommable9 ; il est donc légitime d’échanger
P
n!k!
l’ordre de sommation
∞ ∞
! ∞ ∞
! ∞
!
1X X 1X X X 1 X nk
f (z) = un,k = un,k = z n , z ∈] − R, R[,
e n=0 k=0 e k=0 n=0 k=0
e n=0
n!
soit par unicité des coefficients
∞
1 X nk
Bk = .
e n=0 n!
Q.E.D. o
Exercice 316 (Une suite dans C[X1 , X2 , . . . , Xn ] qui « s’annule » sur C ) [10],
2004.
Ê Soit (Pk )k ⊂ C[X1 , X2 , . . . , Xn ]. On suppose que pour tout x ∈ Cn il existe
k ∈ N tel que Pk (x) = 0. Montrer qu’il existe un entier k tel que Pk soit le polynôme
nul.
Ë Si on remplace C par Q, la conclusion de Ê reste-t-elle valable ?
o
9car P |u | = e|nz| /n! et P P |u | = ee|z| ,voir [12] (T2, page ...).
k n,k n k n,k
Exercice 318 (n(n2 + 1)/2 est valeur propre de toute matrice magique A ∈
Mn (R). ) [34], E 733-1947.
Montrer que toute matrice magique A ∈ Mn (R) admet n(n2 + 1)/2 comme valeur
propre.
Soit A ∈ Mn (R) une matrice magique10. Désignons pâr s la somme des composantes d’une
colonne de A, si X = t (1, . . . , 1) ∈ Rn , comme A est magique
AX = sX.
La somme des n2 premiers entiers vaut n2 (n2 + 1)/2 et c’est la somme des n colonnes de A
soit n2 (n2 + 1)/2 = ns et finalement s = n(n2 + 1)/2. o
Exercice 319 (Une base de deux Banach X et Y n’en est pas forcément une
pour X ∩ Y ) [34]
10Une matrice est magique si les sommes des composantes de chacunes de ses lignes et de ses colonnes sont égales et si
elle est à coefficients dans {1, 2, . . . , n2 } tous ces entiers apparaissant exactement une fois dans A.
(⇒) Fixons nous α ∈ R et une droite (Da,b ) d’équation y = ax+b. Le point (x, y) appartient
à (Cα ) ∩ (Da,b ) si, et seulement si
Il s’agit donc de déterminer une condition sur α pour que la fonction fa,b admette 4 racines
0
réelles distinctes. Si tel est le cas, le théorème de Rolle assure que fa,b admet 3 racines réelles
00 00
distinctes et de même fa,b admet 2 racines réelles distinctes. Mais fa,b (x) = 12x2 + 54x + 2α,
son discriminant ∆ = 542 − 96α doit être strictement négatif, i.e. α < 243/8.
(⇐) Réciproquement, si α < 243/8 et montrons qu’il existe (a, b) ∈ R2 tel que fa,b admette
4 racines réelles distinctes.
00
α < 243/8 assure que le discriminant de fa,b (x) = 12x2 + 54x + 2α est strictement négatif,
00
fa,b admet donc deux racines réelles z1 < z2 et nécessairement
0
fa,b est strictement croissante sur ] − ∞, z1 ]
0
fa,b est strictement décroissante sur [z1 , z2 ]
0
fa,b est strictement croissante sur [z2 , +∞[.
0 0
En particulier fa,b (z1 ) > fa,b (z2 ) et la constante a n’apparaissant que dans le terme constant
0
de fa,b , il est donc possible de choisir convenablement a de telle sorte (faites un dessin) que
0 0
fa,b (z1 ) > 0 et fa,b (z2 ) < 0.
0 0 0
De plus, comme lim+∞ fa,b = +∞, lim−∞ fa,b = −∞ la fonction fa,b admet forcément trois
racines réelles distinctes y1 < y2 < y3 et
On va maintenant jouer sur b qui n’apparait que dans le coefficient constant de fa,b sous la
forme 4 − b : en d’autres termes, augmenter b revient a translater le graphe de fa,b suivant
la direction de l’axe des ordonnées. comme fa,b (y2 ) > fa,b (y1 ) > fa,b (y3 ) il suffit de choisir
correctement b ∈ R pour que fa,b (y2 ) > 0, fa,b (y1 ) < 0, fa,b (y3 ) < 0. o
Exercice 322 (Un théorème d’Erdös sur les fonctions multiplicatives mono-
tones ) [34], (8)1986.
Un fonction arithmétique f : N → N non identiquement nulle est dite multipli-
cative si
f (mn) = f (m)f (n) dès que m ∧ n = 1,
et complétement multiplicative si
f (mn) = f (m)f (n) pour tous n, m ∈ N.
Ê Montrer que si f est complétement multiplicative et croissante, il existe une
constante α telle que f (n) = nα , ∀ n ∈ N? .
Ë Montrer que toute application multiplicative croissante est complétement mul-
tiplicative.
Exercice 323 (Autour d’une ellipse) Problem of the week, spring 2006.
Dans une ellipse E d’aire 1, on considère deux cordes parallèles respectivement aux
deux axes de E . Ces deux cordes divisent l’ellipse en quatre régions, montrer qu’au
moins deux regions ont une aire inférieure ou égale à 1/4.
Sans perdre de généralité, supposons que la somme des aires des deux régions gri-
les deux cordes se coupent dans le premier sées est inférieure ou égale à 1/2. En effet,
cadran (fermé) ; si on rajoute les deux axes elle vaut (B + 2C) + (B + 2D) et comme
de E et les deux cordes symétriques dé- 4(A + B + C + D) = Aire(E ) = 1 on aura
duites des deux premières par une symétrie
1 1
de centre O le centre de E on obtient 16 (B+2C)+(B+2D) = 2(B+C+D) = (1−4A) ≤ .
régions dont les aires sont désignées par les 2 4
lettres A, B, C, D (voir la figure, certaines Ainsi, une des deux quantités B +2C, B +2D
bien entendu, pouvant être d’aire nulle si est inférieure à 1/4 ce qui achève la dé-
une corde est confondu avec un axe). Il déja monstration (les deux domaines seront (B)
clair que B ≤ 1/4, nous allons montrer que et (B, C, C, B) ou (B, D, D, B)). o
Ë Nous avons
|z1 + · · · + zn | ≥ |Re(z1 + · · · + zn )|
= |z1 | cos(θ1 ) + · · · + |zn | cos(θn )
≥ (|z1 | + · · · + |zn |) cos(ψ)
≥ n(|z1 | . . . |zn |)1/n cos(ψ)
où l’on a successivement utilisé la décroissance de la fonction cosinus sur [0, π/2] et l’inégalité
arithmético-géométrique11 sur les réels positifs |zj |, j = 1, . . . , n.
Ê On a pour k ∈ N?
k−1
X
tr(M + H)k = tr(M k ) + tr(M i HM k−1−i ) + O(kHk2 ),
i=0
11 a1 + a2 + · · · + an
(a1 a2 . . . an )1/n ≤ pour a1 . . . an ∈ R+ .
n
Ë Pour X ∈ Mn (R), désignons par ΦX la forme linéaire sur Mn (R) définie par ΦX (H) =
tr(XH). L’application X 7→ ΦX réalise un isomorphisme entre Mn (R) et son dual Mn (R)? .
Ainsi les formes (ΦM k )n−1
0 forment une famille de rang égal à celui des matrices (M k )n−1
0 lui
même égal à d le degré du polynôme minimal de M . Par conséquent, dim ker df (M ) = n2 −d,
donc rgdf (M ) = d.
montrer que √ √
x1 + x2 ≥ 10.
Si x1 ≥ 100 il n’y a rien à démontrer, supposons donc x1 ≤ 100. On peut alors écrire
n
X n
X
103 ≤ x21 + x2j ≤ x21 + x2 xj
j=2 j=2
Exercice 327 (Autour des cordes universelles des fonctions continues ) [21],
[10]-2007/4.
Soit f ∈ C ([0, 1], R) telle que f (0) = f (1). On dira que c > 0 est une corde pour
f s’il existe un nombre réel x tel que x et x + c soient tous deux les dans [0, 1] et
vérifient f (x + c) = f (x). On dira que c est une corde universelle s’il est une corde
pour toute fonction f ∈ C ([0, 1], R) telle que f (0) = f (1).
Ê Montrer que les réels 1, 1/2, 1/3, . . . , 1/n, . . . (n ∈ N? ) sont des cordes univer-
selles.
Ë Soit 0 < c < 1 qui ne soit pas l’inverse d’un entier. Construire une fonction
g ∈ C ([0, 1], R) vérifiant g(0) = 0, g(1) = 1 et g(x + c) = g(x), x ∈ [0, 1 − c]. En
considérant f (x) := g(x) − x, montrer que c n’est pas une corde universelle.
Ì (Application) Un marcheur parcourt (continuement) 40 kilomètres en deux
heures. Montrer qu’il existe une période d’une heure où il parcourt 20 kilomètres
exactement.
Í On suppose que
f (x + 3/10) 6= f (x), ∀ x ∈ [0, 7/10].
Montrer que f s’annule au moins 7 fois sur [0, 1].
Ë Supposons maintenant que c ne soit pas l’inverse d’un entier, comme dans la question
précédente on commence par construire une fonction g continue sur [0, 1] telle que
g(0) = 0, g(1) = 1 et g(x) = g(x + c), x ∈ [0, 1 − c].
Il est possible de s’assurer de l’existence d’une telle fonction en dessinant son graphe mais il
est aussi possible de donner une formule explicite de l’une d’entre-elles :
2πx
sin
c
g(x) = .
2π
sin
c
Considérons alors
f (x) := x − g(x), x ∈ [0, 1].
f est continue sur [0, 1], f (0) = f (1) = 0 mais c ne peut être une corde pour f : en effet, s’il
existe x ∈ [0, 1 − c] tel que f (x) = f (x + c) alors
f (x + c) − f (x) = (x + c − g(x + c)) − (x − g(x)) = c > 0,
et c n’est pas une corde universelle.
1 1
Ê Montrer que < Un − γ < , n ∈ N.
2(n + 1) 2n
Ë Si on modifie légèrement la suite (Un )n en la remplacant par la suite (Vn )n où
1 1 1 1
Vn = 1 + + + · · · + − log n + ,
2 3 n 2
nous allons vérifier que la convergence est notablement accélérée, plus précisément
nous avons
1 1
2
< Vn − γ < , n ∈ N.
24(n + 1) 24n2
Pour cela, soit f (x) = −(x + 1)−1 − log(x + 12 ) + log(x + 23 ).
Vérifier que Vn − Vn+1 = f (n), n ∈ N.
Montrer que −f 0 (x) < 41 (x + 12 )−4 . En déduire que
Z k+1
1 1 −3 dt
f (k) ≤ (k + ) < t−3
12 2 k 12
et montrer l’inégalité de droite.
Faire de même à gauche et conclure.
Ë Comme
1
f 0 (x) = − , x ∈ R?+
4(x + 1)2 (x + 1/2)(x + 3/2)
on a
1
−f 0 (x) <
4
, x ∈ R?+ .
4(x + 1/2)
De là, limx→∞ f (x) = 0 assure que pour tout k ∈ N?
Z ∞
1 ∞
Z
0 1
f (k) = − f (x)dx < (x + 1/2)−4 dx = .
k 4 k 12(k + 1/2)3
Maintenant, en remarquant que (k + 1/2)2 = k 2 + k + 1/4 > k(k + 1) on peut écrire
Z k+1
1 1 1 2k + 1
3
< 2 2
< · 2 2
= x−3 dx,
(k + 1/2) (k + 1/2)k (k + 1) 2 k (k + 1) k
(cette inégalité peut être suggérée par par la figure ci-contre) soit
Z k+1
1
f (k) < x−3 dx, ∀ k ∈ N? .
12 k
o
i Il est essentiel de se souvenir qu’un point isolé est toujours dans l’ensemble. Remarquons
aussi qu’ici E est fermé comme image réciproque du fermé {0} par l’application continue
A 7→ P (A). On peut enfin observer que A ∈ E implique que les valeurs propres de A sont
des racines de P et que la classe de conjugaison SA = { P −1 AP, P ∈ GLn (C) } de A est
aussi dans E . Comme SA est non bornée dès que13 A 6= λIn il en sera de même pour E dès
que P admet au moins deux racines distinctes.
13Voir l’exercice ? ? ? sur les classes de conjugaison.
et dans ce cas, la seule alternative est que le polynome minimal de Nk soit égal à X − λIn
i.e. Nk = λIn à partir d’un certain rang et le point λIn est bien isolé dans E . o
Ê
Ê ê Par le théorème de la base incomplète Oi est non vide et c’est un ouvert (E n−k est
bien entendu muni de sa topologie usuelle d’espace vectoriel normé) vu l’évidente continuité
des applications fi .
Il reste à établir la densité. Soit u = (u1 , . . . , un−k ) ∈ E n−k , pour v ∈ Oi l’application
est un polynôme en x non identiquement nul puisque pi (0) = fi (v) 6= 0. Il existe donc R > 0
tel que x ≥ R implique pi (x) 6= 0. Ainsi xu + v et par suite u + x−1 v (car pour x ∈ R? on a
pi (x) = xn pi (u + x−1 v)...) est dans Oi pour tout x ≥ R. Mais
Toute boule centrée en u rencontre donc Oi qui est bien un ouvert dense de E n−k .
Pour achever la démonstration, le théorème de Baire assure que ∩i∈N Oi est une partie dense
de E n−k et pour tout (e1 , . . . , en−k ) ∈ ∩i∈N Oi l’ensemble F := vect{e1 , . . . , en−k } convient.
Ê
Ê o
P109
Exercice 331 (Partie entière de k=1 k −2/3 ) [15].
X109
Déterminer la partie entière de k −2/3 .
k=1
La fonction t 7→ t−2/3 est positive strictement décroissante sur R?+ , on a donc pour tout
n≥2
Z n+1 Z n
dt 1 dt
2/3
< 2/3 < 2/3
.
n t n n−1 t
Soit
Z 109 109 Z 109
dt X 1 dt
< < ,
2 t2/3 k=2
k 2/3 1 t2/3
donc
109 √
√
3
√
3
X 1 3
3 109 + 1 − 2 < < 3 109 − 1 = 2997
k=2
k 2/3
et comme
√ √ √ √
3 3 3 3
3 109 + 1 − 2 >3 109 − 2 ≥ 2996
10 9
X 1
2997 < 2/3
< 2998.
k=1
k
On se place dans le plan complexe et sans perdre de généralité supposons que le premier
segment relie les points 0 et 1. Il est alors clair que pour θ = 0 (resp. θ = π) la ligne brisée
va (par divergence de la série harmonique) décrire tout le demi axe R+ (resp. R− ) qui n’aura
donc pas d’extrémité finale. Nous allons vérifier que ce sont les seules valeurs de θ qui font
« diverger » la contruction.
iθ
Fixons θ ∈ [0, 2π[, le second segment va relier les points 1 et 1 + e2 ; le n-ième segment
i(n−1)θ
représente le complexe e n et par conséquent, lorsqu’il sera raccordé aux n − 1 précédents
i(k−1)θ
son extrémité finale est nk=1 e k .
P
Ainsi, à θ fixé, la ligne brisée aura une extrémité finale si, et seulement si la série
∞
X ei(k−1)θ
(8)
k=1
k
converge.
ê Si θ ≡ 0(π) on retrouve la série harmonique, la ligne brisée n’a pas d’extrémité finale.
Comme bk = 1/k décroit vers 0 la convergence de la série (8) est assurée par le lemme
d’Abel14.
Il reste maintenant à évaluer la somme de la série, pour cela utilisons un autre théorème
d’Abel15 :
X X X
Si ck converge, alors lim ck r k = ck .
r→1−
k≥1 k≥1 k≥1
ainsi
∞
X ei(k−1)θ
= −eiθ log(1 − eiθ )
k=1
k
= ei(π−θ) log(2 sin(θ/2)ei(θ−π)/2 )
i(π−θ) θ−π
=e log(2 sin(θ/2)) + i ·
2
La distance entre les deux extrémités de la ligne brisée est donc égale à
r
(θ − π)2
log2 (2 sin(θ/2)) + ,
4
et l’angle
θ−π
π − θ + arg log(2 sin(θ/2)) + i · .
2
o
on peut aussi utiliser les séries de Fourier. En effet la série Fourier de la fonction 2π-périodique
impaire égale à θ 7→ (π − θ)/2 sur [0, 2π] est k≥1 sin(kθ)
P
k
, celle de θ 7→ log(2 sin(θ/2)) est
− k≥1 k . Les deux applications étant de classe C sur ]0, 2π[ elles y sont développables
cos(kθ) 1
P
en série de Fourier, soit
X cos(kθ) X sin(kθ) π−θ
= − log(2 sin(θ/2)), = , 0 < θ < 2π.
k≥1
k k≥1
k 2
Exercice 334 (La formule de Stirling via la loi de Poisson ) [34], 2007-3.
Soit Xλ une variable aléatoire définie sur un espace probabilisé (Ω, A , p) suivant
une loi de Poisson de paramètre λ > 0, i.e.
λk −λ
p(Xλ = k) = e , k ∈ N.
k!
Ê Calculer sa fonction caractéristique ϕXλ et montrer que
Z π
k k −λ 1 iθ
Ik := e = ek(e −1−iθ) dθ, ∀ k ∈ N.
k! 2π −π
Ë En déduire la formule Stirling
√
k
k
k! ' · 2πk.
e
Ê On a
∞
iθ −1)
X
ϕXλ (θ) = p(Xλ = k)eikθ = eλ(e .
k=0
La convergence uniforme sur R (par rapport à la variable θ) et donc sur [−π, π] assure une
intégration terme à terme pour en déduire que la série précédente est bien la série de Fourier
de ϕXλ :
Z π
1
p(Xλ = k) = ϕXλ (θ)e−ikθ dθ, ∀ λ > 0, k ∈ N.
2π −π
En particulier, λ = k donne la formule désirée.
√
Ë Faisons le changement de variable y = θ k dans Ik , il vient
√ Z π √k √ i
k k+1/2 −k 1 h √
iy/ k
Ik k = e = exp k(e − 1 − iy/ k) dy
k! 2π −π√k
Z h √ √ i
= 1[−π√k,π√k] (y) exp k(eiy/ k − 1 − iy/ k) dy.
R
2
Mais l’intégrande est simplement convergente sur R vers t 7→ e−t /2 lorsque k → ∞ et on a
la domination
h √ √ i √ 2
√ 2 2
exp k(eiy/ k
− 1 − iy/ k) = ek(cos(y/ k)−1) = e−2k sin (y/2 k) ≤ e−2y /π ∈ L1 (R)
Ê La suite (Xn )n étant une suite de variables aléatoire indépendantes suivant une même loi
de Bernoulli de paramètre x il est bien connu ([35]-1, proposition 3-15) que Sn suit une loi
binomiale de B(n, x). Par le théorème du transfert ([35]-1, théorème 5-2)
X n
Sn k
E f = f P (Sn = k)
n k=0
n
n
X k n k
= f x (1 − x)n−k
k=0
n k
= Bn (f, x).
Sn
|Bn (f, x) − f (x)| = E f
− f (x)
n
S n S n
≤ E 1(| Sn −x|≤ε) f − f (x) + E 1(| Sn −x|>ε) f − f (x)
n n n n
Sn Sn
≤ δ(ε) · P − x ≤ ε + 2kf k∞ · P − x > ε
n n
Sn
δ(ε) + 2kf k∞ · P − x > ε .
n
Sn Sn Var(Sn ) nx(1 − x)
P − x > ε = P − E(Sn /n) > ε ≤
2 2
= ,
n n nε n 2 ε2
kf k∞
sup |Bn (f, x) − f (x)| ≤ δ(ε) + .
x∈[0,1] 2nε2
soit
C.Q.F.D. o
17P (|X − E(X)| > t) ≤ Var(X) , X ∈ L2 , voir [4], pp 59, ou bien [35] pp. 133.
18
t2
E(Sn ) = nx, V ar(Sn ) = nx(1 − x)...
i Remarque : Pour montrer que A = B le calcul différentiel fourni une solution plus
élaborée mais trés élégante : remplaçons X par t−1 X et multiplions par tn , on obtient
d
(det(X + tA))t=0 = tr t(com X)A ,
dt
cette formule devient si X est inversible (car bien sûr X −1 det(X) =t (com X)...) : ∀ X ∈
GLn (R), : tr ( X −1 A) = tr ( X −1 B) soit
qui implique A − B = 0 (en effet tr ( X(A − B)) = hX, A − Bi où hX, Y i = tr ( tXY )) est un
produit scalaire sur Mn (R)...). o
Ë o
Exercice 339 (Une fonction continue nulle part dérivable ) [5], [24].
Ê (Préliminaire) Soient I un intervalle ouvert de R, f : I → R une application
dérivable un point a ∈ I montrer que pour tout ε > 0, il existe δ > 0 tel que
f (t1 ) − f (t2 ) f (u1 ) − f (u2 )
t1 − t2 − <ε
u1 − u2
pour tous t1 < a < t2 , u1 < a < u2 dans ]a − δ, a + δ[.
Ë Soit G : R → R définie P par G(x) = dist(x, Z) ; pour n ∈ N on pose G( x) =
dist(2n x, Z) puis H(x) = n≥0 2−n Gn (x).
ê Montrer que H ∈ C 0 (R) et que la réunion des points de non dérivabilité des
fonctions Gn est dense dans R.
Soient a ∈ R, δ > 0.
ê Montrer qu’il existe k ∈ N, r ∈ Z tels que
r r+1
a − δ < x1 = k < x2 = k < a + δ
2 2
et étudier la quantité
H(ξ) − f (x1 ) H(x2 ) − f (x1 )
−
ξ − x1 x2 − x1
(où ξ = (x1 + x2 )/2) pour en déduire la non dérivabilité de H au point a.
Ëê
ê o
P P
Ê Inutile de s’attarder sur la divergences des deux séries Pn an , n bn etPl’équivalence de
leur termes généraux qui sont élémentaires ; posons An = k=1 an , Bn = nk=1 bn alors
n
(
1 si n ≡ 0(2), 1 1
An = , Bn = An + 1 + + · · · + .
0 sinon. 2 n
(An )n est donc bornée, (Bn )n tends vers +∞ : elles ne peuvent être équivalentes.
P P
Ë Les séries n cn , n dn sont convergentes à termes généraux équivalents. Posons
X X
Cn = c n , Dn = dn
k≥n k≥n
1 1 1 1
en = c2n + c2n+1 = − = ∼ 2
2n 2n + 1 2n(2n + 1) ∞ 4n
Des théorèmes de comparaisons séries/intégrales il vient
Z ∞ Z ∞
1 dt X 1 dt 1
= 2
≤ 2
≤ 2
=
4(n + 1) n+1 4t k≥n
4k n 4t 4n
Z ∞ Z ∞
2 dt X 1 dt 2
√ = √ ≤ √ ≤ √ =√
n+1 n+1 t t k≥n
k k n t t n
soit
X 1 1 X 1 2
2
∼ , √ ∼√ .
k≥n
4k ∞ 4n
k≥n
k k ∞ n
et comme en ∼ 1/4n2 , le théorème sur l’équivalence des restes des séries convergentes à
termes positifs assure que
X 1
en ∼ .
∞ 4n
k≥n
Mais, pour tout n ≥ 1,
X X 1
C2n+1 = = en ∼
k≥2n k≥n
∞ 4n
et comme
1
C2n = − + C2n+1
2n + 1
implique
C2n 1
=1− ,
C2n+1 C2n+1 (2n + 1)
soit
1
|C2n | ∼
∞ 4n
et finalement
1
|Cn | ∼ .
∞ 2n
1
De l’autre coté, dn = cn + √
n n
donne
X 1 1 2 2
Dn = Cn + √ ∼ +√ =√
k≥n
k k ∞ 2n n n
i.e. Cn = o(Dn ). o
i Remarque : La positivité est donc essentielle, les régles sont les suivantes :
P(an )n , (bn )n deux suites de nombres réels, si an > 0 à partir d’un certain rang,
– Soient
si la série n an diverge et si bn ∼ an , alors leurs sommes partielles sont équivalentes.
P(an )n , (bn )n deux suites de nombres réels, si an > 0 à partir d’un certain rang,
– Soient
si la série n an converge et si bn ∼ an , alors leurs suites des restes sont équivalentes.
où encore
xa
(8) exp(σ 2 /2M 2 ) ≤ ≤ exp(σ 2 /2m2 ).
xg
Comme 1 ≤ exp(σ 2 /2M ) on retrouve la forme classique de l’inégalité inégalité Arithmético-
Géométrique : xg ≤ xa ; en outre (8) assure que xg = xa si et seulement si σ = 0 i.e. si et
seulement si x1 = · · · = xn . o
Il ne reste plus qu’à faire tendre n vers l’infini et reconnaitre dans le second terme une somme
de Riemann : Z 1
E(Xn ) x
lim = dx = 1 − log(2).
n→∞ n 0 1+x
o
√
Exercice 344 (Irrationalité de 2) [32]
√
Donner plusieurs démonstrations de l’irrationalité de 2.
√
ê Supposons qu’il existe p, q ∈ N? tels que 2 = p/q, et soit q le plus petit de ces tels
dénominateurs. Alors 2q 2 = p2 , p2 et par conséquent p est √ pair : p = 2p1 . De là, 2q 2 = 4p21
soit q 2 = 2p21 et q est donc aussi pair. Si q = 2q1 on aura 2 = p/q = p1 /q1 et q1 < q ce qui
contredit le choix de q.
√
ê Supposons à nouveau qu’il existe p, q ∈ N? tels que 2 = p/q, et soit q le plus petit de
ces tels dénominateurs. Alors
√
2q − p 2 − (p/q) 2− 2 √
= =√ = 2,
p−q (p/q) − 1 2−1
comme 2p − q et p − q sont des entiers et 0 < p − q < q on a à nouveau une contradiction.
23Ω est la réunion disjointe de ses composantes connexes qui sont des intervalles et sont au plus dénombrable car R admet
une base dénombrable de voisinages (les intervalles à extrémités rationelles).
Exercice 346 (dist(a, ker(T )) où T est une forme linéaire continue. ) Soient E
un espace vectoriel normé, T ∈ E 0 une forme linéaire continue non identiquement
nulle sur E.
|T (a)|
Ê Montrer que pour tout a ∈ E \ ker(T ), ||| T ||| = .
dist(a, ker(T ))
Ë Dans l’espace de Banach c0 (N) des suites réelles convergentes vers 0 (muni de
la norme « sup ») on considère
Ê Pour u ∈ ker(T ) on a
|T (a)| = |T (a − u)| ≤||| T ||| ·ka − uk,
par conséquent, en passant à la borne inférieure pour u ∈ ker(T ) :
kT (a)k ≤||| T ||| ·dist(a, ker(T )),
soit
|T (a)|
||| T ||| ≤
dist(a, ker(T ))
Pour obtenir l’inégalité inverse, avec la définiton de ||| T |||, il suffit de montrer que pour
tout x ∈ E :
|T (a)|
|T (x)| ≤ · kxk.
dist(a, ker(T ))
Si T (x) = 0 il n’y a rien à démontrer, on peut donc supposer T (x) 6= 0 et l’inégalité étant
homogène en x on peut, quitte à remplacer x par TT (x) (a)
x supposer que T (x) = T (a). Alors
x − a ∈ ker(T ) qui implique dist(x, ker(T )) = dist(a, ker(T )) (l’écrire) et finalement comme
kxk ≥ dist(x, ker(T )) :
kxk kxk
|T (x)| ≤ |T (x)| · = |T (a)| ·
dist(x, ker(T )) dist(a, ker(T ))
CQFD. o
[1] M. Aigner and M. Ziegler. Proofs from the Book. Springer, 1998.
[2] E. Amar and E.Matheron. Analyse Complexe. Cassini, 2004.
[3] D. Azé., G. Constans, and J.B. Hiriart-Urruty. Exercices de Calcul Différentiel. Dunod, 2004.
[4] P. Barbe and M. Ledoux. Probabilité. EDP sciences, 2007.
[5] R.P. Boas. A primer of real functions, volume 13 of The Carus Mathematical Monograph. A.M.S., 1981.
[6] D.D. Bonar and M.J. Khoury. Real Infinite Series. Classroom Ressource Materials. M.A.A., 2006.
[7] B.R.Gelbaum and J.M.Olmsted. Counterexamples in analysis. Dover, 2003.
[8] N.N. Chentzov, D.O. Shklarsky, and I.M. Yaglom. The USSR Olympiad Problem Book. Dover, 1993.
[9] Louis Comtet. Analyse Combinatoire, T. 2, volume 13 of Collection sup. P.U.F., 1970.
[10] Revue de Mathématiques Supérieure (RMS). e.net et anciennement Vuibert, http://www.rms-math.com/.
[11] P. Ney de Souza and J.N. Silva. Berkeley Problems in Mathematics. Problem Books in Mathematics. Springer, 2000.
[12] C. Deschamp. and A. Warusfel. Mathématiques, cours et exercices corrigés (deux tomes). Dunod, 2001.
[13] J. Edward and S.Klamkin W.O.J Moser J.Barbeau, S. Murray. Five Hundred Mathematical Challenges. M.A.A., 1995.
[14] P. Eymard and J.P. Lafon. Autour du Nombre π. Hermann, 1999.
[15] S. Francinou, H. Gianella, and H. Nicolas. Exercices de Mathématiques (oraux X-ENS) : algèbre 1. Cassini, 2001.
[16] S. Francinou, H. Gianella, and H. Nicolas. Exercices de Mathématiques (oraux X-ENS) : analyse 1. Cassini, 2001.
[17] S. Francinou, H. Gianella, and H. Nicolas. Exercices de Mathématiques (oraux X-ENS) : analyse 2. Cassini, 2001.
[18] S. Francinou, H. Gianella, and H. Nicolas. Exercices de Mathématiques (oraux X-ENS) : algèbre 2. Cassini, 2006.
[19] A.M. Gleason, R.E. Greenwood, and L.M. Kelly. The William Lowell Putnam Mathematical Competition Problems and
Solutions 1938-1964. MAA Problems Books. M.A.A., 1980.
[20] C. Grunspan and E. Lanzmann. L’oral de mathématiques aux concours : Algèbre. Vuibert, 1994.
[21] P.R. Halmos. Problems for Mathematicians Young and Old, volume 12 of Dolciani Mathematical Expositions. MAA, 1991.
Il existe maintenant également une vertion française « Problèmes pour mathématiciens, petits et grands » aux éditions
Cassini (2000).
[22] P.R. Halmos. Linear Algebra Problem Book, volume 16 of Dolciani Mathematical Expositions. MAA, 1995.
[23] G.H. Hardy and E.M. Wright. Introduction à la Théorie des Nombres. Vuibert-Springer, 2007. C’est la traduction francaise
du très fameux « An Introduction to the Theory of Numbers » (Oxford University Press) 1974.
[24] B. Hauchecorne. Les Contre-exemples en Mathématiques. Ellipses, seconde édition 2007.
[25] W.J. Kaczor and M.T. Nowak. Problems in Mathematical Analysis : Continuity and Differentiation, volume 2 of Student
Mathematical Library. AMS, 2001.
[26] W.J. Kaczor and M.T. Nowak. Problems in Mathematical Analysis : Sequences and Series, volume 1 of Student Mathe-
matical Library. AMS, 2001.
[27] W.J. Kaczor and M.T. Nowak. Problems in Mathematical Analysis : Integration, volume 3 of Student Mathematical Library.
AMS, 2003.
[28] K.S.Kedlaya, B.Poonen, and R.Vakil. The William Lowell Putnam competition 1985-2000. MAA Problem Books. M.A.A.,
2002.
[29] E. Leichtnam. Exercices corrigés de Mathématiques(X, E.N.S.) : Analyse. Ellipses, 1999.
[30] Eric Leichtnam. Exercices corrigés de Mathématiques (X, E.N.S.) : Algèbre et Geométrie. Ellipses, 1999.
[31] Gérard Letac. Problèmes de Probabilité, volume 6 of Collection sup – le mathématicien. P.u.f. edition, 1970.
[32] M.Laczkovoch. Conjecture and Proof. Classroom Ressource Material. M.A.A., 2007.
[33] Jean-Marie Monier. Multiples ouvrages aux éditions Dunod.
[34] American Mathematical Monthly. M.A.A., maa@?????.fr.
[35] J.Y Ouvrad. Probabilités, volume 1 et 2. Cassini, 1998.
403
404/408 Petit Bestiaire d’Exercices pour l’Oral de l’Agrégation Interne Patrice Lassère
[36] G. Polya and G. Szegö. Problems and Theorems in Analysis, volume 1 & 2. Springer, 19 ? ? edition, 2004.
[37] H. Queffélec and C. Zuily. Éléments d’Analyse, Agrégation de Mathématiques. Dunod, 2002.
[38] J.E. Rombaldi. Analyse matricielle, Cours et exercices résolus. EDP sciences, 1999.
[39] J.E. Rombaldi. Thèmes pour l’agrégation de Mathématiques. EDP sciences, 1999.
[40] J.E. Rombaldi. .................. EDP sciences, 2004.
[41] Francois Rouvière. Petit guide de Calcul Différentiel. Cassini, 1999.
[42] Y.U. Shashkin. Fixed Points, volume 2 of Mathematical World. A.M.S., 1990.
[43] J.M. Steele. The Cauchy-Schwarz Master Class. MAA Problem Books Series. M.A.A. & Cambridge University Press, 2004.
[44] P. Tauvel. Exercices de Mathématiques pour l’Agrégation : Algèbre 2. Masson, 1994.
[45] P. Tauvel. Exercices d’Algèbre Linéaire. Dunod, 2004.
[46] Claude Wagschall. Topologie et Analyse Fonctionnelle. Hermann, 1995.
[47] Claude Wagschall. Dérivation, Intégration. Hermann, 2003.
[48] Claude Wagschall. Fonctions holomorphes, équations différentielles. Hermann, 2003.
[49] G.L. Wize and E.B. Hall. Couterexamples in Probability and Real Analysis. Oxford University Press, 1993.
405
406/408 Petit Bestiaire d’Exercices pour l’Oral de l’Agrégation Interne Patrice Lassère